WEEK 14 [ADN 210] Electrolytes and fluid balance

Réussis tes devoirs et examens dès maintenant avec Quizwiz!

You have just finished explaining the disease process of type 2 diabetes mellitus to a newly diagnosed client. Which client statement indicates that your teaching was​ successful? "My pancreas has stopped producing insulin." "I will have to monitor my weight and diet to manage this disease." "The flu I had last month triggered this disease." "I will never have to worry about giving myself insulin injections."

"I will have to monitor my weight and diet to manage this disease."

Michelle Kane is a​ 28-year-old client with a history of leukemia. She comes to your clinic seeking treatment for a recurring candidiasis infection of the mouth. Ms. Kane asks you why she cannot seem to get rid of this infection. What information should you include in your​ response? ​"This may be related to the​ chemotherapy." ​"Candidiasis is always difficult to treat​ effectively." ​"This may be related to an underlying HIV​ infection." ​"Candidiasis is not uncommon in women your​ age, so you should not be​ concerned."

"This may be related to the​ chemotherapy."

Which instructions should you give to clients who are taking ​insulin? Select all that apply. "You should carry a medical alert bracelet or card." "Monitor your glucose more frequently if you get the flu." "You should always keep a quick source of sugar on hand." "Alter the insulin dose based on how you are feeling." "You can store unopened vials of insulin up to 6 months in the freezer."

"You should carry a medical alert bracelet or card." "Monitor your glucose more frequently if you get the flu." "You should always keep a quick source of sugar on hand."

Five types of sympatholytics (aka adrenergic antagonists) :

- Alpha1 adrenergic blockers - Beta adrenergic blockers - Adrenergic neuron blockers - Centrally acting alpha2 agonists - Alpha/beta blockers

Four categories of drugs for treatment of Heart Failure:

- Diuretics - Cardiac glycosides - Sympathomimetics - Phosphodiesterase Inhibitors

Heparin

- anticoagulant. - enhances the action of antithrombin III. -Routes of administration used are: Intravenous Subcutaneous -The treatment for heparin overdose is protamine sulfate.

Four types of RAAS suppressants:

-ACE inhibitors (hypertension, HF, direct nephropathy, and left vent dysfunction following MI) (captopril) -ARBs -Aldosterone antagonists -Direct renin inhibitors

The two etiologies of coagulation disorders are:

-Decreased numbers of platelets, which are essential to clotting. -Deficiencies of one or more clotting factors, leading to delayed coagulation.

Venous thromboembolism (VTE)

-Generally occurs when blood flow through a vein is slowed. -Allows procoagulants to accumulate and overpower natural anticoagulation elements in blood. -common -Two disorders of VTE: Deep vein thrombosis and pulmonary embolus

Four categories of drugs that treat hypertension:

-RAAS suppressants -Calcium channel blockers -Sympatholytics -Direct-acting vasodilators

Deep vein thrombosis (DVT)

-The presence of thrombi in the legs. -Largely due to slower blood flow in lower extremeties. (Immobility, major trauma, surgery, hypercoagulable states, and drug therapy)

Hemostatics (coagulation modifier)

-also known as antifibrinolytics, -used to shorten bleeding times. -increase the stability of clots and prevent excessive bleeding. -bind with sites on plasminogen and plasmin, which prevents the destruction of clots.

Thrombolytics (coagulation modifier)

-mechanism of action that promotes fibrinolysis by converting plasminogen to plasmin. These drugs dissolve existing clots. -Used to treat acute MI, pulmonary embolism, acute ischemic stroke (CVA), and DVT.

Warfarin sodium (Coumadin Sodium)

-the most commonly prescribed oral anticoagulant. -mechanism of action is to inhibit the synthesis of clotting factors II, VII, IX, and X in the liver. -The route of administration for warfarin is oral. -The antidote for warfarin overdose is Vitamin K.

The nurse is assessing an adult client with fluid loss from diarrhea and vomiting. Which tests would indicate that the client is actually​ dehydrated? Select all that apply. 1) Serum osmolality 2) Urine specific gravity 3) Hemoglobin and hematocrit 4) Serum electrolytes 5) Chest​ x-ray

1) Serum osmolality 2) Urine specific gravity 3) Hemoglobin and hematocrit 4) Serum electrolytes

The nurse is educating an adult client on the causes of metabolic acidosis. Which causes would the nurse include in the client ​education? Select all that apply. 1) Tissue hypoxia ​2) Potassium-wasting diuretics 3) Chronic renal failure 4) Acute diarrhea 5) Diabetes mellitus

1) Tissue hypoxia 3) Chronic renal failure 4) Acute diarrhea 5) Diabetes mellitus

Arrange the steps of an initial acute inflammatory response in the correct order.

1. pathogen invades 2. chemical mediators are released 3. nearby blood vessels dilate 4. capillaries become permeable 5. proteins enter affected area 6. pathogen is destroyed 7. damage is repaired

John Adams is a​ 26-year-old male client who was brought to your emergency room following a motor vehicle accident that fractured his left femur. He is pale and in obvious pain. His vital signs are heart rate of 144​ beats/min, blood pressure​ 90/45, and respiratory rate 28​ breaths/min. What do you expect to​ administer? Normal saline​ @ 50​ mL/hr 2 units packed RBCs over 2 hours each ​D5-LR @ 250​ mL/hr ​Plasma-Lyte 56​ @ 100​ mL/hr

2 units packed RBCs over 2 hours each

An older male client is at risk for developing a fluid deficit because he is unaware of the causes of dehydration. When teaching the client about​ dehydration, which causes would the nurse ​include? Select all that apply. 1) Hormone therapy 2) Profuse sweating 3) Loss of a large amount of blood 4) Nausea leading to​ vomiting, and diarrhea 5) Abuse of diuretics

2) Profuse sweating 3) Loss of a large amount of blood 4) Nausea leading to​ vomiting, and diarrhea 5) Abuse of diuretics

The nurse assesses the skin of a newly admitted client. Findings include a 3 cm area with partial thickness dermis loss on the client​'s sacral area. Which documentation is an accurate description of the​ assessment? A stage IV ulceration 3 cm on sacral area 3 cm stage II pressure ulcer sacral area A stage I ulcer 3 cm in diameter on sacral region Stage III pressure ulcer noted on sacrum

3 cm stage II pressure ulcer sacral area

The nurse is assessing a client with alkalosis who is being treated with acid agents. What adverse effects might the nurse observe when treating this client with ammonium​ chloride? 1) Yellow skin color 2) Decreased respiratory rate 3) Central nervous system depression 4) Nausea and vomiting

3) Central nervous system depression

The nurse is educating an older client who has been diagnosed with tuberculosis. What is the minimum time the client can expect to take antibiotic medications for this​ illness?

6-12 months

How long should clients who are diagnosed with tuberculosis expect to take antituberculosis​ medications? 10 days 3 months Remainder of life 6-24 months

6-24 months

Where is​ two-thirds of the​ body's total fluid​ located? A) Intracellular space B) Interstitial space C) Extracellular space D) Intravascular space

A About​ two-thirds of total body fluid is found in the intracellular​ space, located within the cells. About​ one-third of total body fluid is located in the extracellular space outside the cells. The intravascular and interstitial spaces are part of the extracellular space.

Tommy Powell and his mother have come to your clinic. Tommy is a​ 6-year-old who presents with symptoms of chickenpox. What is the drug of choice for controlling​ Tommy's fever and​ pain? Acetaminophen​ (Tylenol) Morphine sulfate​ (Duramorph) Aspirin​ (Bayer) Celecoxib​ (Celebrex

A Acetaminophen​ (Tylenol) is the drug of choice for​ infants, children, and adolescents who present with​ flu-like symptoms or chickenpox. Unlike aspirin​ (Bayer), acetaminophen is not known to place young clients at risk for Reye syndrome. Although celecoxib​ (Celebrex) is effective in decreasing pain and​ fever, celecoxib is generally given to​ adults, not children. Morphine sulfate​ (Duramorph), though effective for​ pain, is not indicated for the initial management of chickenpox.

Rosa​ Dillon, a​ 10-year-old client, is in the hospital following a car accident. She has sustained two broken​ ribs, along with cuts and bruises. As you clean and dress her​ wounds, Rosa worries because they are so swollen. As her​ nurse, what should you do to address​ Rosa's concerns? Explain how inflammation helps her body heal. Consult with her admitting nurse. Explain to her how the lymphatic system works. Assess for relief of symptoms.

A As a​ nurse, an important part of your role is educating your clients at their level of understanding. Most​ 10-year-olds can understand that swelling helps protect and repair the​ body, but perhaps not how the lymphatic system works. Barring other​ issues, there is no need to consult with​ Rosa's admitting nurse. While you are always assessing for relief of​ symptoms, your first goal in this case is calming​ Rosa's concern about inflammation.

What type of ion is released by the lungs to help maintain the correct pH level in the​ body? A) Carbon dioxide B) Phosphate C) Bicarbonate D) Hydrogen

A Carbon dioxide is an acid ion released by the lungs during exhalation. Hydrogen ions are acid ions released by the kidneys in the urine. Bicarbonate and phosphate ions are part of the buffer system within the body that help maintain an optimal pH level by neutralizing acids and bases.

Which​ anti-inflammatory drug inhibits only one type of​ cyclooxygenase? Celebrex Motrin Nuprin Ecotrin

A Celecoxib​ (Celebrex) inhibits only one type of the enzyme​ cyclooxygenase, COX-2.​ However, ibuprofen​ (Advil, Motrin,​ Nuprin) and aspirin​ (Bayer, Ecotrin) elicit a change in both types of​ cyclooxygenase, COX-1 and​ COX-2.

Your client is a​ 65-year-old Asian American​ male, Ben Chang. His initial ECG revealed atrial fibrillation with a ventricular rate of 117​ beats/min. If the​ beta-adrenergic antagonist propranolol​ (Inderal) is prescribed for Mr.​ Chang, what will your main consideration be as his​ nurse? A) Assessment for bradycardia B) Assessment for pulmonary toxicity C) Teaching Mr. Chang to wear protective clothing D) Teaching Mr. Chang to avoid crowds while taking medication

A Clients of Asian descent may metabolize propranolol​ (Inderal) more quickly. As the​ nurse, you should assess your Asian American clients for bradycardia and signs of adverse reactions from high drug levels. Photosensitivity and pulmonary toxicity are adverse effects of amiodarone​ (Cordarone), not propranolol. Avoiding crowds is an instruction given to clients whose immune systems are suppressed.

What is the process by which the body rids itself of a​ clot? A) Fibrinolysis B) Coagulation C) Hemophilia D) Hemostasis

A Fibrinolysis is the process by which the body rids itself of a clot. Hemostasis is the process used by the body to stop bleeding after an injury. Coagulation is the balance between anticoagulants​ (agents that prevent​ clotting) and procoagulants​ (agents that promote​ clotting). Hemophilia is the disease process related to missing clotting factors.

Ms. Riveria came to the clinic today to discuss her lab results of 2 weeks ago. Her cholesterol was 220​ mg/dL and she had a triglyceride level of 344​ mg/dL. As the​ nurse, you explain that lifestyle changes can affect lipid levels. Which explanation about lifestyle modifications is​ best? A) Increase your dietary soluble fiber and decrease your intake of fats. B) Lose at least 25 pounds to eliminate the need for medication. C) Exercise will reduce lipid levels without other lifestyle modifications. D) Limit your use of tobacco until your triglyceride levels improve.

A Increasing fiber and limiting fats will help reduce lipid levels. Limiting tobacco​ use, losing​ weight, and exercising will​ help, but it is not likely that any of these modifications by itself will be enough to successfully reduce lipid levels.​ Often, medication and multiple lifestyle changes are needed to achieve healthy lipid levels.​ Also, lifestyle changes should be​ ongoing, not temporary.

You are caring for Gerald​ Miller, a​ 70-year-old male, admitted for a total hip replacement. You are educating him about DVT. Which is a key risk factor associated with placing Mr. Miller on DVT​ precautions? A) Hip replacement surgery diminishes mobility. B) This is Mr.​ Miller's second total hip replacement. C) Mr. Miller currently takes no home medications. D) Mr. Miller has a family history of blood clots.

A Major surgery and immobility place a person at high risk for developing​ DVT, due to tissue trauma and venous stasis. A family history of blood clots is not a reason for DVT precautions. Neither is the fact that Mr. Miller takes no home medications. A second hip replacement also does not increase his risk for DVT.

Brandt​ Chamberlain, age​ 47, is brought to your acute care clinic by his son. Mr. Chamberlain is having difficulty breathing that worsens when he lies down. He is also coughing up a bloody froth. What should you do​ first? A) Provide emergency medical care B) Take the​ client's medical history C) Perform an electrocardiogram D) Place the client on a cardiac monitor

A Mr.​ Chamberlain's symptoms indicate pulmonary​ edema, a buildup of fluid in the lungs. Pulmonary edema is a possible complication of​ left-sided heart failure that requires immediate emergency medical care. Performing​ electrocardiography, placing the client on a cardiac​ monitor, and taking the​ client's medical history are all appropriate for a client with possible heart​ failure, but they are not the first priority.

The nurse is caring for a client who is receiving spironolactone​ (Aldactone) for heart failure. The client​'s potassium level is 6.2​ mEq/L (normal values 3.5dash-5.3 ​mEq/L). Which should be the nurse​'s priority to​ assess? A) Electrocardiography​ (ECG) results B) Mental status C) Bowel sounds D) Respiratory rate

A Rationale A potassium level of 6.2​ mEq/L (normal values 3.5dash-5.3 ​mEq/L) indicates​ hyperkalemia, which can lead to lethal dysrhythmias that can be detected by electrocardiography. Other signs and symptoms of hyperkalemia include muscle​ weakness, fatigue, and bradycardia. It is not appropriate to assess respiratory​ rate, bowel​ sounds, or mental status for effects of hyperkalemia.

The nurse is teaching a client about normal cardiac function. What should the nurse tell the client is the rate at which electric impulses originate from the SA​ node? 60 to 100 times per minute 70 to 120 times per minute 80 to 140 times per minute 50 to 80 times per minute

A Rationale Electric impulses originate from the SA node at a rate of 60 to 100 times per minute. Electric impulses do not originate from the SA node at a rate of 50 to 80 times per​ minute, 70 to 120 times per​ minute, or 80 to 140 times per minute.

The​ 57-year-old client has been prescribed the​ "drug of​ choice" for treating her severe hypertriglyceridemia. A drug from which class has been​ selected? A) Fibric acid agents B) Nicotinic acid​ (Niacor, Niaspan), also called niacin C) Bile acid resins D) Cholesterol absorption inhibitors

A Rationale Fibric acid agents are the drugs of choice for treating severe hypertriglyceridemia. Other medications lower​ cholesterol; however, they are not the drugs of choice for severe hypertriglyceridemia. Cholesterol absorption inhibitors prevent cholesterol from being absorbed. Nicotinic acid​ (Niacor, Niaspan), also called​ niacin, is occasionally used to lower lipid levels. Bile acid resins are often​ used, along with​ statins, for control of cholesterol to achieve maximal results.

The​ client, who recently suffered a myocardial​ infarction, tells the nurse he has never heard of glycoprotein​ IIb/IIIa inhibitors and asks what this medication does and why it has been prescribed for him. Which response by the nurse answers the client​'s ​questions? A) This medication prevents blood clots from developing. They are often given to clients who are undergoing angioplasty. B) This medication is given only to selected clients at high risk for serious blood clot formation. C) This medication decreases the size of blood clots. That improves the blood flow to the heart. D) This particular medication​ (glycoprotein IIb/IIIa​ inhibitors) is not recommended for clients who have low platelet counts.

A Rationale Glydoprotein​ IIb/IIIa inhibitors prevent blood clots from developing. They are often given routinely to clients who are undergoing angioplasty. That medication does not decrease the size of blood​ clots, and is not given only to selected clients at high risk for serious blood clot formation. While that medication is not given to clients with low platelet​ counts, that fact does not answer the client​'s question about its indication and use in this client.

The​ client, recently diagnosed with myocardial infarction​ (MI), will be discharged in 48 hours. Which action is included in the planning phase of the nursing process in relation to medication​ administration? A) Make sure the client is able to describe the mechanism of action and list at least four of the most common adverse effects for all of the medications prescribed at discharge. B) Determine what medications the client has taken for pain in the past. C) Teach the client about adverse effects of​ vasodilators; for​ example, teach about orthostatic hypotension and the importance of changing positions slowly​ (sitting to​ standing) to prevent​ dizziness, lightheadedness, and fainting. D) Determine if the client is allergic to any medications.

A Rationale In the planning stage in relation to medication​ administration, the nurse is responsible for organizing​ (planning) nursing care to ensure that the client is able to verbalize an understanding of prescribed​ medications, including their​ use, action, adverse​ effects, and precautions. Determining what medications the client has taken for pain in the​ past, and obtaining baseline assessment data such as the client​'s medication​ allergies, are part of the assessment phase of the nursing process. Teaching a client about adverse effects of medications is part of the implementation phase of the nursing process in relation to medication administration.

The nurse is assessing a client with tonsillitis. The client asks the nurse why the tissues in the neck seem swollen. Which nursing response is​ best? ​A) "Your lymph nodes and tissues sometimes swell in attempts to fight​ infection." B) ​"The swelling is a direct effect of histamine being released throughout the​ body." C) ​"T cells are activated in the neck​ area, which causes the neck to​ swell." D) ​"The neck area contains proteins that collect and cause the​ swelling."

A Rationale Lymph tissues and nodes located throughout the body filter pathogens during​ infection, and may swell because of this action. This response by the nurse answers the question best. It is not true that proteins collect and cause​ swelling; proteins that inhibit infection are located in the mucous membranes. Histamines are released to start the inflammatory​ process, but the histamine does not cause the swelling. T cells are a part of the​ cell-mediated immune​ response, which is not related to the swelling of the lymph tissue.

The nurse is caring for a client who is experiencing digoxin toxicity. Which sodium channel blocker does the nurse anticipate will be ordered to treat the dysrhythmias associated with this​ condition? Phenytoin​ (Dilantin) Lidocaine​ (Xylocaine) Mexiletine​ (Mexitil) Procainamide​ (Pronestyl)

A Rationale Phenytoin​ (Dilantin) is the medication that is often prescribed to treat the dysrhythmias associated with digoxin toxicity. Lidocaine​ (Xylocaine), mexiletine​ (Mexitil), and procainamide​ (Pronestyl) are not prescribed to treat such dysrhythmias.

The nurse is caring for a group of clients on a​ medical-surgical unit. Which client is at risk for a deep venous thrombosis​ (DVT) due to a hypercoagulable​ state? A) A pregnant client admitted for a kidney stone B) A client who receives hormone replacement therapy due to menopause C) A client who is immobile due to a spinal cord injury D) A client who has just returned from major surgery

A Rationale Pregnancy is a hypercoagulable disorder that puts a client at risk for DVT. Clients who have just had major​ surgery, are​ immobile, or are on hormone replacement therapy are at risk for a​ DVT, but the increased risk is not because of a hypercoagulable disorder.

The​ nurse, who is caring for a client diagnosed with a​ bradydysrhythmia, reassures the client that her condition is the most common type of heart rate irregularity. What type of bradydysrhythmia does the client​ have? A) Sinus bradycardia B) Sick sinus syndrome C) Atrial flutter D) Atrioventricular conduction block​ (AV block)

A Rationale Sinus bradycardia is the most common bradydysrhythmia. Both sick sinus syndrome and atrioventricular conduction block​ (AV block) are also​ bradydysrhythmias, but they are not the most common. Atrial flutter is a tachydysrhythmia.

The nurse is teaching the client about stable​ angina, which she describes as​ "predictable." The client asks what predictable means. The nurse explains that​ "predictable" means that the same amount of exercise or activity may cause the angina to occur. What should the client be told to expect when the exercise is stopped or slowed​ down? A) The client​'s symptoms should improve or go away. B) The client will require emergency medical treatment. C) The client​'s symptoms will occur even at rest and may recur at night. D) The angina will recur with increasing​ frequency, severity, and duration.

A Rationale Stable angina is chest pain or discomfort that often occurs with activity or stress. Symptoms of stable angina are often​ predictable, which means that the same amount of exercise or activity may cause the angina to occur. Pain from stable angina should improve or go away when the exercise or activity is stopped or slowed down. In​ contrast, myocardial infarction​ (MI), commonly referred to as a heart​ attack, results in myocardial cellular necrosis​ (heart-muscle cell​ death) and is a​ life-threatening event that requires emergency medical treatment. Vasospastic angina or Prinzmetal​'s ​angina, caused by coronary artery spasm with or without the presence of​ atherosclerosis, is unpredictable and not related to​ activity; it often occurs at rest and may occur at night. Unstable angina occurs with increasing​ frequency, severity, and​ duration, and may occur at rest. Clients with unstable angina are at higher risk for MI.

The nurse is caring for a client who was recently diagnosed with vasospastic ​(Prinzmetal​'​s) angina. Which nursing action is included in the implementation phase of the nursing process in relation to medication administration with this​ client? A) Teach the client about the medications​ he/she is taking​ (medication self-administration, mechanism of​ action, and adverse​ effects). B) Interview the client to determine when the angina​ occurs, if the pain occurs in​ clusters, and if there are any identifiable triggers that lead to the chest pain. C) Determine if the client goal of listing two or three of the most common adverse effects of the medications he is taking was met. D) Review the client​'s medical chart to make sure the results of the angiogram​ (a diagnostic test that produces an​ x-ray of the heart​'s arteries on a​ monitor) are placed in the correct section of the client​'s chart.

A Rationale The implementation phase of administering medications for the client with vasospastic ​(Prinzmetal​'​s) angina includes teaching the client about medication​ use, mechanism of​ action, and adverse effects. During the assessment stage of the nursing​ process, the nurse is responsible for obtaining baseline assessment​ data, which includes interviewing the client to determine when the angina​ occurs, if the pain occurs in​ clusters, and if there are any identifiable triggers that lead to the chest pain. Reviewing the chart for proper placement of documentation is not part of the nursing process. Determining if the client goal of listing two or three of the most common adverse effects of the medications​ he/she is taking was met is a component of the evaluation phase of the nursing process.

The nurse is caring for a client who has been placed on anticoagulant therapy. Which intervention would the nurse question for a client at risk for​ bleeding? A) Vitamin K medication order B) ​Keep-open IV C) Cardiac monitoring D) Foley catheter insertion

A Rationale The nurse would question the Vitamin K medication​ order, since Vitamin K is not used in treating a client at risk for bleeding.​ Instead, Vitamin K treats overdoses of warfarin​ (Coumadin). Insertion of a Foley catheter may be ordered to monitor intake and output. A cardiac monitor may be ordered to monitor the client for dysrhythmias.​ Keep-open IVs may be ordered for future medication administration.

The nurse is providing client education on antihypertensive medications. What teaching does the nurse include regarding what may happen if the client suddenly stops taking the​ medication? A) Blood pressure may rebound. B) Blood pressure will not exceed previous high levels. C) Blood pressure may become dangerously low. D) Blood pressure will remain within normal limits.

A Rationale When antihypertensive medication is stopped​ abruptly, rebound hypertension can​ occur, sending the blood pressure even higher than it was before medication therapy was started.

Don Morey presents to the urgent care clinic with tightness in his​ chest, shortness of​ breath, and anxiety. He reports that he was rearranging rocks in his garden. Mr. Morey shares that he felt better after resting in his​ armchair, but his daughter insisted that he see a health care provider. As the​ nurse, you recognize that Mr.​ Morey's symptoms may be related to​ what? A) Vasospastic angina B) Stable angina C) Unstable angina D) Silent angina

A Stable angina occurs with activity or​ stress, due to an increased myocardial oxygen demand. It is relieved by rest and nitrates. Vasospastic angina is not related to activity and often occurs at rest. Unstable angina may take place at​ rest, but it occurs with increasing​ frequency, severity, and duration. Silent angina occurs when the client has an episode of myocardial ischemia but does not experience pain or chest tightness.

Angela Degrasso is a​ 34-year-old noncompliant type 1 diabetic female. She presents to your emergency room with a glucose level of 450​ (normal range is​ 80-120), lethargy, and a respiratory rate of 32​ breaths/min. Laboratory analysis reveals a serum pH level of 7.28. Based on her history and this lab​ result, what is the most likely cause of her​ symptoms? A) Metabolic acidosis B) Respiratory acidosis C) Metabolic alkalosis D) Respiratory alkalosis

A The​ body's normal pH is​ 7.35-7.45. A pH of 7.28 represents an acidotic condition. Diabetic ketoacidosis is a common cause of metabolic acidosis and the contributing cause in this case. Although a respiratory rate of 32 may often indicate respiratory​ alkalosis, Ms.​ Degrasso's pH lab result confirms an acidotic condition. Both respiratory alkalosis and metabolic alkalosis are associated with a pH level greater than 7.45.

Your​ 38-year-old client, Jack​ Hageman, is undergoing angioplasty in your hospital. Mr. Hageman is prescribed tirofiban​ (Aggrastat) via IV. What is one of the potential adverse effects of this​ drug? A) Thrombocytopenia B) Hypokalemia C) Paresthesia D) Muscle tremors

A Tirofiban hydrochloride​ (Aggrastat) is in the class of antiplatelet drugs called glycoprotein​ IIb/IIIa inhibitors. Adverse effects of drugs in this class include hypotension secondary to blood​ loss, abnormal​ bleeding, and thrombocytopenia. Muscle​ tremors, hypokalemia, and paresthesia are not adverse effects associated with tirofiban.

What is the only type of lipid that serves as an energy​ source? A) Triglycerides B) Steroids C) Cholesterol D) Phospholipids

A Triglycerides are the main storage form of fat. They are the only type of lipid that serves as an energy source. Phospholipids are essential to building plasma membranes.​ Steroids, such as​ cholesterol, are a form of lipid.

Yvette Robles is a​ 65-year-old angioplasty client with coronary artery disease​ (CAD). Before being discharged from the​ hospital, she is prescribed clopidogrel​ (Plavix). Mrs. Robles also takes ibuprofen​ (Advil) on occasion for headaches. During the discharge​ teaching, what should you verify that Mrs. Robles​ understands? A) When to contact her health care provider B) When to adjust her two medications C) How to use sublingual nitroglycerin​ (Nitrostat) D) How to practice healthy heart living

A When clopidogrel​ (Plavix), an antiplatelet​ drug, is taken with ibuprofen​ (Advil), the risk for bleeding events increases.​ Thus, Mrs. Robles needs to know when to contact her health care provider to report adverse​ effects, such as any signs or symptoms of GI bleeding. Mrs. Robles does not have a need for sublingual nitroglycerin​ (Nitrostat). A health care​ provider-not Mrs.​ Robles-should determine when to adjust any medications. Although lifestyle changes for healthy heart living should be​ discussed, this education does not have as high a priority.

What does the nurse need to know before administering oral enteral ​nutrition? Select all that apply. That the client will need less direct nursing supervision That the client is able to swallow That a central line must be placed That the client will adhere to the feeding plan That a gastrostomy tube must be in place first

A B D

The nurse is reviewing the use of acetylsalicyclic acid​ (aspirin [Bayer]) in the client population. Which clients could benefit from the use A client with mild back pain after a fall An adult client with a fever A client with inflammatory arthritis . A client with gastrointestinal bleeding A client with primary dysmenorrhea

A client with mild back pain after a fall An adult client with a fever A client with inflammatory arthritis A client with primary dysmenorrhea

Which client would the nurse expect to receive total parenteral nutrition​ (TPN)? A client​ post-surgery for repair of knife wounds to the gastrointestinal system A client who has metastatic cancer and​ malnutrition, and is in hospice A client who is​ post-lobectomy with chest tube placement A client diagnosed with acute gastroenteritis and dehydration

A client​ post-surgery for repair of knife wounds to the gastrointestinal system

Which client would the nurse expect to receive total parenteral nutrition​ (TPN)? A client who has metastatic cancer and​ malnutrition, and is in hospice A client​ post-surgery for repair of knife wounds to the gastrointestinal system A client who is​ post-lobectomy with chest tube placement A client diagnosed with acute gastroenteritis and dehydration

A client​ post-surgery for repair of knife wounds to the gastrointestinal system

Thrombus

A clot that is stationary in a blood vessel

Ms. Green is a​ 68-year-old female. Her​ sister, who is also her​ caregiver, reports that Ms.​ Green's skin has been increasingly dry and flaky. The sister asks the nurse to recommend care for the skin changes. She says she has been using powder to keep the skin dry and smelling clean. What products should the nurse recommend to address the dry​ skin? Soap and water daily on the skin during bathing Continue to use the powder A moisturizing lotion or cream An​ alcohol-based spray

A moisturizing lotion or cream

Embolus

A thrombus that has broken free and travels in the blood vessels

The nurse is taking care of an adult client in the emergency department who was in a motor vehicle crash and is bleeding profusely. Which intravenous fluids will be administered to the client to regulate the body fluid imbalance caused by the profuse​ bleeding? A) Blood products B) Oral fluids C) Hypotonic fluids D) Hypertonic fluids

A) Blood products

The nurse is educating an adult client on the causes of metabolic acidosis. Which causes would the nurse include in the client ​education? A) Chronic renal failure B) Tissue hypoxia C) Acute diarrhea ​D) Potassium-wasting diuretics E) Diabetes mellitus

A) Chronic renal failure B) Tissue hypoxia C) Acute diarrhea E) Diabetes mellitus

The nurse is caring for a client with swallowing difficulties who is expected to require enteral nutrition for 6dash-8 weeks. What type of tube would the nurse expect to be used for this​ client? Nasoduodenal tube Nasogastric tube Percutaneous endoscopic gastrostomy tube Nasojejunal tube

Percutaneous endoscopic gastrostomy tube

The nurse is caring for four clients in the emergency department​ (ED). Based on the information​ available, the nurse would intervene to help correct the​ acid-base balance for which client​ first? • Rob is in the ED with fever​ (T 100.3°F​ [37.9°C]), c/o chills and fatigue x 1​ day, and poor appetite. (Blood pH: 7.55) • Darla is in the ED on a ventilator in respiratory failure. (Blood pH: 6.89) • Sue is in the ED for abdominal cramping and diarrhea. (Blood pH: 7.46) • Kevin was brought to the ED for a laceration to his foot. (Blood pH: 7.36) A) Darla B) Rob C) Sue D) Kevin

A) Darla

The nurse is assessing an adult client who has been prescribed sodium bicarbonate as an antacid. Which side effects would the nurse discuss with the ​client? A) Decreased respiratory rate B) Irritability C) Mood elevation D) Electrolyte imbalances E) Confusion

A) Decreased respiratory rate B) Irritability D) Electrolyte imbalances E) Confusion

The nurse reviews a client​'s diagnostic test results. Which lab values indicate that the client is experiencing a fluid volume​ deficit, or ​dehydration? A) Elevated hematocrit and serum osmolality B) Decreased serum sodium and hematocrit level C) Increased urine specific gravity and serum osmolality D) Increased serum sodium and elevated hematocrit level E) Increased central venous pressure and decreased serum sodium level

A) Elevated hematocrit and serum osmolality C) Increased urine specific gravity and serum osmolality D) Increased serum sodium and elevated hematocrit level

The nurse is assessing an adult client with fluid loss from diarrhea and vomiting. Which tests would indicate that the client is actually​ dehydrated? A) Hemoglobin and hematocrit B) Urine specific gravity C) Chest​ x-ray D) Serum osmolality E) Serum electrolytes

A) Hemoglobin and hematocrit B) Urine specific gravity D) Serum osmolality E) Serum electrolytes

The nurse is caring for an adult client with hypomagnesemia. Which factors does the nurse recognize as causes of this ​disorder? A) Kidney failure B) Laxative abuse C) Diarrhea and vomiting D) Magnesium supplements E) Loop diuretic therapy

A) Kidney failure B) Laxative abuse C) Diarrhea and vomiting E) Loop diuretic therapy

An older male client is at risk for developing a fluid deficit because he is unaware of the causes of dehydration. When teaching the client about​ dehydration, which causes would the nurse ​include? A) Nausea leading to​ vomiting, and diarrhea B) Abuse of diuretics C) Hormone therapy D) Profuse sweating E) Loss of a large amount of blood

A) Nausea leading to​ vomiting, and diarrhea B) Abuse of diuretics D) Profuse sweating E) Loss of a large amount of blood

Prior to administering fluid replacement​ therapy, the nurse is assessing a child for signs of dehydration. Which signs and symptoms indicate that the child is dehydrated and may need IV ​fluids? A) Poor skin turgor B) Dry mouth and mucous membranes C) Decreased urine output D) Increased urine specific gravity E) Increase in weight

A) Poor skin turgor B) Dry mouth and mucous membranes C) Decreased urine output D) Increased urine specific gravity

A client is experiencing a nonacute fluid volume deficit after walking to a nearby clinic for an appointment on a very warm summer day. The client feels slightly thirsty but does not feel lightheaded or have other problems. The nurse monitors the client​'s blood pressure and finds it to be slightly low​ (100/72 mmHg). To efficiently and comfortably bring the client​'s fluid volume back to a more normal​ level, which intervention would the nurse​ implement? A) Teaching the client to drink approximately​ 2,500 mL of water per day B) IV administration of an isotonic solution C) Inserting a feeding tube and administering fluids via the feeding tube D) IV administration of a hypertonic solution

A) Teaching the client to drink approximately​ 2,500 mL of water per day

Hemophilia

a heredity disorder that is caused by a deficiency of one of the clotting factors. -The primary treatment of hemophilia is replacement of the missing clotting factor, either to prevent a bleed or to treat a bleed.

The nurse is administering blood products to a client who was admitted to the emergency department following a motor vehicle crash. Which assessment findings indicate adverse reactions to the blood ​product? A) The nurse notes that the client is developing slurred speech. B) The client reports that he has to urinate. C) The client reports that he is feeling lightheaded and​ "itchy." D) The client reports that his​ face, neck, and upper chest are red and feel warm. E) The client reports that he feels anxious and is beginning to have difficulty breathing.

A) The nurse notes that the client is developing slurred speech. C) The client reports that he is feeling lightheaded and​ "itchy." D) The client reports that his​ face, neck, and upper chest are red and feel warm. E) The client reports that he feels anxious and is beginning to have difficulty breathing.

Plant lipids cause the body to excrete cholesterol and lower LDL levels. Which of these foods are considered sources of plant ​lipids? Select all that apply. A) Rice B) Nuts C) Potatoes D) Corn E) Olive oil

A, B D, E ​Corn, rice,​ nuts, and olive oil are all considered to be sources of plant​ lipids, as are​ oats, wheat, and rye. Potatoes are not considered a plant lipid source because there is limited fat in potatoes.

The nurse is reviewing a hospital memo regarding the proper storage of monoclonal antibodies. Which drugs should be stored under the conditions prescribed in the ​memo? Select all that apply. ​A) Muromonab-CD3 (Orthoclone​ OKT3) B) Basiliximab​ (Simulect) C) Daclizumab​ (Zenapax) D) Thalidomide​ (Thalomid) E) Mycophenolate mofetil​ (CellCept)

A, B, C Rationale Basiliximab​ (Simulect) is a monoclonal antibody and a preventive for kidney transplant rejection. Daclizumab​ (Zenapax) is classified as an MAB and has immunosuppressant properties. Mycophenolate mofetil​ (CellCept) is not classified as an​ MAB, but it can be given to prevent​ heart, kidney, and liver transplant rejection. Thalidomide​ (Thalomid) is not a​ MAB; it is used to treat leprosy and refractory​ Crohn's disease.​ Muromonab-CD3 (Orthoclone​ OKT3) is an MAB that is given to prevent kidney transplant rejection and is administered by the IV route.

What discharge instructions should the nurse give to a client who is prescribed digoxin​ (Lanoxin) therapy for heart ​failure? Select all that apply. A) Report any changes in vision to the health care provider. B) Consume foods high in potassium. C) Call the health care provider if your heart rate is above 110 beats per minute. D) Call the health care provider if your heart rate is below 80 beats per minute. E) If you miss one​ dose, take an extra dose.

A, B, C Rationale The nurse should instruct the client​ to: bullet• Report to the health care provider if the heart rate falls below 60 or rises above 110 beats per​ minute, or if there are skipped beats or changes in rhythm. bullet• Consume foods high in potassium if appropriate and not​ contraindicated, because low potassium levels increase the risk of digoxin toxicity bullet• Report any changes in vision to the health care​ provider; signs of digoxin toxicity include​ bradycardia, nausea and​ vomiting, anorexia,​ depression, changes in level of​ consciousness, and visual changes. bullet• Never take an extra dose of digoxin​ (Lanoxin) if a dose is missed.

The nurse is preparing a presentation for a group of newly hired nurses to give an overview of immunostimulants and the immune response system. Which points would the nurse include in the ​presentation? Select all that apply. A) Immunomodulators are drugs that increase the immune response in the body. B) Cytokines are the chemicals that help facilitate the body​'s immune response. C) The immune response protects the body against invading organisms or agents. D) ​T-cell lymphocytes are responsible for creating​ antibodies, used in the development of​ long-term immunity against antigens. E) ​B-cell lymphocytes regulate the body​'s immune response by releasing cytokines.

A, B, C Rationale When a foreign substance enters the​ body, the immune system recognizes it as​ "non-self" and an immune response is triggered. Immunomodulators are drugs that affect the immune system​'s ability to defend the​ body; an immunostimulant is a type of immunomodulator. When a​ non-self organism is​ recognized, cell-to-cell communication to start an attack on the organism is accomplished by cytokine secretion.​ T-cell lymphocytes regulate the immune response by releasing cytokines.​ B-cell lymphocytes create antibodies used in the development of​ long-term immunity against antigens.

Which laboratory data are helpful in your baseline assessment of angina and MI ​issues? Select all that apply. A) Troponins B) Electrolytes C) CK D) Lipid studies E) ESR sed rate

A, B, C, D As a​ nurse, you may be responsible for obtaining a baseline assessment that includes lab data for​ lipids, creatine kinase​ (CK), electrolytes, and troponins. A sed rate or erythrocyte sedimentation rate​ (ESR) blood test reveals inflammatory activity in the body and would not be automatically ordered for an angina or MI client.

What body functions do electrolytes ​affect? Select all that apply. A) Neurologic activity B) Muscle function C) Water balance D) Bone formation E) Hunger

A, B, C, D Maintaining a balance of electrolytes is essential to the body​'s homeostasis. Electrolytes affect multiple functions in the​ body, including muscle​ function, neurologic​ activity, water​ balance, and bone formation. Although anorexia and changes in appetite are often clinical manifestations associated with electrolyte​ imbalances, electrolytes do not directly affect hunger.

The nurse is assessing an adult client for the first time. Which tasks does the nurse perform to assess for appropriate pharmacologic therapy as part of the treatment for ​hypertension? Select all that apply. A) The nurse assesses the client​'s readiness for learning. B) The nurse obtains a complete set of vital signs. C) The nurse takes a complete health history. D) The nurse obtains the client​'s history of both prescription and​ over-the-counter medications. E) The nurse performs an ultrasound.

A, B, C, D Rationale The nurse should obtain a complete health​ history, including any cardiac problems and any recent cardiac events. Baseline vital signs should be taken before medications for hypertension are administered. The nurse should assess the client​'s readiness for education about medications and lifestyle​ changes, including any barriers to compliance​ (such as​ cost). It is important for the nurse to know all medications the client is takinglong dash—both prescription and ​over-the-counterlong dash—before giving treatment for hypertension. The nurse would not perform an ultrasound before administering antihypertensive​ medication, unless ordered by the health care provider.

The nurse is caring for a client who has been given lidocaine​ (Xylocaine) for a cardiac dysrhythmia. Lab values indicate lidocaine toxicity. Which assessment findings would support this lab ​diagnosis? Select all that apply. A) Paresthesia B) Confusion C) Anxiety D) Tremors E) Blurred vision

A, B, C, D Rationale ​Confusion, anxiety,​ tremors, and paresthesia are all symptoms of lidocaine toxicity. Blurred vision is not a symptom of lidocaine toxicity.

What are some of the risk factors for developing heart ​failure? Select all that apply. A) Obesity B) Family history C) Smoking D) Diabetes mellitus E) Asthma

A, B, C, D The risk factors for​ slow-developing heart failure include coronary artery​ disease, hypertension, family​ history, cardiotoxic​ drugs, smoking,​ obesity, alcohol​ abuse, and diabetes mellitus. Asthma is a chronic inflammatory disease of the airways and not a risk factor for heart disease.

Which are considered contraindications for the use of warfarin ​(Coumadin)? Select all that apply. A) Active bleeding B) Recent trauma C) Intracranial hemorrhage D) Bacterial endocarditis E) Recent valve surgery

A, B, C, D Warfarin​ (Coumadin) is contraindicated in clients with active​ bleeding, intracranial​ hemorrhage, and recent trauma because it inhibits the synthesis of clotting​ factors, which increases bleeding time. Valve replacement surgery requires anticoagulation​ therapy, such as​ warfarin, to prevent clots from forming.

What are appropriate goals for clients with heart ​failure? Select all that apply. A) Improving cardiac function B) Slowing progression of the disease C) Treating underlying causes D) Decreasing the workload of the heart E) Applying treatments that cure the disease

A, B, C, D Your goals for clients with heart failure include treating or removing underlying​ causes, slowing disease​ progression, decreasing the heart​'s ​workload, and improving cardiac function. Because heart failure is a chronic​ condition, the goals for treatment are typically not curative.

Prostaglandins provide several beneficial effects. Which beneficial effects are reduced when aspirin​ (Bayer) is​ used? Select all that apply. Maintenance of bronchial smooth muscle Blood flow to the kidneys Protection from stomach acid Reduction of inflammation Clotting of blood

A, B, C, E Aspirin​ (Bayer) reduces these beneficial effects provided by​ prostaglandins: protecting the stomach against​ acid; making sure the kidneys receive sufficient blood​ flow; maintaining smooth muscles in the​ uterus, blood​ vessels, and bronchial​ tubes; and clotting of blood.​ Aspirin, not​ prostaglandins, decreases inflammation. Next Question

An older male client is at risk for developing a fluid deficit because he is unaware of the causes of dehydration. When teaching the client about​ dehydration, which causes would the nurse ​include? Select all that apply. A) Loss of a large amount of blood B) Abuse of diuretics C) Profuse sweating D) Hormone therapy E) Nausea leading to​ vomiting, and diarrhea

A, B, C, E Rationale Excessive vomiting with nausea and diarrhea can cause​ dehydration, especially in children and older adults. Excessive or profuse sweating can cause dehydration. Hemorrhage and profuse bleeding can cause dehydration. The use and abuse of diuretics can cause a client to become dehydrated if fluid intake is not properly managed. Proper administration of hormone therapy should not cause dehydration.

The nurse is caring for an adult client with hypomagnesemia. Which factors does the nurse recognize as causes of this ​disorder? Select all that apply. A) Diarrhea and vomiting B) Laxative abuse C) Loop diuretic therapy D) Magnesium supplements E) Kidney failure

A, B, C, E Rationale Hypomagnesemia is decreased serum magnesium. In kidney​ failure, electrolytes, including​ magnesium, are not maintained properly. Loop diuretic therapy may cause too much magnesium to be excreted. Diarrhea and vomiting cause dehydration and low levels of​ electrolytes, including magnesium. Laxative abuse can cause electrolyte imbalances and low levels of magnesium. Taking a magnesium supplement may lead to too much magnesium in the​ body, not too little.

When administering a vaccination to a​ client, what should the nurse ​consider? Select all that apply. A) Anaphylaxis and adverse reactions B) Precautions and contraindications C) Dose and timing D) Time of last food intake E) Route of administration

A, B, C, E Rationale Several vaccines must be given at scheduled time periods to maintain immunity. Some vaccines also require booster doses. The nurse must know the proper route of administration before administering a vaccine. The nurse also needs to consider precautions and​ contraindications, to ensure that the vaccine is not administered to individuals whom the drug will harm. The nurse must​ know, be able to​ identify, and teach clients the clinical manifestations of anaphylaxis and adverse​ reactions, to ensure client safety. The time that the client last ate is not a factor in the administration of vaccines.

The nurse is assessing an adult client and discussing the risk factors for primary hypertension in adults. What risk factors can contribute to the adult​ client's risk for ​hypertension? Select all that apply. A) Age B) Stress C) Race D) The use of oral contraceptives E) Family history

A, B, C, E Rationale Stress can cause an increase in blood pressure. A family history of hypertension increases the risk for hypertension in the adult client. Increasing age also increases the risk for developing hypertension. African Americans are at higher risk for hypertension. The use of oral contraceptives increases the risk for secondary​ hypertension, not primary hypertension.

The nurse is reviewing the use of acetylsalicyclic acid​ (aspirin [Bayer]) in the client population. Which clients could benefit from the use of ​aspirin? Select all that apply. A) A client with mild back pain after a fall B) An adult client with a fever C) A client with inflammatory arthritis D) A client with gastrointestinal bleeding E) A client with primary dysmenorrhea

A, B, C, E Rationale The analgesic action of aspirin reduces mild to moderate pain. The inflammatory action of aspirin reduces pain in clients with arthritis. Aspirin can be given to decrease painful menstruation​ (dysmenorrhea). Aspirin has antipyretic actions to reduce fever but should only be given to​ adults, not​ children, because aspirin increases the risk of Reyes Syndrome in children with​ flu-like symptoms and chicken pox. Aspirin use is contraindicated in clients with gastrointestinal bleeding because it reduces platelet aggregation and could cause serious bleeding.

The nurse is monitoring an adult client with hypertension who will be given a strong antihypertensive. What is the​ nurse's role in monitoring this ​client? Select all that apply. A) Heart rate should be taken before medication administration and a​ half-hour thereafter to ensure effectiveness. B) The nurse should monitor the client before and after dosing. C) Blood pressure should be monitored before and after administering medication. D) As long as the client has no​ symptoms, there is no need to monitor. E) The client should be monitored for dizziness.

A, B, C, E Rationale The nurse should monitor the client both before and after medication administration to check for side effects and to see if the medication is working effectively. The client should be monitored for a decrease in heart rate by taking the pulse before medication is administered and a​ half-hour thereafter. The blood pressure should be measured and recorded before and after medication administration. The client should be monitored for dizziness after taking the medication to prevent the client from falling. It is always important to monitor a client after giving a new medication.

Several clients on the​ nurse's unit have thrombocytopenia. Clients with which conditions have thrombocytopenia due to increased platelet ​destruction? Select all that apply. A) Thrombocytic purpura B) Systemic lupus erythematosus C) Bone marrow suppression D) Disseminated intravascular coagulation E) Vitamin B12 deficiency

A, B, D Rationale Clients with thrombocytic​ purpura, disseminated intravascular​ coagulation, and lupus have thrombocytopenia due to increased platelet destruction. Clients with bone marrow suppression or a vitamin B12 deficiency have thrombocytopenia due to reduced platelet production.

The nurse is assessing a client who has been on one type of medication for hypertension and has had a low potassium level for several weeks. Which statements about​ potassium-sparing diuretics are ​correct? Select all that apply. A) Hyperkalemia is a risk when taking spironolactone​ (Aldactone) and an ACE inhibitor. B) Spironolactone​ (Aldactone) will prevent the client from becoming hypokalemic by sparing normal potassium. C) All diuretics are potassium sparing. D) Potassium levels do not diminish with​ potassium-sparing diuretics. E) Hyperkalemia is not a risk of​ potassium-sparing diuretic therapy when an ACE inhibitor is taken concurrently.

A, B, D Rationale ​Potassium-sparing diuretics spare the potassium during​ diuresis, helping the client to maintain a normal potassium level. Spironolactone​ (Aldactone) is a​ potassium-sparing diuretic. Hyperkalemia is a risk of​ potassium-sparing diuretic therapy if that therapy is used concurrently with ACE inhibitors. Not all diuretics work the same​ way; thiazides are an example of diuretics that are not potassium sparing.

As a​ nurse, what should you do before administering adenosine​ (Adenocard, Adenoscan) to a ​client? Select all that apply. A) Set up IV. B) Place client in supine position. C) Place head of bed at a 45degrees° angle. D) Place client on ECG monitors. E) Warn client that​ he/she may feel faint.

A, B, D, E Before the administration of adenosine​ (Adenocard, Adenoscan), you will need to set up the IV. The client should be placed on ECG monitors. Warn the client that​ he/she may feel faint because of the brief period of​ asystole, usually 1dash-5 ​seconds, that adenosine can cause. Because of this​ asystole, the client should be placed in a supine​ position, not at a 45degrees° angle.

The nurse is teaching the client about clinical manifestations of angina pectoris. Which signs and symptoms should be included in the teaching ​plan? Select all that apply. A) Pain in the left​ shoulder, arm,​ jaw, neck, and epigastric and upper abdominal​ area, and between the shoulders B) Shortness of breath C) Feeling changes in heart rhythm D) Chest pain that is​ stabbing, crushing, or squeezing​ (tightness) E) Feeling fear or anxiety

A, B, D, E, Rationale Chest pain associated with angina pectoris is usually characterized as​ stabbing, crushing, or squeezing​ (tightness); some have described angina pain as​ "feeling like a vise is​ squeezing" the chest. Clients with angina pectoris often describe pain in the left shoulder or​ arm, jaw,​ neck, epigastric and upper abdominal​ area, and between the shoulders. When the angina​ occurs, clients report feelings of​ anxiety, fear, impending​ doom, and shortness of breath. Feeling changes in the heart rhythm is not associated with angina pectoris.

The health care provider prescribed nitrates for a client who is admitted with stable angina pectoris. Nitrates would be contraindicated if this client had which additional health ​conditions? Select all that apply. A) Head trauma B) Hypertension C) Hypotension D) Dehydration E) Diabetes mellitus

A, C, D, Rationale Nitrates have been a preferred initial treatment for angina pectoris for many years. Nitrates are vasodilators that act upon vascular smooth muscle. They promote oxygenation to cardiac muscle by decreasing myocardial oxygen demand. This is achieved by decreasing the amount of blood returning to the heart​ (preload) and by increasing the flow of blood through the coronary arteries. The coronary vasodilatation is also effective in the treatment of angina caused by vasospasm. Contraindications and precautions for using nitrates include preexisting​ hypotension; head injury or head​ trauma; and​ dehydration, as well as​ shock, pericardial​ tamponade, and constrictive pericarditis. Caution should be used when administering the​ sustained-release form of nitrates to clients with glaucoma or to clients who use sildenafil​ (Viagra) or similar​ medications, because concurrent use may cause severe hypotension. Nitrates are not contraindicated for the client with diabetes or hypertension.

The nurse is administering blood products to a client who was admitted to the emergency department following a motor vehicle crash. Which assessment findings indicate adverse reactions to the blood ​product? Select all that apply. A) The client reports that his​ face, neck, and upper chest are red and feel warm. B) The client reports that he has to urinate. C) The client reports that he is feeling lightheaded and​ "itchy." D) The nurse notes that the client is developing slurred speech. E) The client reports that he feels anxious and is beginning to have difficulty breathing.

A, C, D, E Rationale Adverse reactions to blood products include feeling​ lightheaded; itchiness or​ hives; flushing of the​ skin, especially of the​ face, neck, and upper​ chest; anaphylaxis, which may include feelings of anxiety and breathing​ problems; slurred​ speech; and confusion. A need to urinate is not an indication that the client is experiencing an adverse reaction to a blood product.

The client tells the nurse that one health care provider told him that he has too much cholesterol in his blood. Another health care provider told him he has elevated lipids. The client asks the nurse to explain the meaning of these statements. Which will the nurse include in the ​explanation? Select all that apply. A) Cholesterol levels in the bloodstream are an important measure of heart health. Some types of cholesterol are considered​ "good" and some are considered​ "bad." B) It is rare that clients are able to manage hyperlipidemia with only lifestyle changes and no medications. C) There are two causes of​ hyperlipidemia: heredity and​ lifestyle, which includes diet and activity level. D) Cholesterol is fat that the body produces and needs to work properly. The terms cholesterol and lipids are often used interchangeably. E) Eating​ wheat, corn,​ oats, and olive oil will help reduce your cholesterol level.

A, C, D, E Rationale Cholesterol is needed for cell wall functioning and for the production of​ hormones, such as estrogen and testosterone. The body makes the needed cholesterol in every​ cell, so there is no dietary need for it. People have high levels of lipids​ (cholesterol) for two​ reasons: heredity, which is what we get from our parents and​ ancestors; and​ lifestyle, which includes diet and activity level. HDL contains the highest amount of protein and is referred to as​ "good cholesterol." HDL transports cholesterol from the body to the liver for destruction. LDL contains the highest amount of cholesterol and is referred to as​ "bad cholesterol." LDL transports cholesterol from the liver to the tissues and​ organs, to build plasma membranes. Increased levels of LDL are shown to be a risk factor for coronary artery disease. To reduce the risk of heart​ disease, one must raise HDL cholesterol levels and reduce LDL cholesterol levels. Many clients can manage hyperlipidemia with lifestyle changes and no​ medications; however, this is not true for everyone. Eating plant lipids causes the body to excrete cholesterol and lower LDL levels. Sources of plant lipids include​ wheat, corn,​ rye, oats,​ rice, nuts, and olive oil.

The nurse is educating an adult client on the treatment of hypertension with diuretics. Which statements accurately describe diuretic pharmacologic therapy for ​hypertension? Select all that apply. A) Diuretics increase urine output. B) Thiazides are not diuretics. C) Diuretics decrease blood volume. D) Diuretics are not given to clients who are anuric. E) Diuretics are used in​ first-line treatment of hypertension.

A, C, D, E Rationale Diuretics are used as the​ first-line treatment of hypertension because of the success rate with these medications. Diuretics decrease the absorption of sodium and increase the release of​ water, so urine output increases. When diuretics increase urine​ output, fluid vascular volume decreases. Clients who do not urinate normally are not given diuretics as a treatment for hypertension. Thiazides are a​ first-line group of diuretics.

How do hormones regulate blood pressure in the ​body? Select all that apply. A) Change peripheral resistance B) Send messages to the arteries C) Receive information from receptors in the carotid arteries D) Cause vasoconstriction E) Increase blood volume

A, D, E The body uses hormones to help regulate blood pressure. The antidiuretic hormone​ (ADH) causes vasoconstriction and increases blood volume. The​ renin-angiotensin-aldosterone system​ (RAAS) affects blood pressure by changes in both blood volume and peripheral resistance. The vasomotor system in the​ brain, not​ hormones, receives information from receptors in the aorta and the carotid arteries. Based on this​ information, the vasomotor system sends messages to the arteries to either constrict the arterial walls to increase blood​ pressure, or to vasodilate the arterial walls to decrease blood pressure.

The nurse has taught the​ client, who has stable​ angina, about home care and the most common side effects of​ nitrates, which have been prescribed to treat the client​'s angina. The nurse determines that the teaching has been effective when the client makes which ​statements? Select all that apply. ​A) "I may feel like my heart is racing while my body is adjusting to the​ medication." B) ​"My blood sugar levels will increase. I will have to check my blood sugar levels after each​ dose." C) ​"I may feel dizzy or lightheaded after I take my angina​ medication." D) ​"I may develop a throbbing headache when I take the medication for my​ angina." E) ​"My skin may become irritated in the area where I have put the​ patch."

A, C, D, E Rationale Nitrates are very effective antianginal and​ anti-ischemic agents.​ However, side effects with nitrate therapy are common. A throbbing headache is the most common side effect of nitrates.​ Nitrate-induced hypotension is also common and is most often asymptomatic. To prevent lightheadedness or​ dizziness, and to prevent a fall or​ injury, the nurse must teach clients to get up slowly when they are rising from a seated or lying position. Nitrate skin patches may cause skin​ irritation, which can be avoided by putting the patch in a different place each time. These side effects are not serious and usually become less severe with continued use of the nitrate medication. Tachycardia may occur as the body is adjusting to the medication. Elevated blood sugar levels are not an adverse effect of nitrates.

What are the goals of pharmacotherapy for clients who are taking medications for high ​cholesterol? Select all that apply. A) Decreased levels of lipids B) Weight loss C) Knowledge of the​ drug's adverse effects D) Lack of adverse effects E) Understanding​ self-administration of the drug

A, C, D, E The primary purpose of using​ lipid-lowering agents is to decrease lipid levels and cholesterol. Health care providers do not want any client to experience adverse effects related to pharmacotherapy. It is also important for the client to demonstrate an understanding of the​ medication, including its proper​ administration, adverse​ effects, and contraindications. Weight loss is not a purpose of therapy with​ cholesterol-lowering medications.

The nurse is teaching a client with an ankle sprain about the drug ibuprofen​ (Advil). What should be included in the ​teaching? Select all that apply. A) Avoid the use of alcohol while taking ibuprofen. B) Expect to have visual changes. C) Use sunscreen and protective clothing when outdoors. D) Do not take aspirin and ibuprofen together. E) Report any unusual bruising to the health care provider.

A, C, D, E, Rationale Clients taking ibuprofen should limit the amount of time in the sun and cover any exposed skin. The combination of aspirin and ibuprofen can increase the risk of adverse effects. The client should notify the health care provider of any​ bruising, which might indicate an adverse reaction to the drug. The use of alcohol in combination with ibuprofen can increase the risk of stomach irritation. Visual​ changes, such as blurriness or a decrease in​ vision, are not expected or normal with the use of​ ibuprofen, and should be reported to the health care provider.

The nurse has provided teaching to a client who recently experienced a myocardial infarction​ (MI) about the use of aspirin​ (Bayer). Which client response indicates that the teaching has been ​successful? Select all that apply. A) ​"I may bleed longer than usual if I accidently cut myself or if I have a​ nosebleed." B) ​"I don​'t need to worry about having any side effects with aspirin because side effects with aspirin are pretty​ rare." C) ​"Taking aspirin will help to prevent another heart​ attack." D) ​"The doctor ordered aspirin to relieve my chest​ pain." E) ​"Being on this​ medication, I need to watch out for any stomach problems like heartburn or upset​ stomach."

A, C, E Rationale Aspirin​ (Bayer) can reduce mortality in the weeks following acute myocardial infarction​ (MI). Aspirin is used not only to treat​ MI, but also to prevent it. Aspirin has antiplatelet aggregation effects that prevent thrombus​ formation, and also an​ anti-inflammatory action that decreases the formation of​ C-reactive protein​ (the formation of​ C-reactive protein is associated with an increased risk for​ MI). All medications have adverse effects​ (side effects). The most common side effects of aspirin include stomach problems​ (irritation, heartburn, stomach​ pain, and upset​ stomach) and bleeding problems​ (gastrointestinal bleeding, prolonged​ bleeding). Other side effects include allergic​ reaction, tinnitus and hearing loss​ (with high​ doses), and​ hepatotoxicity/nephrotoxicity (with​ long-term use).

The nurse is preparing to administer diltiazem​ (Cardizem) to a client for treatment of a cardiac dysrhythmia. Which condition noted in the client​'s history would cause the nurse to question administration of this ​medication? Select all that apply. A) History of sick sinus syndrome B) History of atrial fibrillation C) History of severe bradycardia D) History of ventricular fibrillation E) History of AV block

A, C, E Rationale Diltiazem​ (Cardizem) is contraindicated for clients with a history of severe​ bradycardia, AV​ block, or sick sinus syndrome. This medication would not be questioned for a client with atrial or ventricular fibrillation.

What are examples of the type of dysrhythmia described as ectopic ​beats? Select all that apply. A) Premature ventricular contraction​ (PVC) B) Torsades de pointes C) Sick sinus syndrome D) Premature atrial contraction​ (PAC) E) Wolff Parkinson White syndrome

A, D Ectopic beats are an interruption in the usual conduction​ sequence; for​ example, premature ventricular contraction​ (PVC) and premature atrial contraction​ (PAC). Sick sinus syndrome is an example of a bradydysrhythmia. Torsades de pointes and Wolff Parkinson White syndrome are examples of tachydysrhythmias. Next Question

The pediatric nurse is assessing a child in the clinic who has tested positive for an influenza virus. Assessment vital signs reveal a temperature of 102.4​°F ​(39.4​°​C). Which medication does the nurse anticipate will be ordered for the​ client? Select all that apply. Select all that apply. A) Ibuprofen​ (Motrin) B) Acetylcyclic acid​ (aspirin) C) ​Brompheniramine/phenylephrine (Dimetapp) D) Acetaminophen​ (Tylenol) E) Diphenhydramine​ (Benadryl)

A, D Rationale Acetaminophen is effective in reducing temperature in children and is not contraindicated in children. Ibuprofen is classified as a nonsteroidal​ anti-inflammatory drug​ (NSAID). Ibuprofen has antipyretic effects and is commonly used in children.​ Infants, children, and adolescents with flulike symptoms should not be given​ aspirin, due to the increased risk of Reye syndrome with this drug. Diphenhydramine is an antihistamine that does not lower body temperature.​ Brompheniramine/phenylephrine is a combination decongestant with no antipyretic properties.

Which statements about cholesterol are ​true? Select all that apply. A) There is no need for dietary​ cholesterol, because the liver is able to synthesize it. B) The body needs large amounts of cholesterol. C) Cholesterol is a building block for amino​ acids, vitamin​ D, and estrogen. D) Cholesterol has a definite role in the development of atherosclerosis. E) Cholesterol is a vital component of plasma membranes.

A, D, E Cholesterol is a vital component of plasma membranes. It helps maintain the integrity of the membranes and facilitates the way cells communicate with each other. There is no need for dietary cholesterol because the liver is able to synthesize it. Too much cholesterol is linked to atherosclerosis​ (hardening of the​ arteries); the body needs only small amounts of​ cholesterol, not large amounts. Cholesterol is a building block for vitamin​ D, bile​ acids, cortisol,​ estrogen, and​ testosterone, but not amino acids. Amino acids are building blocks for protein.

Which conditions would contraindicate the use of direct​ vasodilators? Select all that apply. A) History of myocardial infarction B) Low sodium levels C) Altered skin integrity D) Underlying tachycardia E) Angina

A, D, E Direct vasodilators are commonly used in hypertensive emergencies.​ However, they are not a​ first-line treatment for hypertension due to their many adverse effects. Direct vasodilators are contraindicated for clients who have​ angina, a history of myocardial​ infarction, or an underlying​ tachycardia, as the medication could worsen these conditions or precipitate another cardiac event. Direct vasodilators are not contraindicated for clients with low sodium levels or altered skin integrity.

The nurse is assessing an adult client for hypertension. Which factors influence blood ​pressure? Select all that apply. A) Peripheral resistance B) Lipid levels in the bloodstream C) Glucose levels D) Cardiac output E) Blood volume

A, D, E Rationale The cardiac output​ (the amount the heart​ pumps) is responsible for the blood pressure. Low cardiac output will lower​ pressure, increased cardiac output will increase blood pressure. The pressure inside the arteries will directly affect the blood pressure. The amount of blood volume directly influences blood pressure. Glucose levels do not directly affect blood​ pressure, but the​ long-term effects of diabetes can contribute to hypertension. Lipid levels in the blood stream do not affect blood​ pressure, although high levels contribute to the risk of stroke from hypertension.

Which are characteristics of the drug class of cholesterol absorption ​inhibitors? Select all that apply. A) Acts primarily in the small intestine B) Includes many medications for prescription C) Administered IM only D) Safe for pregnant women E) Contraindicated in clients with liver disease

A, E Cholesterol absorption inhibitors block up to​ 50% of the absorption of cholesterol in the small intestine. This class of drugs is contraindicated in clients with liver disease and those who take bile acid​ resins; pregnancy is another contraindication. There is only one medication in this class for health care providers to​ prescribe: ezetimibe​ (Zetia).The route of administration for Zetia is​ oral, not IM.

Tommy Powell and his mother have come to your clinic. Tommy is a​ 6-year-old who presents with symptoms of chickenpox. What is the drug of choice for controlling​ Tommy's fever and​ pain? Aspirin​ (Bayer) Celecoxib​ (Celebrex Acetaminophen​ (Tylenol) Morphine sulfate​ (Duramorph)

Acetaminophen​ (Tylenol)

A client admitted with gastric discomfort and peptic ulcer disease has a temperature of 102.8​°F ​(39.65​°​C). Which ordered medication does the nurse select to​ administer? Acetylsalicylic acid​ (aspirin) Ketoprofen​ (Orudis) Acetaminophen​ (Tylenol) Ibuprofen​ (Advil)

Acetaminophen​ (Tylenol) TYLENOL DOES NOT CAUSE GASTRIC IRRITATION OR BLEEDING

When vaccinations are administered to prevent​ disease, what is the result​ called? Active immunity Immunoactivation Immunosuppression Passive immunity

Active immunity

Which are characteristics of the drug class of cholesterol absorption ​inhibitors? Select all that apply. Includes many medications for prescription Acts primarily in the small intestine Contraindicated in clients with liver disease Safe for pregnant women Administered IM only

Acts primarily in the small intestine Contraindicated in clients with liver disease

Which are appropriate nursing actions for the client receiving enteral feeding who has the complication of ​diarrhea? Select all that apply. Add fiber to the nutritional supplement. Dilute the feeding. Decrease the use of​ opioids, to slow down the digestive process. Change the formula to a​ low-fat preparation. Slow down the infusion.

Add fiber to the nutritional supplement. Dilute the feeding Slow down the infusion.

Crystalloids

Administered to clients who require fluid replacement therapy. Crystalloids contain electrolytes and are similar to extracellular fluid. These solutions PROMOTE urine output. The three classes of crystalloid solutions are isotonic, hypertonic, and hypotonic.

Which characteristics pertain to an Enterobius vermicularis ​infection? Select all that apply. Afflicts mostly children Worms as long as 7dash-25 meters Symptoms include perineal itching Causes insomnia Most frequently diagnosed helminthic infection in the United States

Afflicts mostly children Symptoms include perineal itching Causes insomnia Most frequently diagnosed helminthic infection in the United States

Which pharmacologic agents are used to treat visceral ​leishmaniasis? Select all that apply. Amphotericin B​ (Amphocin) Primaquine phosphate​ (Primaquine) Pentamidine isethionate​ (Pentam 300) Chloroquine phosphate​ (Aralen) Sodium stibogluconate​ (Pentostam)

Amphotericin B​ (Amphocin) Pentamidine isethionate​ (Pentam 300) Sodium stibogluconate​ (Pentostam)

Which diagnostic tests are used to identify and confirm​ tuberculosis? Select all that apply. Amplified​ DNA/RNA tests Computed tomography Sputum cultures CBC with differential Tuberculin skin test

Amplified​ DNA/RNA tests Sputum cultures Tuberculin skin test

Mr. Parker is prescribed a​ lipid-lowering agent. As you write up the medication reconciliation​ form, you notice that he is also taking several other medications. Which action is most important before Mr. Parker is​ discharged? Schedule a​ follow-up lipid profile Explain lifestyle modifications to Mr. Parker Assess for possible​ drug-drug interactions Obtain Mr.​ Parker's pharmacy information

Assess for possible​ drug-drug interactions

Hwong​ Li, who had a kidney​ transplant, is prescribed azathioprine​ (Imuran). He asks how this medication works and why he needs to take it. You explain that azathioprine is​ what? A corticosteroid used to suppress the immune system and minimize transplanted tissue rejection An antimetabolite used to prevent the​ body's rejection of transplanted tissue by suppressing​ T-cell effects An antibiotic used to prevent infection in transplanted tissue A chemotherapeutic agent used to improve blood supply to a newly transplanted organ

An antimetabolite used to prevent the​ body's rejection of transplanted tissue by suppressing​ T-cell effects

Which test is used to assess ejection​ fraction? A) Creatinine levels B) Serum electrolyte levels C) Echocardiography D) Electrocardiography

An echocardiogram is used to assess heart function and ejection fraction. Electrocardiography is performed to assess for​ dysrhythmias, myocardial​ ischemia, or myocardial infarction. Creatinine levels are drawn to assess renal​ function, whereas serum electrolyte levels are read to assess fluid and electrolyte balance.

Sharon Carter presents to your clinic complaining of an injury to her right hand. Mrs. Carter states that she cut her hand on an unknown sharp object when planting flowers three days ago. The partially healed laceration has significant​ redness, swelling, and warmth on the skin surface around the edges of the wound. A moderate amount of​ yellow-green drainage is present at the​ wound's open edges. Mrs. Carter asks why the wound does not seem to be healing. What is your best​ answer? Antibiotics should not be prescribed until culture and sensitivity test results are available. The drainage from the wound is a positive sign and represents one of the stages in healing. The wound is not likely infected and only requires cleaning and bandaging. An infection is caused by a microorganism that may enter the body through a break in the skin.

An infection is caused by a microorganism that may enter the body through a break in the skin.

Classes of coagulation modifiers include:

Anticoagulants are the most frequently prescribed coagulation modifiers. Antiplatelet drugs prevent arterial clot formation. Thrombolytics dissolve existing clots. Hemostatics enhance clot formation.

What​ "triggers" an immune system​ response? ​Redness/swelling at an injury site Fever Antibody entering the body Antigen entering the body

Antigen entering the body

Ms. Small was admitted for an emergency appendectomy 2 days ago. She no longer has a​ dressing, and the wound is exposed to air. The nurse is assessing her abdomen. Which of the following will the nurse include in the wound ​assessment? Select all that apply. Are the wound edges well​ approximated? Appearance of the​ wound: Is it​ healing, size,​ drainage, swelling,​ redness? Inspect for presence of foul odor​ (from wound) and assess for pain. Any other injuries such as​ fractures, internal​ bleeding, abscess? Is there a​ dressing? If​ yes, check for drainage​ amount, color,​ odor, and use of drains.

Are the wound edges well​ approximated? Appearance of the​ wound: Is it​ healing, size,​ drainage, swelling,​ redness? Inspect for presence of foul odor​ (from wound) and assess for pain.

Ms. Small was admitted for an emergency appendectomy 2 days ago. She no longer has a​ dressing, and the wound is exposed to air. The nurse is assessing her abdomen. Which of the following will the nurse include in the wound ​assessment? Select all that apply. Are the wound edges well​ approximated? Appearance of the​ wound: Is it​ healing, size,​ drainage, swelling,​ redness? Inspect for presence of foul odor​ (from wound) and assess for pain. Any other injuries such as​ fractures, internal​ bleeding, abscess? Is there a​ dressing? If​ yes, check for drainage​ amount, color,​ odor, and use of drains.

Are the wound edges well​ approximated? Appearance of the​ wound: Is it​ healing, size,​ drainage, swelling,​ redness? Inspect for presence of foul odor​ (from wound) and assess for pain.

Which location most accurately reflects where helminthic infections​ thrive? ​Third-world countries Areas with poor sanitation Areas with​ stagnant, standing water Areas where raw meat is ingested

Areas with poor sanitation

Maxine Carlson is a​ 73-year-old woman who was admitted to the hospital after suffering a cerebrovascular accident. Her deficits include​ right-side paralysis, inability to​ swallow, and global aphasia. Mrs. Carlson has a nasogastric tube inserted and begins bolus enteral​ feedings, 300 mL every 6 hours. What is a significant risk associated with bolus​ feedings? Aspiration Constipation Hypoproteinemia Hyperglycemia

Aspiration

The nurse is caring for an older adult client who is diagnosed with inflammatory bowel​ disease, and receiving enteral nutrition therapy to rest the bowel. The nurse should watch for which complications of enteral ​nutrition?

Aspiration Dehydration Nausea

Jeff Mitchell is a​ 78-year-old male client who is being treated for a persistent sore throat with fever and cough. The health care provider prescribes penicillin G. Mr. Mitchell is instructed to avoid fruit juices for at least 1 hour before and after taking the medication. What is the most critical nursing aspect of preparing Mr. Mitchell for​ discharge? Assess Mr. Mitchell for signs of​ thrombocytopenia, such as nosebleed and bruising. Assess Mr. Mitchell for drug allergies and a history of hypersensitivity reactions. Evaluate Mr.​ Mitchell's nutritional habits that could affect the effectiveness of the medication. Obtain a baseline set of vital​ signs, including a WBC.

Assess Mr. Mitchell for drug allergies and a history of hypersensitivity reactions.

The nurse has a client with a new onset of a skin disorder. Which assessments would the nurse perform to aid in determining the cause of the ​condition? Select all that apply. Review hemoglobin and hematocrit levels. Assess legs for color of​ skin, warmth, and hair distribution. Assess the color of the​ sclera, skin, and mucous membranes. Review the BUN and creatinine levels. Palpate the abdomen.

Assess legs for color of​ skin, warmth, and hair distribution. Assess the color of the​ sclera, skin, and mucous membranes. Review the BUN and creatinine levels. Palpate the abdomen.

The nurse is assessing an adult client before antibiotics are prescribed. What would be appropriate tasks for the nurse before initiating antibiotic ​therapy? Select all that apply. Only assess the area of complaint Assess the client for allergies Ensure that ordered cultures have been obtained and sent to the lab Ensure that the blood work includes a WBC count Obtain a baseline set of vital signs and appropriate blood work

Assess the client for allergies Ensure that ordered cultures have been obtained and sent to the lab Ensure that the blood work includes a WBC count Obtain a baseline set of vital signs and appropriate blood work

The nurse has properly administered statin therapy to a male client with hyperlipidemia and has provided education about common adverse effects of the medication. Which actions will the nurse carry out after administering the ​medication? Select all that apply. Assess the client to determine if he has experienced any related adverse effects. Teach the client about lifestyle changes that will enhance effectiveness of overall treatment. Teach the client that he may discontinue taking the medication once serum lipid levels have decreased and are within the normal range. Explain to the client that statins should be taken in the evening for greatest effectiveness. Reinforce the reason for taking the medication and the importance of reporting side effects experienced to the nurse​ and/or the health care provider.

Assess the client to determine if he has experienced any related adverse effects. Teach the client about lifestyle changes that will enhance effectiveness of overall treatment. Explain to the client that statins should be taken in the evening for greatest effectiveness. Reinforce the reason for taking the medication and the importance of reporting side effects experienced to the nurse​ and/or the health care provider.

What instructions should you give to a client who is taking​ enteric-coated ​aspirin? Select all that apply. Take with 240 mL of water or milk. Avoid alcoholic beverages. Stop medication 2 days before dental work. Report pregnancy. Do not crush or chew the medication.

Avoid alcoholic beverages. Report pregnancy. Do not crush or chew the medication.

Ms. Kane is prescribed clotrimazole​ (Mycelex) lozenges for the oral candidiasis infection. When teaching her about the appropriate use of this​ medication, what instruction should you​ include? Avoid drinking and eating for at least 30 minutes after oral administration. Abstain from sexual intercourse during the treatment regimen. You may chew the lozenge or swallow it whole with water if desired. Avoid exposure to sunlight while undergoing treatment.

Avoid drinking and eating for at least 30 minutes after oral administration.

Jill Herman is a​ 40-year-old client who presents with a persistent redness of her​ face, diagnosed as rosacea. Which instruction should you include in the client education for Ms.​ Herman? Spend 20 minutes 3 times daily exposed to sunlight. Rosacea is a type of contact dermatitis easily spread by close contact. Apply antibiotic ointment to the rash as directed. Avoid drinking hot liquids and alcoholic beverages.

Avoid drinking hot liquids and alcoholic beverages.

Which topical and oral medications are used to treat​ rosacea? Tazarotene​ (Avage) and etanercept​ (Enbrel) Dibucaine​ (Nupercainal) and cyclosporine​ (Neoral) Azelaic acid​ (Azelex) and isotretinoin​ (Claravis) Sulfacetamide sodium​ (Cetamide) and isotretinoin​ (Claravis)

Azelaic acid​ (Azelex) and isotretinoin​ (Claravis)

A nurse is evaluating skin care needs. Which client poses the greatest risk for skin​ breakdown? A​ 36-year-old with coronary artery disease An​ 18-month-old with bronchitis A​ 20-year-old with a urinary tract infection A​ 74-year-old with a diagnosis of diabetes

A​ 74-year-old with a diagnosis of diabetes

What is the primary cause of myocardial infarction​ (MI)? A) Excessive weight B) Advanced coronary artery disease C) Coronary artery spasm D) Prolonged aerobic exercise

B Advanced coronary artery disease ​(CAD)long dash—a narrowing of the lumen of one or more coronary arterieslong dash—is the primary cause of myocardial infarction​ (MI). CAD results from atherosclerosis or fatty plaque buildup inside the artery walls that leads to blocked blood flow. Prolonged aerobic​ exercise, coronary artery​ spasm, and too much weight all put increased demands on the heart​ muscle, but are not​ primary, direct causes of MI.

Why does rest often reduce angina​ pain? A) Opens narrowed arteries in the heart B) Decreases myocardial oxygen demand C) Reverses tissue ischemia in the heart D) Increases myocardial oxygen demand

B Angina pain is usually relieved by rest because relaxing reduces myocardial oxygen demand that was increased with physical exertion or stress.​ However, rest cannot reverse tissue ischemia or open narrowed arteries.

One of your​ clinic's clients, Raymond​ Chu, has hypertension without angina. Mr. Chu is prescribed amlodipine​ (Norvasc), a calcium channel blocker. Which should you teach Mr. Chu before he leaves the​ clinic? A) Note if there is an increase in angina. B) Report constipation. C) Take extra calcium supplements. D) Listen for bowel sounds.

B Calcium channel blockers​ (CCBs), such as amlodipine​ (Norvasc), slow the movement of calcium into the​ heart's cells. One adverse effect that can occur with CCBs is constipation. CCBs prevent calcium from entering the cardiac​ cells, which slows the heart​ rate; taking extra calcium supplements is not indicated. Although CCBs can cause an increase in​ angina, your client does not have this​ condition, so he would not be noting an increase. Bowel sounds are not related to hypertension issues.

The nurse is caring for a client who is receiving enteral nutrition through a feeding​ tube, and is experiencing diarrhea. What actions should the nurse ​take? Select all that apply. Increase the infusion rate. Add fiber to the nutritional supplement. Administer loperamide​ (Imodium), an​ antidiarrheal, as ordered. Increase the concentration of the feeding. Test for Clostridium difficile.

B C E

Jennifer Sanchez is a​ 24-year-old female who presents to your clinic after four days of worsening​ nausea, vomiting, and diarrhea. She states that she has tried to drink fluids but cannot keep anything down. Her heart rate is 126​ beats/min, blood pressure​ 118/70 lying down and​ 86/50 sitting, and temperature​ 99.8°F (37.7°C). What is Ms. Sanchez most likely​ experiencing? A) Hypernatremia B) Fluid volume deficit C) Anxiety D) Excessive thirst

B Ms. Sanchez is most likely suffering from fluid volume deficit caused by a loss of gastrointestinal​ (GI) fluids from vomiting and diarrhea. Clinical manifestations related to fluid volume deficit include tachycardia and orthostatic hypotension. Although tachycardia may be associated with​ anxiety, orthostatic hypotension is not. Although thirst may accompany fluid volume​ deficit, there is no indication that Ms. Sanchez is experiencing excessive thirst. Hypernatremia may occur in a dehydrated​ client, but this is not indicated by the information provided.

Professor Ethel Lipkin is a​ 56-year-old female with a history of​ osteoarthritis, glaucoma, and two episodes of angina pectoris pain. She presents with another angina​ episode, and is prescribed a​ sustained-release form of nitrates to relieve her acute pain. Before you administer the medication to Ms.​ Lipkin, which precaution should be your​ priority? A) Assess the client for headache B) Ask the health care provider about the prescription order C) Measure swelling of the​ client's feet and ankles D) Assess the client for dizziness

B Nitrates are the drugs of choice for relieving acute angina pain and treating an episode.​ Nevertheless, the nurse needs to be cautious about administering the​ sustained-release form of nitrates to a client for treatment of angina if the client also has glaucoma. It is important to remember that nitrates dilate the blood vessels which can increase the intraocular pressure.​ Therefore, the priority precaution should be talking to the health care provider about the prescription order for Ms. Lipkin. Although dizziness and headache are potential adverse effects of nitrates that should be​ monitored, the medication has not yet been​ administered, and the priority is talking with the health care provider first. Swelling of the feet and ankles is not typically associated with nitrates.

Cathy​ Hoyt, age​ 52, is leaving the hospital after receiving treatment for​ left-sided heart failure. You know from her history that she also has type 2 diabetes. Which statement is most appropriate to include in your discharge​ teaching? ​A) "Your two conditions will not affect each​ other." B) ​"Managing the diabetes may help improve your heart​ function." ​C) "Lifestyle changes related to the diabetes should be your top​ priority." D) ​"Your heart condition will probably worsen even though you manage the diabetes​ well."

B Proper management of a precipitating disease​ process, such as diabetes or​ hyperthyroidism, may help to improve heart function. Diabetes and heart failure are often​ interrelated, but managing a precipitating disease should not worsen heart function. Treating heart​ failure, however, is a higher priority than lifestyle changes to control diabetes.

What is the treatment for heparin​ overdose? A) Enoxaparin​ (Lovenox) B) Protamine sulfate C) Lepirudin​ (Refludan) D) Vitamin K

B Protamine sulfate is the treatment for heparin overdose because it binds to the heparin. Enoxaparin​ (Lovenox) is a​ low-weight-molecular heparin. Vitamin K is used for warfarin overdose. Lepirudin​ (Refludan) is a medication used to treat thrombocytopenia caused by heparin therapy.

The nurse is preparing to administer furosemide​ (Lasix) and captopril​ (Capoten) to an older adult client with heart failure. When the nurse assists the client out of bed to the​ chair, the client becomes lightheaded and dizzy. After assisting the client to lie in​ bed, the nurse obtains vital signs. The client​'s blood pressure is​ 85/52. The nurse should take which​ action? A) Assist the client to sit up in bed. B) Withhold the medications and notify the health care provider. C) Administer the furosemide​ (Lasix) and withhold the captopril​ (Capoten). D) Administer the medications.

B Rationale Adverse effects of furosemide​ (Lasix) and captopril​ (Capoten) include severe​ hypotension, and the client is already hypotensive. The nurse should withhold both medications and notify the health care provider. The client may continue to be lightheaded and dizzy when sitting​ up, so the client should lie down until the blood pressure stabilizes.

The nurse is teaching a client who was recently diagnosed with chronic stable angina about the most common side effects​ (adverse reactions) of​ beta-adrenergic antagonists​ (beta blockers). Which instructions will the nurse integrate into the teaching​ plan? A) Beta blockers decrease myocardial workload by decreasing heart rate and blood​ pressure, which reduces the contractility of the heart muscle. B) Clients taking beta blockers may experience​ dizziness, lightheadedness, blurred​ vision, and nausea and vomiting as the body adjusts to the medication. C) Beta blockers are contraindicated in clients with a history of​ bradycardia, heart disease​ (e.g., heart​ block, heart​ failure, cardiogenic​ shock), stroke, lung disease​ (e.g., asthma,​ bronchitis, emphysema), or depression. D) ​Beta-adrenergic antagonists reduce blood pressure and block the effects of​ adrenaline, causing the heart to beat more slowly and with less​ force; with decreased blood​ pressure, blood vessels widen and cardiac blood flow is improved.

B Rationale As with any​ medication, adverse effects are possible with​ beta-adrenergic antagonists​ (beta blockers).​ However, not all users of beta blockers will experience problems. In​ fact, most people tolerate beta blockers well. When side effects do​ occur, they are usually minor and require no​ treatment, or are easily treated. The most common side effects of taking​ beta-adrenergic antagonists include​ bradycardia, hypotension,​ fatigue, weakness, cold hands and​ feet, dizziness, and nausea and vomiting. This question asks about the adverse effects​ (side effects) of​ beta-adrenergic antagonists, not the mechanism of action or the contraindications for these types of medications.

The nurse is preparing to administer bivalirudin to a client with thrombocytopenia. The nurse anticipates that this medication will be administered by which​ route? A) Subcutaneous B) Intravenous C) Intramuscular D) By mouth

B Rationale Bivalirudin is administered by intravenous injection. Bivalrudin is not administered by subcutaneous​ injection, by intramuscular​ injection, or by mouth.

The nurse is preparing to administer total parenteral nutrition​ (TPN) to an older adult client. Because TPN is a hypertonic​ solution, where must the catheter tip of the central venous line be​ positioned? Select the correct answer choice below. A B C D

B (superior vena cava)

The nurse is administering pharmacotherapy to a client with severe hyperkalemia. Which agent helps the body to eliminate potassium through​ urination? A) Polystyrene sulfonate​ (Kayexalate) B) Furosemide​ (Lasix) C) Insulin with dextrose D) Calcium gluconate

B Rationale Furosemide​ (Lasix) is a diuretic that reduces potassium levels through urination. Insulin with dextrose administered intravenously will cause potassium to enter the cells and reduce serum potassium​ levels, but it does not eliminate potassium through urination. Calcium gluconate is given to prevent cardiac complications when serum potassium levels are too high. Polystyrene sulfonate​ (Kayexalate) decreases potassium levels by eliminating potassium through the intestinal​ route, not through urination.

What instruction should the nurse give a client who is starting a new antihypertensive​ medication? A) Drink alcohol with the medication. B) Rise slowly. C) Increase salt intake. D) There is no need to report​ headaches; they are expected.

B Rationale Instructing clients to rise slowly to prevent dizziness will help avoid falls. Clients should avoid drinking alcohol with antihypertensive​ medications, to prevent an increased risk of dizziness. Clients should be instructed to report headaches and any other adverse effects from the medications. Increasing salt intake is not an appropriate instruction for clients who are taking antihypertensive medications.

The nurse is preparing to administer procainamide hydrochloride​ (Pronestyl) to a client. Which statement by the nurse correctly describes the mechanism of action for this​ medication? This medication works by increasing conduction​ velocity, which delays repolarization. This medication works primarily by slowing the conduction velocity and delaying repolarization. This medication increases the refractory​ period, which can stabilize dysrhythmias. This medication shortens the refractory period by accelerating repolarization and decreasing the duration of the action potential.

B Rationale Procainamide​ (Pronestyl) is a class 1A sodium channel​ blocker, and it works by slowing the conduction velocity and delaying repolarization. Procainamide does not shorten or increase the refractory​ period, nor does it decrease the duration of the action potential. Procainamide decreases conduction velocity rather than increasing it.

The nurse is reviewing the pathophysiology of the immune system. Which condition may occur in clients who have continual secretion of the immune response​ chemicals? A) Hypertension B) Rheumatoid arthritis C) Diabetes D) Peptic ulcers

B Rationale Rheumatoid arthritis is an autoimmune condition that can occur when cytokines are chronically secreted.​ Hypertension, peptic ulcer​ disease, and diabetes are not autoimmune disorders and are not affected by chronic cytokine secretion.

The nurse is assessing the skin of a newly admitted​ 70-year-old client when a rash is noted on the trunk of the body. What​ condition, caused by a virus but preventable by​ vaccination, does the nurse​ suspect? A) Rheumatoid arthritis B) Shingles C) Psoriasis D) Atopic dermatitis

B Rationale Shingles is caused by the varicella zoster​ virus, which can be prevented by vaccination. Psoriasis is an inflammatory skin condition associated with an autoimmune response. Psoriasis is characterized by plaques with surrounding inflammation. Atopic​ dermatitis, also referred to as​ eczema, is a chronic skin condition. It can be exhibited as oozing blisters or dry patches on the skin. Rheumatoid arthritis is a​ chronic, progressive autoimmune disease that causes joint inflammation and disfigurement of the affected joints.

The nurse is caring for a client who has been placed on streptokinase for the treatment of a blood clot. The nurse anticipates that this medication will be administered by which​ route? A) Subcutaneous injection B) Intravenous infusion C) Intramuscular injection D) Oral

B Rationale Streptokinase is given by intravenous infusion. Streptokinase is not given orally or by subcutaneous or intramuscular injection.

The​ client, a​ 35-year-old woman with a history of rheumatoid arthritis​ (RA), was recently diagnosed with hypertriglyceridemia. A statin has been prescribed. When teaching the client about taking this​ medication, which instructions will the nurse​ include? A) When taking​ statins, diet and lifestyle changes are not necessary. B) Take the statin medication in the evening for optimal results. C) It is safe to take statins while also taking immunosuppressants. D) A fibric acid agent may also be prescribed to take with your statin.

B Rationale The client should take statins in the​ evening, because cholesterol production by the liver is highest at night. Taking statins can produce a reduction in​ LDL, VLDL, and triglyceride cholesterol levels in the​ blood, which must be sustained by the client through continued drug adherence and by maintaining diet and lifestyle changes. Statins are contraindicated in clients who are taking fibric acid agents or immunosuppressants.

The nurse is assessing a client with alkalosis who is being treated with acid agents. What adverse effects might the nurse observe when treating this client with ammonium​ chloride? A) Yellow skin color B) Central nervous system depression C) Nausea and vomiting D) Decreased respiratory rate

B Rationale The most severe adverse effect that the nurse might observe when treating a client​'s alkalosis with an acid agent would be central nervous system depression. Yellow skin​ color, or​ jaundice, is not an adverse effect of treatment with ammonium chloride. Decreased respiratory​ rate, nausea, and vomiting are possible adverse effects of sodium​ bicarbonate, not ammonium​ chloride, administration.

The nurse is reviewing the orders of a client admitted with a diagnosis of gastrointestinal​ (GI) bleeding. Which order would the nurse​ question? A) Acetaminophen​ (Tylenol) 650 mg PO as needed for fever​ (38.4 C) every 4 hours B) Ibuprofen​ (Advil) 200​ mg, 2 tablets PO every 6 hours as needed for pain C) Serum hemoglobin and hematocrit levels now D) Intravenous infusion of​ 5% dextrose in​ half-normal saline at 125​ mL/hr

B Rationale The nurse would question the order of ibuprofen​ (Advil) 2 tablets as needed for pain because ibuprofen should not be given to clients with GI​ bleeding; bleeding is an adverse effect of this drug. An order for hemoglobin and hematocrit levels would be necessary to determine whether blood volume levels are depleted in a client with GI bleeding. Intravenous access and continuous fluid administration is an appropriate order that presents no reason to question the health care provider. Acetaminophen would be the drug of choice for temperature elevation in a client with GI bleeding.

Electrical impulses in heart

Begins at - SINOATRIAL NODE (set pace for the heart) 2nd antrioventicular node 3rd Bundle of His 4th Divide into Right and Left Bundle Branch 5th Purkinje Fibers (Both Atrium contract first, the both ventricles)

A client is experiencing a nonacute fluid volume deficit after walking to a nearby clinic for an appointment on a very warm summer day. The client feels slightly thirsty but does not feel lightheaded or have other problems. The nurse monitors the client​'s blood pressure and finds it to be slightly low​ (100/72 mmHg). To efficiently and comfortably bring the client​'s fluid volume back to a more normal​ level, which intervention would the nurse​ implement? A) IV administration of a hypertonic solution B) Teaching the client to drink approximately​ 2,500 mL of water per day C) Inserting a feeding tube and administering fluids via the feeding tube D) IV administration of an isotonic solution

B Rationale When a client is dehydrated​ (fluid volume​ deficit) and it is a nonacute​ situation, the nurse needs to teach the client to drink approximately​ 2,500 mL of water per​ day, especially when walking or exercising in the sun during the summer months. If the client is not able to drink approximately​ 2,500 mL of water per​ day, this amount of fluid can be administered through a feeding tube.​ However, there is no indication that this client is unable to drink fluids.During acute​ situations, isotonic IV fluids are administered to return blood volume and blood pressure to within normal parameters. A client experiencing a severe fluid volume deficit can develop hypovolemic shock if immediate fluid replacement is not administered. Administration of a hypertonic solution would make dehydration even​ worse, because it would draw more fluid from the cells into the intravascular space.

A client was diagnosed with hyperlipidemia one month ago. Which client actions indicate that the client has a good understanding of how to lower lipid​ levels? A) The client plans to lose at least​ 10% to​ 20% of her body weight within 6 months. B) The client can verbally express how to​ self-administer her own​ medication, indicates the correct​ dose, and describes the most common side effects of the drug. C) The client adheres to a​ low-calorie diet. D) The client has started to exercise by walking at a very fast pace once per week.

B Rationale When evaluating how well the client understands the treatment​ plan, the nurse will assess whether the client is able to demonstrate knowledge of appropriate​ self-administration of the​ medication, knows the correct dose and common side effects of ordered​ medications, and understands when to notify the health care provider. Clients with hyperlipidemia do not always need a​ low-calorie diet, nor do they always need to lose weight. Other factors must be considered before expecting a client with hyperlipidemia to adhere to a low calorie diet or​ weight-loss plan. The safety of exercise must be discussed with and agreed upon by the health care​ provider, but several times a week is generally recommended.

The nurse on a surgical unit is assessing the incision of a client who underwent an exploratory laparotomy. Which incision description indicates a normal inflammatory​ response? ​A) Odor, necrosis, and hot to touch B) ​Redness, edema, and warmth to touch C) ​Drainage, pallor, and pain D) Pink​ skin, separation, and pain

B Rationale ​Redness, edema, and warmth to touch indicate that the protective mechanism of inflammation is present. Although it is normal to have some drainage from the area of the​ incision, pallor and pain are not a part of the inflammatory response.​ Odor, necrosis, and hot to touch are abnormal clinical manifestations that could indicate a complication warranting notification of the health care provider. These are not part of the inflammatory response.​ Separation, pain, and pink skin are not associated with inflammation.

What is the most common physical issue related to Nonsteroidal Antiinflammatory Drug​ (NSAID) therapy that the nurse should assess for before and during​ administration? Autoimmune disorder Gastrointestinal​ (GI) bleeding Dysrhythmia Migraine

B The most common major concern with the administration of Nonsteroidal Antiinflammatory Drug​ (NSAID) is a client history or occurrence of GI bleeding or peptic ulcer disease. The client may be receiving the NSAID due to an autoimmune disorder. Migraines and dysrhythmias are not contraindications for NSAID​ therapy, but they should be monitored.

While administering magnesium sulfate to Mr.​ Stevens, for which clinical manifestations of magnesium toxicity do you need to​ watch? A) Seizures B) Depressed tendon reflexes C) Twitching muscles D) Nausea and vomiting

B Toxicity associated with magnesium sulfate administration​ (hypermagnesemia) includes CNS depression and depressed deep tendon reflexes.​ Seizures, muscle​ twitching, and nausea and vomiting are clinical manifestations associated with hypomagnesemia and would not indicate toxicity.

Your client is a professional soccer player whose heart rate is often less than 60 beats per minute. She is​ healthy, with adequate cardiac output. What is her condition likely to​ be? A) Atrial flutter B) Athletic heart syndrome C) Sinus bradycardia D) Tachydysrhythmia

B With athletic heart​ syndrome, the heart beats slower and more forcefully with adequate cardiac​ output, as the result of​ long-term training of the heart muscle. Some athletes have a heart rate of less than 60​ beats/min, but have no symptoms of bradycardia. With sinus​ bradycardia, the heartbeat is less than 60​ beats/min, with inadequate cardiac output. With tachydysrhythmia and atrial​ flutter, the heartbeat is more than 100​ beats/min. Next Question

Lorencio de Santos is a​ 54-year-old man who was admitted to the hospital with severe atrial fibrillation. After receiving a surgically implanted​ ICD, Mr. de Santos is worried because he does not understand what has happened. What explanation by the nurse is the most important at this time to help Mr. de Santos understand the purpose of an​ ICD? A) Tell him the ICD will prevent his caffeine intake from aggravating the dysrhythmias. B) Explain how the ICD defibrillates to regulate his heart rate. C) Tell him the ICD will correct his electrolyte imbalances. D) Explain cardioversion in simple terms.

B You should explain in understandable terms that an ICD is a permanent device that uses electric pulses or shocks to help control​ life-threatening arrhythmias. Mr. de Santos has not had​ cardioversion, so there is no need to explain it. The issues with his electrolyte imbalances and caffeine intake would be part of your later role of assessment and​ correction, so that dysrhythmias are not exacerbated.​ However, electrolyte imbalances and the effects of caffeine are not directly corrected by the ICD. Next Question

An older adult client undergoing cancer chemotherapy is receiving total parenteral nutrition​ (TPN) through a central line. The nurse knows that the client is at risk for which mechanical ​complications? Select all that apply. Bone demineralization Brachial plexus injury Hemothorax Fluid volume overload Endocarditis

B C E

The nurse is admitting a new client with a medical diagnosis of fluid volume​ deficit, or dehydration. The client asks the nurse what causes dehydration. What does the nurse teach the client about the factors and conditions that can lead to fluid volume deficit ​(dehydration)? A) Having dry​ mouth, orthostatic​ hypotension, and decreased skin​ turgor, capillary​ refill, and urine output B) Exercising outdoors during very hot weather conditions C) Dysphagia secondary to a stroke​ (cerebral vascular​ accident) D) Eating contaminated food that causes severe nausea with vomiting and diarrhea E) Taking prescribed​ "water pills"​ (diuretics) to control hypertension​ (high blood​ pressure)

B) Exercising outdoors during very hot weather conditions C) Dysphagia secondary to a stroke​ (cerebral vascular​ accident) D) Eating contaminated food that causes severe nausea with vomiting and diarrhea E) Taking prescribed​ "water pills"​ (diuretics) to control hypertension​ (high blood​ pressure)

The nurse is administering pharmacotherapy to a client with severe hyperkalemia. Which agent helps the body to eliminate potassium through​ urination? A) Calcium gluconate B) Furosemide​ (Lasix) C) Polystyrene sulfonate​ (Kayexalate) D) Insulin with dextrose

B) Furosemide​ (Lasix)

What can cause secondary ​hypertension? Select all that apply. A) Rheumatoid arthritis B) Cushing disease C) Use of oral contraceptives D) Use of corticosteroids E) Use of​ B-complex vitamins

B, C, D Hypertension can be​ primary, when there is no identifiable​ cause; or it can be​ secondary, where the cause or causes can be identified and treated. Some causes of secondary hypertension are use of oral​ contraceptives, Cushing​ disease, and use of corticosteroids. Use of​ B-complex vitamins and rheumatoid arthritis do not cause hypertension.

A client who is receiving enalapril​ (Vasotec) for heart failure asks the nurse about adverse effects. The nurse teaches the client about which common adverse effects of​ angiotensin-converting enzyme​ (ACE) ​inhibitors? A) Hypertension B) Angioedema C) Cough D) Hyperkalemia E) Hypokalemia

B, C, D Rationale Possible adverse effects of​ angiotensin-converting enzyme​ (ACE) inhibitors include​ headache, dizziness, orthostatic​ hypotension, cough,​ syncope, angioedema,​ hyperkalemia, and blood dyscrasias.

Which conditions can affect ​hemostasis? Select all that apply. A) Pneumonia B) Myocardial infarction C) Liver disease D) Cerebral vascular accident E) Deep venous thrombosis

B, C, D, E Myocardial infarction​ (MI), liver​ disease, cerebral vascular accidents​ (CVAs), and deep venous thrombosis​ (DVT) all affect hemostasis because they affect the flow of blood. MI is caused by a blocked artery. Liver disease leads to less thrombopoietin production and interferes with clotting. A CVA is caused by a clot in the brain that disrupts blood​ flow, and DVT impairs blood flow in the legs. Pneumonia is a collapse of the alveoli in the​ lungs; it affects the flow of​ air, not​ blood, in the body.

What are common symptoms of myocardial infarction ​(MI)? Select all that apply. A) Hyperventilation B) Chest pain or tightness C) Excessive sweating D) Nausea and vomiting E) Unusually pale skin

B, C, D, E Clinical manifestations of MI include extreme chest pain or​ tightness, diaphoresis​ (sweating), nausea and​ vomiting, and pallor. Shortness of​ breath, not​ hyperventilation, is another common symptom of MI.

The nurse in the emergency department begins intravenous administration of a thrombolytic​ medication, streptokinase​ (Streptase), to a newly admitted client who had a myocardial infarction​ (MI). Which instructions should the nurse provide to the client​ (and family) related to this medication​'s mechanism of action and adverse ​effects? Select all that apply. ​A) "You will be receiving this medication for the next three to four days. Be sure to let the nursing staff know if you experience any​ bleeding, fever, or​ chills." B) ​"Clients receiving this medication sometimes experience bleeding. If you notice any​ bleeding, use the nurse call button immediately so that we can control the​ bleeding." C) ​"This medication works to break up and dissolve blood clots that can block arteries. It is often used as early treatment for heart​ attack." D) ​"I will be checking your blood pressure often after you receive this medication to make sure you don​'t experience​ complications." E) ​"Let me know if you have any allergic reactions such as​ fever, chills,​ itching, or problems breathing after you receive this​ medication."

B, C, D, E Rationale Clients have a better chance of surviving and recovering from myocardial infarction if they receive a thrombolytic​ drug, such as streptokinase​ (Streptase), within 12 hours after the heart attack occurs. Thrombolytics are most effective when administered between 20 minutes and 12 hours after the onset of MI symptoms.​ Ideally, the client presents to the emergency department and has the drug administered within 30 minutes or less. Thrombolytics work by dissolving major clots​ quickly, which helps restore some blood flow to the heart muscle and prevent damage to heart muscle.​ However, the blood flow may not return completely to​ normal, and some muscle damage may occur. Additional​ therapy, such as cardiac catheterization or​ angioplasty, may be needed. The most commonly reported adverse reactions to streptokinase treatment include unusual​ bleeding; severe/uncontrolled​ hypertension, internal​ hemorrhage, and allergic reactions​ (e.g., fever,​ chills, itching, and problems​ breathing, which are the most commonly reported allergic reactions to intravenous use of streptokinase after​ MI). Research has shown that thrombolytic therapy is of no value if administered more than 24 hours after the onset of MI.​ Thus, it would be incorrect for the nurse to teach the client that thrombolytics will be given for four days.

What is one important difference between peripheral parenteral nutrition and central parenteral​ nutrition? Only peripheral infusion requires an infusion pump. Central vein administration has fewer risks. Peripheral vein solutions have a lower osmolality. Peripheral vein administration is used for​ long-term therapy.

Peripheral vein solutions have a lower osmolality.

The nurse is educating an adult client on the causes of metabolic acidosis. Which causes would the nurse include in the client ​education? Select all that apply. ​A) Potassium-wasting diuretics B) Chronic renal failure C) Tissue hypoxia D) Diabetes mellitus E) Acute diarrhea

B, C, D, E Rationale Diabetes mellitus can develop into diabetic ketoacidosis. Tissue hypoxia can alter electrolytes and cause metabolic acidosis. Chronic renal failure can lead to metabolic acidosis. Acute diarrhea alters the aciddash-base balance in the body and can cause metabolic acidosis. The use of​ potassium-wasting diuretics may cause metabolic​ alkalosis, not acidosis.

The nurse is assessing an adult client with new onset of primary hypertension. Which statements accurately describe primary hypertension in the adult ​client? Select all that apply. A) Primary hypertension leads to secondary hypertension. B) Stage 1 primary hypertension is a blood pressure of 140dash-​159/90dash-99. C) Stage 2 primary hypertension is a blood pressure of​ 160/100 or higher. D) Primary hypertension is managed with medications. E) No specific cause might be found for primary hypertension.

B, C, D, E Rationale Primary hypertension is managed with medications and lifestyle changes. Primary hypertension may not have a discernible cause. Stage 1 hypertension is defined as a blood pressure reading of 140dash-​159/90dash-99 and requires intervention to reduce the numbers. Stage 2 hypertension is a blood pressure reading of​ 160/100 or higher. Primary hypertension and secondary hypertension have different​ etiologies; primary hypertension does not lead to secondary hypertension.

The nurse is assessing an adult client with fluid loss from diarrhea and vomiting. Which tests would indicate that the client is actually​ dehydrated? Select all that apply. Chest​ x-ray Serum osmolality Serum electrolytes Hemoglobin and hematocrit Urine specific gravity

B, C, D, E Rationale When a client is​ dehydrated, the serum electrolytes would show an elevated sodium level. Serum​ osmolality, hemoglobin,​ hematocrit, and urine specific gravity would be increased with dehydration. A chest​ x-ray may be part of a diagnostic​ work-up, but would not indicate that a client is dehydrated.

A client who experienced an MI is about to be discharged after one week in the hospital. What issues should you consider when evaluating the effectiveness of the​ medication(s) administered for ​MI? Select all that apply. A) Clear breath sounds B) Decrease in blood pressure C) Decrease in heart rate D) No further occurrence of MI E) Relief of chest pain

B, C, D, E Your role in the evaluation phase of administering medications for MI involves assessing the effectiveness of​ medication(s) for relief of chest​ pain, decrease in heart rate and blood​ pressure, with no further myocardial infarction occurrence. Medications for MI do not affect breath sounds. OK

The CCU nurse works in a hospital that has just instituted a new​ policy, which requires that an individual be dedicated to watching the cardiac monitors to keep tabs on all clients with tachydysrhythmias. Which clients would require the type of continuous monitoring mandated by this new ​policy? Select all that apply. A) A client with sick sinus syndrome B) A client with atrial fibrillation C) A client with torsades de pointes D) A client with atrioventricular conduction block​ (AV block) E) A client with atrial flutter

B, C, E Rationale Atrial​ flutter, atrial​ fibrillation, and torsades de pointes are all tachydysrhythmias​ (torsades de pointes is a lethal​ tachydysrhythmia). Sick sinus syndrome and AV block are bradydysrhythmias.

When administering a recommended dose of acetaminophen​ (Tylenol) to an adult​ client, what results can you​ expect? Select all that apply. No progression of inflammation Reduction of inflammation Reduction of pain Reduction of fever Few adverse effects

B, C, E Acetaminophen​ (Tylenol) is used to reduce fever and mild to moderate pain. Although​ life-threatening effects can occur with chronic ingestion or​ overdose, adverse effects in adults are typically negligible when taken in recommended doses for a short period of time. It does not have​ anti-inflammatory properties.

What serum laboratory values should you as a nurse assess before administering a diuretic to a ​client? Select all that apply. A) Hemoglobin B) Uric acid C) Electrolytes D) Hematocrit E) Glucose

B, C, E Diuretics are often used as a​ first-line treatment in the management of hypertension.​ However, they are not given to clients who are anuric. Diuretics increase the excretion of water and electrolytes by the urinary​ system, thereby causing a potential risk for dehydration and excessive loss of electrolytes. Electrolyte levels should be monitored during​ therapy, assessing for​ hypochloremia, hypomagnesmia,​ hyponatremia, and alterations in potassium levels. Because diuretics can cause hyperglycemia and​ hyperuricemia, careful assessment of​ glucose, as well as uric acid​ levels, is important. The hemoglobin or hematocrit levels have no association with the administration of a diuretic.

Both beta1​-adrenergic antagonists and nonselective​ beta-adrenergic antagonists are used in the treatment of angina. Which are true statements about these ​medications? Select all that apply. A) Only​ beta-antagonists may decrease cerebral blood flow. B) Both nonselective and​ beta-adrenergic antagonists may hide the symptoms of hypoglycemia. C) Both nonselective and​ beta-adrenergic may worsen angina caused by vasospasm. D) Nonselective beta antagonists are weaker than​ beta-antagonists. E) Nonselective beta antagonists may cause bronchoconstriction.

B, C, E Nonselective​ beta-adrenergic antagonists may cause​ bronchoconstriction, whereas beta1​-adrenergic antagonists are less likely to do so. Both types may mask the symptoms of hypoglycemia because of the depression of​ tachycardia, which is a symptom of hypoglycemia. Both types can worsen vasospastic angina. Both typeslong dash—not just beta1​-antagonistslong dash—are contraindicated for clients with low cerebral blood​ flow, as they can decrease cerebral tissue perfusion. Nonselective​ beta-adrenergic antagonists are not weaker than beta1​-adrenergic antagonists.

Which statements are true concerning venous thromboembolism ​(VTE)? Select all that apply. A) VTE occurs very rarely. B) An example of VTE is pulmonary embolus. C) In​ VTE, procoagulants accumulate and overpower natural anticoagulation elements. D) VTE occurs when blood flow through a vein is increased. E) DVT is a form of VTE.

B, C, E Pulmonary embolus is an example of VTE. It is a clot in the lung frequently associated with death. VTE generally occurs when blood flow through a vein is​ slowed, not increased. This stasis allows procoagulants to accumulate and overpower natural anticoagulation elements in the blood.​ DVT, or deep venous​ thrombosis, is a form of VTE because it occurs largely due to slower blood flow in the leg veins. VTE is​ common, not rare.

Which agents are used to treat severe ​hyperkalemia? Select all that apply. A) Magnesium citrate​ (Citrate of​ Magnesia) B) Furosemide​ (Lasix) C) Potassium chloride D) Sodium polystyrene sulfonate​ (Kayexalate) E) Insulin with glucose

B, D, E Treatment for severe hyperkalemia​ (elevated potassium) may include furosemide​ (Lasix), a diuretic that reduces​ potassium; insulin with glucose or​ dextrose, which causes potassium to enter the​ cells; calcium gluconate or calcium​ chloride, which decrease cardiac​ complications; sodium​ bicarbonate, which corrects​ acidosis; or sodium polystyrene sulfonate​ (Kayexalate), which binds with potassium in the intestinal tract for elimination. Administering potassium chloride would make the hyperkalemia worse. Magnesium citrate​ (Citrate of​ Magnesia) is given to clients who have​ hypomagnesemia, not hyperkalemia.

The nurse is providing care for a client who experienced a myocardial infarction​ (MI) 48 hours ago. This client reports occasional sharp chest pain that moved to his​ neck, shoulder,​ abdomen, and back after the MI. He is without​ dyspnea, and his oxygenation saturation level is​ 98% (normal: ​95%dash-​100%). Which nursing actions are consistent with the goals of treatment for this ​client? Select all that apply. A) Turn and reposition the client every 1 to 2 hours. B) Discuss pain management strategies with the client and family members at the bedside. C) Administer​ high-flow oxygen. D) Administer prescribed medications to restore circulation and oxygen supply to the heart. E) Reduce the heart​'s oxygen demand.

B, D, E Rationale Goals of treatment after a client experiences myocardial infarction​ (MI) include restoring circulation and oxygen supply to the damaged portion of the​ heart, as quickly as​ possible; reducing myocardial oxygen demand with​ nitrates, beta​ blockers, and calcium channel blockers to decrease the risk of subsequent MIs or the risk of extending the current area of​ damage; managing MI pain and​ anxiety; controlling or preventing dysrhythmias or other​ post-MI complications; reducing​ post-MI mortality;​ and, after cardiac arrest has​ occurred, restarting the heart and restoring cardiac function to as near normal as possible. There is no evidence that this client is hypoxic and in need of​ oxygen; further assessment must be completed before initiating this nursing action.​ Also, repositioning and turning a client after myocardial infarction​ (MI) increases energy expenditure and may lead to​ dyspnea, so this nursing action is not warranted at this time.

The nurse knows that which symptoms can occur with either​ right- or​ left-sided heart ​failure? Select all that apply. A) Lower extremity edema B) Pallor C) Orthopnea D) Palpitations E) Nocturia

B, D, E Rationale ​Nocturia, palpitations, weight​ gain, and pallor can occur with either​ right- or​ left-sided heart failure. Lower extremity edema is a symptom of​ right-sided heart failure. Orthopnea is a symptom of​ left-sided heart failure.

Which signs are associated with aspirin​ (Bayer) toxicity? Select all that apply. Inflammation of joints Decreased hearing Blood clots Stomach discomfort Tinnitus

B, D, E Signs of an aspirin overdose include​ tinnitus, stomach​ discomfort, and decreased hearing. Aspirin may prevent blood​ clots, not cause​ them; it also acts as an​ anti-inflammatory agent, not an inflammatory agent.

The nurse is teaching the client about his recently diagnosed angina pectoris. She tells him that his type of angina is not predictable and is associated with a higher risk for myocardial infarction​ (MI). What types of angina might this client ​have? Select all that apply. A) Vasospastic angina B) Unstable angina C) Stable angina D) Prinzmetal​'s angina E) Silent angina

B, E Rationale Two types of​ angina, unstable angina and silent​ angina, place a client at higher risk for myocardial infarction. In​ addition, clients with silent angina are at higher risk for sudden death. Unstable angina is characterized as​ "unstable" for two reasons.​ First, symptoms occur in a more random and unpredictable​ manner, often without any apparent trigger​ (often occurs at​ rest, or may awaken a person from a restful​ sleep). Secondly, unstable angina is most often caused by an actual rupture of a​ plaque, which often leads to the formation of a blood clot in the coronary artery that partially blocks the artery. If the clot causes complete obstruction of the​ artery, the heart muscle​ (supplied by that affected​ artery) is in grave danger of sustaining irreversible damage and the imminent risk of a complete myocardial infarction is very high. Unstable angina is considered a medical emergency. Cardiac ischemia​ (silent angina) refers to a decrease of​ oxygen-rich blood flowing to the heart muscle caused by a narrowed or blocked coronary artery. If ischemia is severe or lasts for an extended​ time, it can lead to myocardial infarction and heart​ tissue/muscle death. With silent​ angina, there is no pain. Silent angina​ (ischemia) may disturb the heart​'s normal rhythm​ (causing ventricular tachycardia or ventricular​ fibrillation) and may interfere with the heart​'s pumping ability and cause loss of consciousness. People who have diabetes or have had previous heart attacks are especially at risk for developing silent angina. Clients with silent angina are at higher risk for MI and sudden death.Stable angina is the most common type of chest pain and occurs most often during activity or stress. Stable angina is caused by diminished blood flow through the coronary arteries to the heart muscle​ (myocardium). A higher risk for myocardial infarction​ (MI) is not associated with this type of angina. Vasospastic​ angina, or coronary artery​ spasm, refers to a temporary and sudden narrowing​ (spasm) of one of the coronary arteries. This spasm slows or stops blood flow through the coronary artery and reduces the amount of​ oxygen-rich blood getting to the heart muscle. A higher risk for myocardial infarction​ (MI) is not associated with this type of angina. Prinzmetal​'s angina is another name for vasospastic angina. Prinzmetal​'s ​angina, also referred to as​ "variant" angina, is a temporary and sudden increase in coronary vascular tone​ (or vasospasm), causing a marked and transient reduction in arterial luminal diameter and leading to chest pain​ (angina). A higher risk for myocardial infarction​ (MI) is not associated with this type of angina.

The nurse is preparing to administer warfarin to a client with a history of deep venous thrombosis. The nurse correctly states that this medication inhibits clotting by affecting which clotting factors produced in the ​liver? Select all that apply. A) Factor V B) Factor II C) Factor I D) Factor VI E) Factor VII

B, E Rationale Warfarin​'s mechanism of action is to inhibit the synthesis of several different clotting​ factors, including Factors II and VII. Factors​ I, V, and VI are not affected.

The nurse is assessing an adult client with signs of a serious bacterial infection. Which facts about the etiology of bacterial infections will guide the​ nurse's assessment when looking for sources of ​infection? Select all that apply. Bacteria can be generated by the immune system. Bacteria can be found in food products. Bacteria can enter a break in the skin. Bacteria can be inhaled. Bacteria can enter the body through mucous membranes.

Bacteria can be found in food products. Bacteria can enter a break in the skin. Bacteria can be inhaled. Bacteria can enter the body through mucous membranes.

The nurse is preparing a teaching plan for administering immunizing agents. What should the nurse include as teaching points for clients or ​caregivers? Select all that apply. Be aware of common side effects of the vaccine. Report a temperature over 103degrees°F ​(39.4degrees°​C). Comply with the vaccination schedule. Expect a change in mental status after administering a vaccine. A mild pain reliever to minimize pain or other discomforts may be administered.

Be aware of common side effects of the vaccine. Report a temperature over 103degrees°F ​(39.4degrees°​C). Comply with the vaccination schedule. A mild pain reliever to minimize pain or other discomforts may be administered.

Place each adverse effect in the appropriate classification. ​Instructions: Use the dropdown menus in the left​ column, to select the correct category for each statement in the right column. Category Statement

Bile Acid Resins Constipation Statins Joint pain Nicotinic Acid Flushing Statins Fatigue Bile Acid Resins Potentiation of action of warfarin​ (Coumadin) Fibric Acid Hot flashes Statins Muscle pain Nicotinic Acid Gout Fibric Acid Blurred vision Bile Acid Resins Potentiation of action of digoxin​ (Lanoxin) Statins Headache Bile Acid Resins Bloating Fibric Acid Abdominal pain Nicotinic Acid Hepatotoxicity Fibric Acid Dizziness

Select all that apply. Blood pressure Weight gain Odd sensations in extremities Blood urea nitrogen​ (BUN) and creatinine levels Hair distribution

Blood pressure Odd sensations in extremities Blood urea nitrogen​ (BUN) and creatinine levels Hair distribution

IV Fluids for fluid replacement.

Blood products for blood loss. HYPERTONIC fluids for sodium and salt replacement. HYPOTONIC fluids for dehydration. Assess for: fluid overload, fluid in lungs and cardiovascular stress.

After the blood has infused for 15​ minutes, Mr. Adams complains of chills and a headache. His oral temperature is​ 101.6°F (38.7°C). What is the most likely cause of his clinical​ manifestations? Pain medication reaction Blood transfusion reaction Anxiety related to his accident Severe pain

Blood transfusion reaction

The client has been taught nutrition needs for healthy skin. Which client diet selection best indicates to the nurse that understanding has taken​ place? A bowl of​ chili, crackers, and a baked potato A Caesar​ salad, broth, and a chocolate chip cookie Hamburger​ patty, green leafy salad with​ dressing, and steamed broccoli Boiled​ potatoes, steamed green​ beans, baked​ chicken, and fruit

Boiled​ potatoes, steamed green​ beans, baked​ chicken, and fruit

The nurse is assessing a client with abdominal pain and a history of daily nonsteroidal antiinflammatory drug​ (NSAID) use for 6 months. Which assessment finding causes the nurse the most​ concern? Nausea related to the intake of food Dry mucous membranes in the oral cavity Bright red blood in the stools Pain in the epigastric area of the abdomen

Bright red blood in the stools rationale: could have an active bleed in the GI tract

The nurse is explaining the stages of pressure ulcers to a group of new RNs. Which area of the body is most likely to develop a stage III​ ulcer? Sacrum Ankle Buttocks Patella

Buttocks

When the osmolality of plasma​ increases, what is triggered in addition to the thirst​ mechanism? A) Excretion of bicarbonate B) Excretion of sodium chloride C) Release of antidiuretic hormone D) Release of epinephrine

C An increase in the osmolality of plasma indicates a concentration of solutes and the need for additional water. This condition triggers the thirst mechanism and the release of antidiuretic hormone. In​ turn, the kidneys reabsorb water and sodium chloride. The increase of osmolality in the plasma does not trigger the excretion of bicarbonate or sodium chloride. Epinephrine is not released as a direct response to increased osmolality of the plasma. Bicarbonate is excreted when a​ person's serum pH level becomes higher than 7.45.

Mr. Parker is prescribed a​ lipid-lowering agent. As you write up the medication reconciliation​ form, you notice that he is also taking several other medications. Which action is most important before Mr. Parker is​ discharged? A) Obtain Mr.​ Parker's pharmacy information B) Schedule a​ follow-up lipid profile C) Assess for possible​ drug-drug interactions D) Explain lifestyle modifications to Mr. Parker

C Assessing for possible​ drug-drug interactions is the most important action to ensure Mr.​ Parker's safety. Pharmacy information is not​ necessary, although it is good practice to transmit medication reconciliation forms to the pharmacy to keep Mr.​ Parker's record up to date. Scheduling​ follow-up lab tests and explaining lifestyle modifications are important components of successful​ pharmacotherapy, but assessing for possible​ drug-drug interactions is more critical.

Which drugs are considered the drugs of choice for vasospastic​ angina? A) Nitrates B) ​Beta-adrenergic antagonists C) Calcium channel blockers D) Beta blockers

C Calcium channel​ antagonists, also called calcium channel blockers​ (CCBs), relax arterial smooth​ muscle, thereby lowering blood pressure. CCBs are considered the drugs of choice for vasospastic angina because they dilate the coronary​ arteries, bringing more oxygen to the heart muscle. For clients with persistent​ symptoms, CCBs may be used as adjunctive therapy with​ nitrates, but nitrates are not the first choice for vasospastic angina. Beta blockers​ (beta-adrenergic antagonists) are not effective for treating vasospastic​ angina, and may even worsen this condition.

As a​ nurse, you will be teaching clients with heart failure to follow a healthy lifestyle to slow or halt the progression of their disease. What is an example of a reasonable​ salt-restricted diet? A) 175-200 ​mg/day B) Not more than​ 2000/mg day C) Less than 1500​ mg/day D) 2000-2200 ​mg/day

C Following a​ salt-restricted diet of less than 1500 mg of salt per day is a reasonable lifestyle change that you can encourage your clients to follow. An amount of 175dash-200 ​mg/day is not reasonable in our​ culture, and 2000 or 2200​ mg/day is too high an upper limit.

Based on Ms.​ Sanchez's condition, what order do you expect to receive from the healthcare​ provider? A) Indwelling urinary catheter B) ​3% sodium bolus C) Intravenous fluids D) Clear liquid diet

C Intravenous isotonic fluids are administered to restore fluid balance in a client with dehydration. Ms. Sanchez probably has very little urine in her bladder because of her lack of fluid intake. Inserting an indwelling catheter would only confirm that little urine is​ present; it would not address Ms.​ Sanchez's dehydration. Ms. Sanchez is not tolerating oral intake and therefore a clear liquid diet would not be warranted until her symptoms resolve. A​ 3% sodium bolus is a hypertonic solution that would contribute further to her intravascular dehydration.

The nurse is caring for a client who is taking an anticoagulant for a coagulation disorder. Which vital sign parameter indicates that the client may be experiencing​ bleeding? A) Respiratory rate of 30​ breaths/min B) Blood pressure of​ 180/100 mmHg C) Heart rate of 150​ beats/min D) Body temperature of 102.0degrees°F ​(39.2degrees°​C)

C Rationale: An increased heart rate can be an indication of bleeding. Neither an increased temperature nor an increased respiratory rate indicates bleeding. A decreased blood​ pressure, not an increased blood​ pressure, indicates bleeding

Mrs. Simmons was admitted to the hospital for hyperglycemia. As you take a medication​ history, she reports that she started taking​ over-the-counter niacin supplements to reduce her cholesterol because it is cheaper than her prescription medication. As the​ nurse, you know that​ niacin: A) Lowers blood glucose levels B) Has no effect on lipid levels C) Can elevate blood glucose levels D) Is contraindicated for diabetic clients

C Nicotinic​ acid, also called​ niacin, can elevate blood glucose levels. Niacin is not contraindicated for diabetic​ clients, but should be used with caution. Niacin is known to decrease LDL and triglyceride levels.​ However, niacin will​ increase, not​ lower, blood glucose levels.

What instructions should be included in the client education for​ lipid-lowering agents? A) Go to the emergency department if any adverse effects are experienced. B) Take these medications first thing in the morning on an empty stomach. C) See the health care provider for periodic lipid and liver profiles. D) Diet and lifestyle modifications are unnecessary but encouraged.

C Periodic monitoring of lipid and liver profiles is necessary to evaluate pharmacotherapy. Diet and lifestyle modifications are often​ necessary, as well as​ encouraged, to achieve desired lipid levels. Maximum drug benefits are achieved when the medication is taken at the evening meal. Although clients should notify the health care provider if they experience adverse​ effects, a trip to the emergency department is usually not necessary.

What is the term for the amount of blood that remains in the ventricles during​ diastole, right before the heart​ contracts? A) Afterload B) Stroke volume C) Preload D) Ejection fraction

C Preload is the amount of blood that remains in the ventricles during​ diastole, right before the heart contracts. Afterload is the force that is necessary for the heart to eject​ blood, during​ contraction, to the lungs and the rest of the body. Stroke volume is the amount of blood that each heartbeat ejects to the lungs and the rest of the body. Ejection fraction refers to the percentage of blood ejected from the ventricles during systole

The nurse is caring for a client who has been placed on clopidogrel​ (Plavix). Which adverse reaction would the nurse educate the client to report to a health care​ provider? A) Diarrhea B) Abdominal pain C) Bloody stool D) Rash

C Rationale A bloody stool could indicate gastrointestinal​ bleeding, which is a severe adverse reaction to this medication. The client should report this to the health care provider. Abdominal​ pain, rash, and diarrhea are expected adverse reactions and would not require the client to report to the health care provider.

The nurse is providing education to a client who has been started on pharmacotherapy for treatment of a cardiac dysrhythmia. The nurse would educate the client to notify the health care provider if the heart rate exceeds which​ rate? 90 beats per minute 70 beats per minute 100 beats per minute 80 beats per minute

C Rationale A pulse rate of 100 beats per minute is the criterion that clients are told is abnormal and should be reported to the health care provider. Pulse rates of 70 beats per​ minute, 80 beats per​ minute, and 90 beats per minute are​ normal, and the client would not notify the health care provider of these.

The nurse is discussing the ACE inhibitor lisinopril​ (Prinivil) with an adult client. Which statement about ACE inhibitors is​ correct? A) ACE inhibitors end with​ "zide" in their name. B) ACE inhibitors increase blood pressure. C) ACE inhibitors block angiotensin​ II, which prevents vasoconstriction and decreases peripheral resistance. D) ACE inhibitors can decrease the effects of thiazides.

C Rationale ACE inhibitors block angiotensin II. This action prevents the vasoconstriction of arterioles. In​ turn, the vasoconstriction decreases blood pressure by reducing peripheral resistance. ACE inhibitors can increase the effects of​ thiazides, so dosing must be monitored. ACE inhibitor names end with​ "pril"; lisinopril​ (Prinivil) is an example.

The nurse is educating a client about what he should expect as the health care team seeks to identify what type of dysrhythmia he has. Which diagnostic test does the nurse anticipate will be ordered to correctly diagnose the​ dysrhythmia? Measurement of cardiac enzymes Cardiac ultrasound Electrocardiogram Stress test

C Rationale An electrocardiogram is the diagnostic test that will be ordered to correctly diagnose a dysrhythmia. A cardiac​ ultrasound, stress​ test, and the measurement of cardiac enzymes are not used to diagnose dysrhythmias.

The nurse is reviewing the mechanisms the body uses to regulate blood pressure. Which hormone helps regulate blood​ pressure? A) Insulin B) Estrogen C) Antidiuretic hormone​ (ADH) D) Serotonin

C Rationale Antidiuretic hormone​ (ADH) causes vasoconstriction and increases blood volume. Serotonin is a natural hormone in the body that affects sleep and brain​ function, but not blood pressure. Insulin regulates blood sugar. Estrogen is the hormone that helps to regulate​ menstruation, breast​ health, and​ pregnancy; it does not affect blood pressure.

The nurse has administered propranolol​ (Inderal) to several clients on a coronary care unit. Clients of which race are at greatest risk for experiencing the adverse effect of​ bradycardia? Caucasian Hispanic Asian African American

C Rationale Asian clients have an increased risk of developing bradycardia when given propranolol​ (Inderal). African​ American, Caucasian, and Hispanic clients do not.

The nurse is preparing to discharge an adult client who was admitted with heart failure and has been receiving digoxin​ (Lanoxin) and is now on maintenance therapy. Which assessment should concern the​ nurse? A) The client has lost 6 pounds during the admission. B) The client states​ he/she will take his pill every evening. C) The client is nauseated and does not have an appetite. D) The client​'s apical pulse is 64.

C Rationale Digoxin​ (Lanoxin) is a cardiac glycoside that has a positive inotropic effect and antiarrhythmic properties. Nausea and vomiting are signs of digoxin toxicity. An apical pulse of 64 is​ therapeutic; weight loss is normal with digoxin​ (Lanoxin) therapy because of the positive inotropic effect of the​ drug, which helps pump fluid to the kidneys for excretion. Digoxin​ (Lanoxin) should be taken at the same time daily as directed.

The nurse is preparing to administer phenytoin​ (Dilantin) 100 mg IV push to treat a client​'s dysrhythmias associated with digoxin toxicity. How much time should the nurse allow for this​ infusion? A) 5 minutes B) 4 minutes C) 2 minutes D) 3 minutes

C Rationale Dilantin may be administered rapid IV push but no more quickly than 50 mg per minute.​ Therefore, two minutes would be adequate time for administration. In a​ non-emergency administration of​ Dilantin, the dose could be administered more slowly at a rate of​ 1-2 mg per kg per minute at a rate no faster than 50 mg per minute. Three​ minutes, four​ minutes, and five minutes would represent the medication being given too slowly.

The nurse is caring for a client who is being treated for heart failure with digoxin​ (Lanoxin). During​ therapy, which electrolyte imbalance may place the client at risk for digoxin​ toxicity? A) Hypermagnesemia B) Hypernatremia C) Hypokalemia D) Hypocalcemia

C Rationale Hypokalemia due to diuretic​ use, hypomagnesemia,​ hypercalcemia, and impaired renal function can place clients at risk for​ dixogin-induced dysrhythmias.​ Hypermagnesemia, hypocalcemia, and hypernatremia are not associated with a risk for digoxin toxicity.

The nurse is caring for a client who has just experienced a heparin overdose. Which medication does the nurse anticipate will be ordered to treat this​ client? A) Vitamin K B) Dalteparin C) Protamine sulfate D) Desirudin

C Rationale Protamine sulfate is the antidote for​ heparin, so the nurse would expect the health care provider to order this medication. Vitamin K is the antidote for a warfarin​ overdose, not a heparin overdose. Dalteparin is a​ low-molecular-weight heparin, not the antidote for a heparin overdose. Desirudin is a thrombin​ inhibitor, so the nurse would not anticipate that this medication would be ordered to treat a heparin overdose.

The nurse is monitoring a client​'s blood pressure after having administered furosemide​ (Lasix). Which phase of the nursing process is being​ utilized? A) Assessment B) Evaluating C) Implementation D) Planning

C Rationale Taking the client​'s blood pressure after having administered prescribed medication is an element of the implementation phase of the nursing process. The planning phase includes developing a plan of action and formulating goals and outcomes. The assessment phase includes collecting subjective and objective data. In the evaluation​ phase, the nurse evaluates the progress toward the goals and outcomes previously developed in the planning phase.

The nurse is teaching a​ 68-year-old woman, who was admitted with angina pectoris and a history of type 2​ diabetes, why taking her medications as prescribed and making diet and exercise lifestyle changes will be important in treating her angina. Which response indicates that further teaching is​ needed? ​A) "If I follow the treatment​ plan, I will probably have fewer episodes of chest​ pain." B) ​"If I follow the treatment​ plan, which includes exercise and​ diet, my chances of having a heart attack are​ reduced." C) ​"If I exercise and watch my​ diet, my chest pain can be​ cured." D) ​"I can lower my risk for heart attack if I keep​ active, eat​ right, and take my medications as they are​ prescribed."

C Rationale The chest pain of angina is a​ symptom, not a disease to be cured. It alerts both clients and caregivers of a possible​ life-threatening situation. The client has three ways to reduce the risk of heart​ attacks: exercise,​ nutrition, and medication compliance. That same treatment plan supports a decrease in the probability of future angina.

The​ 58-year-old male client with​ Crohn's disease​ (chronic inflammation of the digestive​ tract) was admitted because of hyperlipidemia. His nurse has provided education regarding his prescribed medications. Which client response indicates that more teaching is​ needed? ​A) "I need to call the health care provider if I begin to have severe muscle or joint​ pains, or if my other health care provider prescribes medications for my​ Crohn's disease​ (chronic inflammation of the digestive​ tract)." B) ​"If I develop an ear​ infection, which usually happens each​ winter, I need to be careful about taking the statin medication along with the antibiotic that is usually​ prescribed." C) ​"I have to take this medication for the next 12​ months, no matter​ what, even if I have to begin taking the immunosuppressants again. I will just do​ it." D) ​"The nurse said I need to take this medication in the evening because the liver produces more cholesterol during the evening and night​ hours."

C Rationale The nurse is responsible for evaluating the effectiveness of​ treatment, including recommended lifestyle changes and prescribed​ pharmacotherapy, for clients with hyperlipidemia. When evaluating​ effectiveness, the nurse must determine if the client understood the education provided. Education will cover how to appropriately​ self-administer medications; when medications should be taken​ (statins should be taken in the​ evening); related adverse​ effects; why the medication has been​ prescribed; contraindications or​ precautions; and when to notify the health care provider. Adhering to the prescribed medication regimen for hyperlipidemia is​ important; however, the client must recognize the importance of reporting contraindications or the presence of serious adverse effects to the health care​ provider, as some adverse effects or the presence of specific contraindications may necessitate discontinuation of the medication. The health care provider must be notified of any important changes in client situation and condition.

The nurse is preparing to administer an anticoagulant to a client who has been admitted with a deep venous thrombosis. Which assessment parameter should the nurse complete​ before, during, and after administration of the​ medication? A) Assess for medication allergies B) Assess for recent surgeries or trauma C) Monitor vital signs D) Monitor for adverse effects

C Rationale The nurse should monitor vital signs before administering the​ medication, during medication​ administration, and after the medication has been administered. Assessing for adverse effects should be completed during and after medication​ administration, not before. Assessments for medication allergies and recent surgeries or trauma should be completed prior to medication administration.

The nurse is reviewing a client​'s current laboratory values. Which laboratory test is indicative of congestive heart​ failure? A) Elevated troponin T B) Elevated creatine phosphokinase​ (CPK) C) Elevated​ B-type natriuretic peptide​ (BNP) D) Elevated cholesterol

C Rationale ​B-type natriuretic peptide​ (BNP) is a hormone secreted primarily in the​ ventricles; its secretion is increased when pressure in the heart is​ increased, and it aids in the diagnosis of heart failure.​ Cholesterol, creatine phosphokinase​ (CPK), and troponin tests do not aid in the diagnosis of heart failure. An increased cholesterol level increases the risk of acute myocardial infarction​ (AMI), and can indicate​ hypothyroidism, uncontrolled​ diabetes, or biliary cirrhosis. Creatine phosphokinase​ (CPK) is an enzyme found in the​ heart, skeletal​ muscles, and brain​ tissue; the level rises within 4dash-6 hours after an acute myocardial infarction. Troponins are biochemical markers present in heart and skeletal​ muscles, and are used in the diagnosis of acute myocardial infarction.

The nurse is caring for a client who has been placed on aminocaproic acid​ (Amicar). The nurse reviews the client​'s medical record and notes which condition as a contraindication to this​ medication? A) Congestive heart failure B) History of hypertension C) Urinary tract infection D) History of migraine headaches

C Rationale Urinary tract infection is a known contraindication to the administration of aminocaproic acid​ (Amicar). Hypertension, history of migraine​ headaches, and congestive heart failure are not known contraindications to the administration of this medication.

The nurse is assessing a client with alkalosis who is being treated with acid agents. What adverse effects might the nurse observe when treating this client with ammonium​ chloride? A) Decreased respiratory rate B) Yellow skin color C) Central nervous system depression D) Nausea and vomiting

C) Central nervous system depression

The nurse is assessing a child with mild dehydration secondary to diarrhea. In a nonacute​ situation, what is the preferred way to replace electrolytes whenever​ possible? A) IV fluids B) Blood product infusion C) Oral hydration D) IV plasma expanders

C) Oral hydration

The nurse is reviewing a client​'s electrolyte panel. Which statements about sodium are ​true? A) Normal sodium levels range from 145-165 ​mEq/L. B) To replace sodium in the​ body, drink more water. C) The kidneys regulate sodium levels. D) High sodium levels can cause weakness. E) Hypernatremia can cause anorexia.

C) The kidneys regulate sodium levels. D) High sodium levels can cause weakness.

What are common clinical manifestations of fluid volume​ deficit? Select all that apply. A) Headache B) Nystagmus C) Tachycardia D) Orthostatic hypotension E) Decreased capillary refill

C, D, E

The nurse admits a new client to the nursing unit with a baseline medical diagnosis of stable angina. Which nursing actions occur during the assessment phase of the nursing ​process? Select all that apply. A) Teach the client about the medication​ (sublingual nitroglycerin): how to safely​ self-administer, side​ effects, precautions, and when to call for emergency medical help​ (9-1-1). B) Give the client feedback as​ he/she self-administers the sublingual nitroglycerin medication. C) Determine if results of monitoring​ (for example, vital​ signs, electrocardiograms) and laboratory data​ (troponin and electrolyte​ levels, lipid​ studies, C-reactive protein​ tests) are in the client​'s medical chart​ (all tests ordered by the health care provider on​ admission). D) Complete a health history and gather specific information regarding the signs and symptoms that brought the client to the emergency department and nursing unit. E) Complete a thorough pain assessment.

C, D, E Rationale During the assessment phase of the nursing process​ (in relation to medication administration for a new client admitted with stable​ angina), the nurse has many​ responsibilities, including obtaining a complete medical and social​ history, obtaining the client​'s medication​ history, and verifying that baseline laboratory and diagnostic tests are completed and that the results are in the client​'s medical chart. Other responsibilities include monitoring baseline vital​ signs, with continued and frequent monitoring​ thereafter; completing a pain assessment​ (onset, location,​ intensity, duration,​ character, precipitation, and alleviating​ factors); determining drug​ allergies; and having a thorough understanding of all prescribed medications and monitoring for therapeutic effect and for any contraindications. Teaching clients about their medication​ (in this​ example, sublingual​ nitroglycerin), including how to safely​ self-administer, side​ effects, precautions, and when to call for emergency medical help​ (9-1-1), is part of the nurse​'s role in the implementation phase of the nursing process. Giving clients feedback as they​ self-administer medication is part of the planning phase of the nursing process.

Which clinical manifestations are associated with ​acidosis? Select all that apply. A) Hyperactive reflexes B) ​Slow, shallow breathing C) Cardiac arrest D) Kussmaul breathing E) Lethargy

C, D, E Clinical manifestations associated with acidosis include​ lethargy, confusion, CNS​ depression, coma,​ weakness, fatigue, cardiac​ dysrhythmias, cardiac​ arrest, and Kussmaul breathing​ (deep and rapid​ respirations). Slow, shallow breathing and hyperactive reflexes are adverse effects associated with alkalosis.

What instructions should you give to a client who is taking​ enteric-coated ​aspirin? Select all that apply. Take with 240 mL of water or milk. Stop medication 2 days before dental work. Avoid alcoholic beverages. Report pregnancy. Do not crush or chew the medication.

C, D, E If your client is told to take​ enteric-coated aspirin,​ he/she must not crush or chew the tablet or capsule. As with all types of​ aspirin, alcoholic beverages should be avoided and pregnancy should be reported to the health care provider.​ Enteric-coated aspirin should not be taken with milk because the medication will dissolve too quickly. Clients who are scheduled for dental work or surgery must stop taking aspirin 7dash-14 days prior to the surgery

The client asks the nurse to teach him about lifestyle changes that will help lower his blood lipid levels. Which strategies will give the client clear guidance for reducing ​hyperlipidemia? Select all that apply. A) Encourage the client to begin monitoring his tobacco use. B) Encourage the client to decrease consumption of plant lipids. C) Emphasize the importance of exercising most days of the week and maintaining a healthy body weight. D) Teach the client the differences between soluble and insoluble fiber so that he can choose foods that will increase his soluble fiber intake. E) Teach the client how to read nutrition labels to make better choices based on​ fat, cholesterol, and fiber content.

C, D, E Rationale Learning how to read and understand food labels can help a person make healthier food​ choices, as well as help to decrease the amount of fats and cholesterol in the diet. Learning the differences between soluble and insoluble fiber will help the client make better choices to increase the dietary intake of soluble​ fiber, which helps change lipid levels. Regular physical activity will help to lower LDL​ ("bad") cholesterol and raise HDL​ ("good") cholesterol. These benefits come even with moderate​ exercise, such as brisk walking. Carrying even just a few extra pounds contributes to high​ cholesterol; losing as little as 5 to 10 pounds can help reduce cholesterol levels. Consumption of plant lipids is also​ beneficial, as they cause the body to excrete cholesterol and lower LDL levels. Merely monitoring tobacco​ use, by​ itself, will not reduce lipid​ levels, although cholesterol and triglyceride levels are influenced by smoking. Smoking raises LDL​ ("bad") cholesterollong dash—the type that can clog arteries with hard plaque. A chronically high level of LDL cholesterol is a major risk factor for heart disease because it is the primary contributor to plaque buildup in the arteries.

The nurse is preparing to discharge a client who has recently begun anticoagulant therapy. Which teaching points should the nurse include in the discharge instructions for this ​client? Select all that apply. ​A) "Avoid swimming." B) ​"Report bruising or abnormal bleeding at the next​ appointment." ​C) "It is important to wear a medical alert​ bracelet." ​D) "Use an electric​ razor." ​E) "Report shortness of breath​ immediately."

C, D, E Rationale The client should be instructed to report shortness of breath immediately. The client should be instructed to wear some form of medical alert jewelry and be taught to use an electric razor and to avoid anything that can cause a cut or break in the skin. The client should be instructed to report bruising and abnormal bleeding immediately. Although the client should be taught to avoid contact​ sports, swimming is not a contact​ sport; it is permitted while on anticoagulant therapy.

The nurse is caring for a client who has been admitted to the hospital for the treatment of hemophilia A. Which treatment options does the nurse anticipate will be ordered for this ​client? Select all that apply. A) Factor vWF replacement B) Heparin therapy C) Fresh frozen plasma D) Factor IX replacement E) Factor VIII replacement

C, E Rationale Factor VIII replacement and fresh frozen plasma are treatment choices for this client. Factor IX replacement is indicated for hemophilia B. Factor vWF is indicated for von Willebrand disease. Heparin therapy is indicated for a client with hypercoagulation

Cardiac Output

CO = HR x SV Contractility = Force of hearts contractions Preload = amnt of blood in heart prior to contraction Afterload = amnt of pressure heart exerts to move blood

The nurse is caring for an adult client with a history of type 2 diabetes mellitus. In the​ past, the client has had multiple episodes of hyperglycemia.​ Currently, the client is receiving insulin via infusion​ pump, to be discontinued at bedtime. The​ client's most recent blood glucose​ level, at​ 4:00 p.m., was 57​ mg/dL. (The normal range is​ 60-100 mg/dL.) What action should the nurse​ take? Obtain advice from another nurse. Recheck the​ client's blood glucose in one hour. Call to advise the healthcare provider of the most recent blood glucose level and discuss discontinuing the insulin infusion. Follow the healthcare​ provider's orders and continue the insulin infusion until bedtime.

Call to advise the healthcare provider of the most recent blood glucose level and discuss discontinuing the insulin infusion.

Mrs. Simmons was admitted to the hospital for hyperglycemia. As you take a medication​ history, she reports that she started taking​ over-the-counter niacin supplements to reduce her cholesterol because it is cheaper than her prescription medication. As the​ nurse, you know that​ niacin: Lowers blood glucose levels Has no effect on lipid levels Is contraindicated for diabetic clients Can elevate blood glucose levels

Can elevate blood glucose levels

Kako Yamamoto recently visited her hometown in rural Japan. She returned to the United States 3 weeks ago with a mild upper respiratory infection.​ Today, she was admitted to the​ medical-surgical unit with a persistent​ cough, high​ fever, chills, and lung congestion of unknown origin. The admitting health care provider prescribed 500 mg of IV​ imipenem-cilastatin (Primaxin) while waiting for culture and sensitivity test results. Ms. Yamamoto asks why she​ wasn't given penicillin tablets to take at home. What knowledge is the correct basis for your​ response? Penicillins are only effective against​ gram-negative bacteria and would not be selected in this case. The​ 3-week duration of the symptoms indicates that the infection is too severe for a penicillin derivative. Carbapenems are used to treat infections from​ gram-positive, gram-negative,​ anaerobic, and aerobic​ micro-organisms. ​Imipenem-cilastatin (Primaxin), a​ cephalosporin, is the most effective drug for upper respiratory infections.

Carbapenems are used to treat infections from​ gram-positive, gram-negative,​ anaerobic, and aerobic​ micro-organisms.

Which types of nutrients are administered to provide energy for the body and to maintain normal glucose​ levels? Carbohydrates Proteins Vitamins Lipids

Carbohydrates

Identify the category of immunizing agent for each condition. ​Instructions: Use the dropdown menus in the left column to select the correct category of immunizing agent for each condition in the right column.

Category Condition Immunoglobulins Gammigard Inactive Bacterial Toxins Diphtheria Viral Vaccines Hepatitis A Inactive Bacterial Toxins Pneumococcus Immunoglobulins CytoGam Inactive Bacterial Toxins Pertussis Viral Vaccines Influenza Inactive Bacterial Toxins Tetanus Immunoglobulins ​Nabi-HB Viral Vaccines Measles Immunoglobulins HyperHep Viral Vaccines Rabies

Which are mechanical complications associated with parenteral ​nutrition? Select all that apply. Catheter malposition Infection Cardiac arrhythmias Fluid volume overload Hemothorax

Catheter malposition Cardiac arrhythmias Hemothorax

The nurse is caring for an older adult client who is receiving total parenteral nutrition​ (TPN). What should the nurse assess to minimize ​complications?

Catheter placement Vital signs . Weight and lab results Intake and output

Classify bacteria according to the structure of their cell​ wall, shape, or use of oxygen. ​Instructions: Use the dropdown menus in the left column to select the classification of bacteria for each description in the right column. Classification Description

Cellular shape classification Spherical​ (cocci) Cell wall classification ​Gram-negative bacteria Oxygen use classification Aerobic Cell wall classification ​Gram-positive bacteria Cellular shape classification Rod​ (bacilli) Oxygen use classification Anaerobic Cellular shape classification Spiral​ (spirilla)

Identify the class of​ anti-infectives based on the drug name. ​Instructions: Use the dropdown menus in the left column to select the class for each drug in the right column. Class Drug Name

Cephalosporins Cefotaxime​ (Claforan) Aminoglycosides Amikacin​ (Amikin) Cephalosporins Cefepime​ (Maxipime) Aminoglycosides Tobramycin​ (Nebcin) Carbepenum Doripenem​ (Doribax) Macrolides Azithromycin​ (Zithromax) Carbepenum ​Imipenem-cilastatin (Primaxin) Aminoglycosides Kanamycin​ (Kantrex) Carbepenum Ertapenem​ (Invanz) Aminoglycosides Gentamicin​ (Garamycin) Cephalosporins Cephalexin​ (Reflex) Macrolides Dirithromycin​ (Dynamic) Cephalosporins Cefadroxil​ (Duricef) Macrolides Erythromycin​ (E-Mycin) Cephalosporins Cefazolin​ (Ancef) Macrolides Clarithromycin​ (Jiaxin) Carbepenum Meropenem​ (Merrem)

Which nursing actions are taken to decrease the risk for infection at the parenteral administration ​site? Select all that apply. Change the IV site of a peripheral line every 48 hours. Leave the site open to the air. Clean the insertion site area with an antimicrobial solution. Use the TPN catheter only for TPN. Cover the insertion site with a dry gauze dressing.

Change the IV site of a peripheral line every 48 hours. Clean the insertion site area with an antimicrobial solution. Use the TPN catheter only for TPN.

The nurse is providing discharge teaching to a client prescribed steroids for​ long-term use. Which adverse effects should the nurse include in the ​discussion? Select all that apply. Changes in mental status Hypotension Porous bones​ (osteoporosis) Obesity Cataracts

Changes in mental status Porous bones​ (osteoporosis) Obesity Cataracts

The nurse is caring for a client who is receiving enteral therapy through a gastrostomy tube. Which interventions by the nurse will reduce the risk of​ aspiration? Select all that apply. Check for proper tube placement. Elevate the head of the bed 30degrees° during feeding. Reduce the rate of administration. Check for gastric residual volumes. Check for signs of respiratory​ distress, abnormal lung​ sounds, or frothy sputum.

Check for proper tube placement. Elevate the head of the bed 30degrees° during feeding. Check for gastric residual volumes. Check for signs of respiratory​ distress, abnormal lung​ sounds, or frothy sputum.

The nurse is preparing to administer insulin to a client with type 1 diabetes mellitus. What​ action(s) does the nurse recognize as important to do during​ administration? Select all that apply. Check the​ client's blood glucose level Shake insulin preparations that are cloudy Monitor the​ client's dietary intake Use a tuberculin syringe Rotate the site of administration

Check the​ client's blood glucose level Monitor the​ client's dietary intake Rotate the site of administration

The client tells the nurse that one health care provider told him that he has too much cholesterol in his blood. Another health care provider told him he has elevated lipids. The client asks the nurse to explain the meaning of these statements. Which will the nurse include in the ​explanation? Select all that apply. Cholesterol levels in the bloodstream are an important measure of heart health. Some types of cholesterol are considered​ "good" and some are considered​ "bad." Eating​ wheat, corn,​ oats, and olive oil will help reduce your cholesterol level. It is rare that clients are able to manage hyperlipidemia with only lifestyle changes and no medications. Cholesterol is fat that the body produces and needs to work properly. The terms cholesterol and lipids are often used interchangeably. There are two causes of​ hyperlipidemia: heredity and​ lifestyle, which includes diet and activity level.

Cholesterol levels in the bloodstream are an important measure of heart health. Some types of cholesterol are considered​ "good" and some are considered​ "bad." Eating​ wheat, corn,​ oats, and olive oil will help reduce your cholesterol level. Cholesterol is fat that the body produces and needs to work properly. The terms cholesterol and lipids are often used interchangeably. There are two causes of​ hyperlipidemia: heredity and​ lifestyle, which includes diet and activity level.

Which medications are used to treat seborrheic ​dermatitis? Select all that apply. Ciclopiroxolamine​ (Loprox) Ketoconazole​ (Nizoral) Adapalene​ (Differin) Fluconazole​ (Diflucan) Azelaic acid​ (Azelex)

Ciclopiroxolamine​ (Loprox) Ketoconazole​ (Nizoral) Fluconazole​ (Diflucan)

Which nonspecific body defenses are considered the first line against ​infection? Select all that apply. Cilia Phagocytes Lymph node Mucous membranes Skin

Cilia Phagocytes Mucous membranes

The nurse is reviewing the medication supplied by a pharmacy against the healthcare​ provider's order for a quinolone antibiotic. Which medication in this group could be given either IV or​ PO?

Ciprofloxacin​ (Cipro) Levofloxacin​ (Levaquin)

Classify integumentary system disorders.

Class Disorder Parasitic Infestations Pediculosis Bacterial Infections Boils Viral Infections Rubella Inflammatory Disorders Rosacea Viral Infections Rubeola Inflammatory Disorders Dermatitis Parasitic Infestations Scabies Inflammatory Disorders Psoriasis Bacterial Infections Impetigo

What client would the nurse anticipate to be prescribed fluconazole​ (Diflucan)? Client with acne and tinea capitis Asthma client with tinea pedis and ingrown toenail Client with a history of asthma with oral candidiasis Client with HIV with oral candidiasis

Client with HIV with oral candidiasis

The nurse is caring for an adult client who has type 2 diabetes mellitus. After reviewing the​ client's chart, which​ item(s) should concern the​ nurse? Select all that apply. Client's comorbidity Healthcare​ provider's diet order for client ​Client's glucose results Healthcare​ provider's choice of antidiabetic drug ​Client's medication to treat comorbidity

Client's comorbidity Healthcare​ provider's choice of antidiabetic drug ​Client's medication to treat comorbidity

Which​ high-risk clients are often given​ anti-infective agents to prevent ​infections? Select all that apply. Siblings of​ HIV-positive clients Clients exposed to tuberculosis Clients who have early signs of an upper respiratory infection Immunosuppressed clients Preoperative cardiovascular surgery clients

Clients exposed to tuberculosis Immunosuppressed clients Preoperative cardiovascular surgery clients

Before administering an antifungal​ agent, which nursing action should you​ take? Monitor intake and output Collect a specimen of the organism Monitor liver function studies Change linens on the beds

Collect a specimen of the organism

A client is seen in the clinic after receiving a vaccination. Which reported clinical manifestation causes the nurse the most​ concern? Loss of appetite Irritability Malaise Confusion

Confusion

Which skin disorders would the nurse classify as inflammatory ​conditions? Select all that apply. Contact dermatitis Impetigo Tissue injury Psoriasis Fever​ blisters, or cold sores

Contact dermatitis Tissue injury Psoriasis

Mr.​ Thompson, a​ 53-year-old African American client with chronic uncontrolled​ hypertension, was admitted to the hospital after a massive​ stroke, which has left him immobile and unable to complete any activities of daily living​ (ADLs) (e.g.​ eating, bathing,​ toileting, transferring) independently. Which of the following are critical components of the assessments the nurse will complete on Mr. ​Thompson? Select all that apply. Mental status and his ability to report pain or discomfort His ability to independently move from the bed to the chair or to the wheelchair Continence​ (fecal and​ urinary) Skin​ integrity, especially over bony prominences and common pressure areas Level of nutrition and intake of​ protein, carbohydrates,​ fluids, vitamins, and other​ nutrition-related issues

Continence​ (fecal and​ urinary) Skin​ integrity, especially over bony prominences and common pressure areas Level of nutrition and intake of​ protein, carbohydrates,​ fluids, vitamins, and other​ nutrition-related issues Mental status and his ability to report pain or discomfort

A client who is scheduled to go to Ghana for several weeks is prescribed chloroquine phosphate​ (Aralen) to prevent malaria. What would the nurse teach the client about this ​medication? Select all that apply. Changes in vision are expected effects of this medication. Continue to take this medication for 4 weeks after you return home. Take immediately before or after meals to minimize stomach upset. Do not drink lemon juice while taking this medication. Start this medication as prescribed 2 weeks before you leave.

Continue to take this medication for 4 weeks after you return home. Take immediately before or after meals to minimize stomach upset. Do not drink lemon juice while taking this medication. Start this medication as prescribed 2 weeks before you leave.

Which goal is the focus of pharmacotherapy for​ psoriasis? Controlled inflammation Prevented eruptions Decreased scarring Decreased pain

Controlled inflammation

Plant lipids cause the body to excrete cholesterol and lower LDL levels. Which of these foods are considered sources of plant ​lipids? Select all that apply. Corn Nuts Potatoes Olive oil Rice

Corn Nuts Olive oil Rice

Which diagnostic​ test(s) is essential for prescribing the correct antibiotic for a bacterial​ infection? CBC with differential Baseline vital signs including temperature Allergy assessment Culture and sensitivity

Culture and sensitivity

Enteral feedings delivered over an​ 8- to​ 16-hour period of time are​ called: Intermittent feedings Cyclic feedings Bolus feedings Continuous infusion feedings

Cyclic feedings

Mr. Hirschbaum asks you how he will know if he contracts malaria. What common​ symptom(s) do you describe in your​ response? Persistent​ fever, nausea, and vomiting Cyclic recurrence of​ chills, rigor, and fever Diuresis and weight loss Difficulty sleeping

Cyclic recurrence of​ chills, rigor, and fever

The nurse is caring for a client after a kidney transplant. Which drug does the nurse expect to administer to prevent rejection of the​ kidney? Cyclophosphamide​ (Cytoxan) Cyclosporine​ (Neoral, Sandimmune) Methotrexate sodium​ (Rheumatrex) Etanercept​ (Enbrel)

Cyclosporine​ (Neoral, Sandimmune) RATIONALE: cyclosporine is given to prevent kidney, liver, and heart transplant rejection

The nurse is preparing a presentation for a group of newly hired nurses to give an overview of immunostimulants and the immune response system. Which points would the nurse include in the ​presentation? Select all that apply. Cytokines are the chemicals that help facilitate the body​'s immune response. Immunomodulators are drugs that increase the immune response in the body. ​T-cell lymphocytes are responsible for creating​ antibodies, used in the development of​ long-term immunity against antigens. The immune response protects the body against invading organisms or agents. ​B-cell lymphocytes regulate the body​'s immune response by releasing cytokines.

Cytokines are the chemicals that help facilitate the body​'s immune response. Immunomodulators are drugs that increase the immune response in the body. The immune response protects the body against invading organisms or agents.

Sodium​ bicarbonate, 44 mEq times two IV bolus is ordered for Ms. Degrasso. As you administer the​ solution, for which common adverse effect should you​ monitor? A) Hypercapnia B) Generalized hives C) Diarrhea D) Vomiting

D Adverse reactions associated with the administration of sodium bicarbonate include​ confusion, irritability, decreased respiratory​ rate, and vomiting. Although hypercapnia may be associated with a severely depressed respiratory​ rate, it is not an adverse effect associated with the administration of sodium bicarbonate. Neither diarrhea nor hives is associated with sodium bicarbonate administration.

What do you need to consider when caring for a client who is prescribed a potassium channel​ blocker? A) Noting if there is a decrease in urinary output B) Warning​ him/her about possible weight gain C) Checking for an increase in serum potassium level D) Advising​ him/her to use sunscreen regularly

D Because of the risk of​ photosensitivity, a client taking a potassium channel blocker needs to use sunscreen regularly and avoid direct sunlight. Weight gain is not an adverse effect associated with potassium channel​ blockers, but nausea and vomiting are. A potassium channel blocker does not increase or decrease the amount of potassium in the​ serum, nor does it reduce urinary output.

Mark​ Rivera, a​ 28-year-old male, had knee replacement surgery due to sports injury. You are discharging him home on enoxaparin​ (Lovenox). Mr. Rivera asks​ you, "Are these shots safe to take at​ home?" What is the best​ reply? ​A) "Yes, Lovenox comes in a pill so it is easier to administer at​ home." B) ​"Yes, Lovenox has a shorter duration of​ action, so it is considered much​ safer." C) ​"Yes, just be sure to continue your daily Coumadin​ dose." D) ​"Yes, Lovenox requires fewer​ follow-up laboratory tests and there is less chance of​ overdose."

D Enoxaparin​ (Lovenox) is more stable than heparin and does not require weekly PTT tests. Lovenox has a​ longer, not​ shorter, duration of action than heparin. Lovenox should not be taken with any other​ anticoagulant, such as warfarin​ (Coumadin), due to increased risk of bleeding. Lovenox is given by SQ​ injection, not in pill form.

Your​ client, Mrs. Valya​ Nelson, has been taking celecoxib​ (Celebrex) for three years for rheumatoid arthritis. She recently​ self-medicated with 800 mg of ibuprofen​ (Advil) twice a day for pain. Mrs. Nelson reports being chronically tired. Her eyes and skin are jaundiced. What adverse effect do you​ suspect? Kidney disease High cholesterol Low potassium Liver dysfunction

D Jaundice is due to liver​ dysfunction, which is an adverse effect resulting from the​ client's use of celecoxib and ibuprofen. Kidney​ disease, high​ cholesterol, and low potassium do not lead to symptoms of jaundice.

Larry Collins has a family history of hypertension. Mr. Collins is​ pre-hypertensive with a blood pressure of​ 130/85 mmHg. What is your primary goal for this​ client? A) Asking client to look for symptoms of hypertension B) Assessing​ client's breath sounds C) Asking client to keep a blood pressure diary D) Educating client on lifestyle modification

D Left​ untreated, consistent high blood pressure can lead to serious health consequences or death. Your primary goal for​ pre-hypertensive clients is decreased blood pressure by means of lifestyle changes such as diet​ modifications, increased​ exercise, and stress reduction. A client with​ hypertension, not​ pre-hypertension, needs to keep a blood pressure diary. Assessing breath sounds is not necessary in this case scenario.​ Typically, only​ minimal, if​ any, symptoms occur initially with​ hypertension, so the client would not be able to look for symptoms.

What should be assessed prior to the administration of a medication for​ hyperlipidemia? A) Complete blood count B) Abdominal ultrasound C) Urinalysis D) Liver function tests

D Liver function tests must be assessed because many medications for hyperlipidemia can cause hepatotoxicity. Neither a urinalysis nor an ultrasound is indicated. Agents for hyperlipidemia do not directly affect the complete blood count.

Robert​ Stevens, a​ 46-year-old male​ client, presents to your emergency room with progressive chest pain diagnosed as a​ lateral-wall myocardial infarction. Mr. Stevens has a history of hypertension treated with a calcium channel blocker. He states that he consumes​ 3-4 glasses of whiskey every evening. What electrolyte imbalance would likely be treated in the presence of a myocardial​ infarction? A) Hypernatremia B) Hypokalemia C) Hyponatremia D) Hypomagnesemia

D Magnesium is an active cardiovascular electrolyte.​ Thus, symptoms of hypomagnesemia may include cardiac complications such as dysrhythmias. Clients with a myocardial infarction are often treated for hypomagnesemia to improve their cardiac conduction. Chronic alcoholics also often develop hypomagnesemia. Although hypokalemia may be a concern in a client with a​ dysrhythmia, there is no indication that Mr. Stevens is at risk for this imbalance. There are no predisposing factors that would contribute to hyponatremia or hypernatremia in this client.

The nurse is preparing to administer heparin to a client who is hospitalized for a deep venous thrombosis. Which condition would cause the nurse to question this medication​ order? A) Nausea and vomiting B) Minor thrombocytopenia C) Shingles D) Severe hypertension

D Rationale A history of severe hypertension is a contraindication for the administration of heparin. Neither shingles nor minor thrombocytopenia is a contraindication for the administration of heparin. Nausea and vomiting are known adverse effects of​ heparin, but are not​ contraindications; therefore, the nurse would not question this order.

The nurse is preparing to discharge a client who has been started on an antidysrhythmic medication. The nurse educates the client about the importance of weighing himself​ daily, at the same time each day. Which statement by the client indicates understanding of the​ teaching? ​"I should report a weight loss of 2 pounds in a​ 14-day period." ​"I should report a weight gain of 5 pounds in a​ 14-day period." ​"There is no need to monitor my​ weight." ​"I should report a weight gain of 2 pounds in a​ 24-hour period."

D Rationale A weight gain of 2 pounds in a​ 24-hour period would require notification of the health care provider. Neither a weight gain of 5 pounds over a​ 14-day period nor a weight loss of 2 pounds in 14 days would require notification of the health care provider. Clients on antidysrhythmic medications are educated to monitor their weight daily.

The nurse has taught the client about the major differences between angina pectoris and myocardial infarction​ (MI). Which client statement indicates that further teaching is​ needed? ​A) "When a person has an​ MI, part of their heart muscle actually​ dies." B) ​"I get angina because the oxygen demands of my heart are not being​ met." C) ​"You just taught me that rest and relaxation will decrease my pain by decreasing my​ heart's demand for​ oxygen." D) ​"Angina and MI are two different names for the same​ condition."

D Rationale Angina pectoris and myocardial infarction are not different names for the same condition. If a client made this statement after nursing education has been​ provided, further teaching would be warranted. Angina pectoris is chest pain caused by a deficiency in oxygen supply to meet the metabolic demands of the heart muscle and results in myocardial ischemia. Angina occurs due to an increased oxygen demand by the heart muscle​ (myocardial oxygen​ demand) and is often associated with physical exertion or emotional excitement. Angina pain is usually relieved by rest and​ relaxation, which decrease the myocardial oxygen demand. Myocardial infarction​ (MI), commonly referred to as a heart​ attack, refers to myocardial cellular necrosis​ (death of​ heart-muscle cells), and is a​ life-threatening event requiring immediate medical attention.

The nurse is caring for a client with newly diagnosed thrombocytopenia. Which medication from the client​'s medication administration record has caused this​ condition? A) Herbal supplements B) Oral contraceptives C) Multivitamins D) Anticoagulants

D Rationale Anticoagulants place a client at risk for developing thrombocytopenia. Oral​ contraceptives, multivitamins, and herbal supplements do not place a client at risk for developing thrombocytopenia.

The nurse is reviewing the action of angiotensin II receptor blockers​ (ARBs) so that she can explain the action to the adult client with hypertension who has an appointment today. What action of ARBs lowers blood pressure in the adult​ client? A) Slow the movement of calcium into the heart muscle cells B) Pull water from the extracellular space C) Cause the veins to dilate D) Block angiotensin II in the smooth muscle of the artery wall

D Rationale By blocking angiotensin II in the smooth muscle of the artery​ wall, ARBs decrease peripheral resistance in the arteries and thus lower blood pressure.​ Alpha1-adrenergic blockers, not​ ARBs, cause the veins to dilate. Diuretics pull water from the extracellular spaces and encourage​ excretion, thereby lowering blood volume and blood pressure. Slowing the movement of calcium into the heart muscle cells is an action of calcium channel blockers​ (CCBs), not ARBs

The nurse is caring for a client after a kidney transplant. Which drug does the nurse expect to administer to prevent rejection of the​ kidney? A) Methotrexate sodium​ (Rheumatrex) B) Etanercept​ (Enbrel) C) Cyclophosphamide​ (Cytoxan) D) Cyclosporine​ (Neoral, Sandimmune)

D Rationale Cyclosporine​ (Neoral, Sandimmune) is given to prevent​ kidney, liver, and heart transplant rejection. Cyclophosphamide​ (Cytoxan) is a cytotoxic agent used for​ cancer, and would not help with preventing kidney transplant rejection. Etanercept​ (Enbrel) is given for arthritic​ conditions, and would not help in preventing kidney transplant rejection. Methotrexate sodium​ (Rheumatrex) is given for​ cancer, rheumatoid​ arthritis, and​ psoriasis; it would not be beneficial following a kidney transplant.

The nurse is caring for a client who is being treated for heart failure with digoxin​ (Lanoxin). During​ therapy, the nurse should monitor which laboratory​ test? A) Urine ketones B) ​B-type natriuretic peptide​ (BNP) level C) Troponin T level D) Potassium level

D Rationale During digoxin​ therapy, the nurse should monitor the client​'s potassium and magnesium​ levels, because hypomagnesemia and hypokalemia can place the client at risk for digoxin toxicity. Ketones are present only in clients with diabetic​ ketoacidosis; troponins, which are biochemical markers present in heart and skeletal​ muscles, are used in the diagnosis of acute myocardial​ infarction; and​ B-type natriuretic peptide​ (BNP) is a hormone​ secreted, primarily in the​ ventricles, when pressure in the heart is​ increased, and aids in the diagnosis of heart failure. Digoxin​ (Lanoxin) does not affect urine​ ketones, troponin​ T, or BNP levels.

The nurse is caring for a client who sustained a liver laceration in a motor vehicle accident 6 hours ago. When should the nurse anticipate that fibrinolysis will​ begin? A) Within the next 8 to 12 hours B) After 36 to 60 hours C) Within the next 2 to 6 hours D) Within the next 18 to 36 hours

D Rationale Fibrinolysis occurs within 24 to 48 hours after an injury. Because this injury occurred 6 hours​ ago, the nurse can expect fibrinolysis to begin within the next 18 to 36 hours.

The nurse is caring for a client who is receiving furosemide​ (Lasix) for heart failure and is experiencing orthostatic hypotension. How should the nurse​ intervene? A) Administer a vasodilator as ordered. B) Administer IV fluids as ordered. C) Insert a Foley catheter as ordered. D) Instruct the client to sit on the side of the bed before standing.

D Rationale Furosemide​ (Lasix) is a loop diuretic that lowers blood pressure and can cause orthostatic hypotension. The nurse should instruct the client to change positions​ slowly, by sitting for several minutes before standing to minimize the adverse effects of diuretic therapy. Administering IV fluids is inappropriate when a client is receiving diuretic​ therapy, as it may lead to fluid imbalance. Inserting a Foley catheter would not minimize the effects of orthostatic hypotension. Administering a vasodilator would exacerbate the orthostatic hypotension and further lower the client​'s blood pressure.

The emergency room nurse is caring for a client who is experiencing dependent​ edema, hepatomegaly,​ crackles, and dyspnea on exertion. The nurse suspects which​ condition? A) Tension pneumothorax B) Cardiac tamponade C) Pulmonary embolism D) Heart failure

D Rationale In heart​ failure, the heart​'s ability to pump is​ decreased, causing decreased cardiac output. Fluid builds up and leads to dependent​ edema, hepatomegaly,​ crackles, dyspnea on​ exertion, and jugular vein distention. Manifestations of pulmonary embolism include chest​ pain, anxiety,​ tachycardia, abrupt onset of​ dyspnea, and cyanosis. A client with cardiac tamponade exhibits muffled heart​ sounds, dyspnea,​ tachycardia, and narrowed pulse pressure. A client with tension pneumothorax exhibits​ hypotension, severe​ dyspnea, shock,​ tachycardia, and deviated trachea with absent breath sounds on the affected side.

The nurse is assessing a client who is receiving lisinopril​ (Prinivil) for heart failure. Which finding would indicate to the nurse that therapy is​ ineffective? A) Hypotension B) Dizziness C) Anorexia D) Peripheral edema

D Rationale Lisinopril​ (Prinivil) is an​ angiotensin-converting enzyme​ (ACE) inhibitor that lowers blood pressure by enabling the heart to be more efficient by pumping more blood. This causes a decrease in heart failure symptoms such as shortness of​ breath, fatigue, and peripheral edema. If the nurse observes peripheral edema during lisinopril​ (Prinivil) therapy, it is a sign of fluid volume excess and heart failure that has worsened.​ Dizziness, hypotension, and anorexia are potential adverse reactions to lisinopril​ (Prinivil), but they do not indicate that therapy is ineffective.

The nurse is preparing to administer an antidysrhythmic medication to a client. Which serum electrolyte value should the nurse assess prior to administering the​ medication? A) Bicarbonate B) Chloride C) Calcium D) Potassium

D Rationale Potassium is the electrolyte that must be monitored before an antidysrhythmic medication is administered.​ Calcium, chloride, and bicarbonate need not be monitored prior to administration of an antidysrhythmic medication.

The nurse is teaching a client with an ankle sprain about the drug ibuprofen​ (Advil). What should be included in the ​teaching?

Report any unusual bruising to the health care provider. Avoid the use of alcohol while taking ibuprofen. Do not take aspirin and ibuprofen together. Use sunscreen and protective clothing when outdoors.

The nurse is planning care for an older adult client with​ right-sided heart failure. Which set of symptoms would the nurse expect this client to​ exhibit? A) Cyanosis and inspiratory rales B) Tachypnea and orthopnea C) Dry cough and inability to lie flat D) Peripheral edema and decrease in appetite

D Rationale Symptoms of​ right-sided heart failure include edema in the legs and​ feet, decrease in​ appetite, nausea, abdominal​ distention, jugular vein distention​ (JVD), and right upper quadrant pain from blood backing up in the liver. Symptoms of​ left-sided heart failure include​ cyanosis, dry​ cough, orthopnea,​ tachypnea, inspiratory crackles and​ rales, and worsening symptoms at night.

The nurse is preparing to administer the scheduled dose of diltiazem​ (Cardizem) to a client who will be discharged from the hospital later in the shift. The​ client's pulse is 58. What should the nurse instruct the client to do if he encounters this situation at​ home? Administer the medication as usual and contact the health care provider. Administer the medication as usual. Hold the medication. Hold the medication and notify the health care provider.

D Rationale The client should be taught to hold the dose and notify the health care provider if his pulse is less than 60.

The nurse is preparing to administer adenosine​ (Adenocard) for the treatment of paroxysmal supraventricular tachycardia. What is the appropriate time duration for administration by IV​ push? A) 9 to 10 seconds B) 14 to 15 seconds C) 4 to 5 seconds D) 1 to 2 seconds

D Rationale The nurse would correctly administer adenosine​ (Adenocard) over 1 to 2 seconds. The other intervalslong dash—4 to 5​ seconds, 9 to 10​ seconds, and 14 to 15 secondslong dash—are too​ long; the nurse would not correctly administer the medication in these time frames.

A client with heart failure is prescribed lisinopril​ (Prinivil). What is the mechanism of action for​ angiotensin-converting enzyme​ (ACE) inhibitors? A) Constriction of veins B) Increased cardiac workload C) Decreased cardiac output D) Increased cardiac output

D Rationale ​Angiotensin-converting enzyme​ (ACE) inhibitors increase cardiac output by lowering blood pressure and decreasing fluid​ volume; they also dilate​ veins, leading to an improvement in edema and symptoms of pulmonary congestion.

What is the most important adverse reaction to monitor for when administering intravenous​ potassium? A) Pain at the insertion site B) Confusion C) Nausea D) Cardiac dysrhythmia

D The most important nursing action during the administration of potassium chloride is to monitor the client for cardiac dysrhythmias. Although pain at the insertion site and GI symptoms​ (such as​ diarrhea, nausea, and​ vomiting) can occur with a potassium​ infusion, the development of cardiac dysrhythmia is the most critical of these potential reactions. Confusion may be seen during the administration of either magnesium sulfate or high concentrations of sodium chloride.

What is the primary action of a​ statin? A) Prevent cholesterol from being absorbed B) Decrease VLDL levels C) Bind to bile​ acids, increasing the excretion of cholesterol in the stool D) Inhibit​ HMG-CoA reductase, decreasing the​ liver's production of cholesterol

D The primary mechanism of action of a statin is to inhibit​ HMG-CoA reductase, thereby decreasing the​ liver's production of cholesterol. The primary effect of nicotinic acid​ (niacin) is to decrease VLDL levels. Increasing the excretion of cholesterol in the stool is the action of bile acid resins. Cholesterol absorption inhibitors prevent cholesterol from being absorbed.

Which description relates to the action of thrombin​ inhibitors? A) Inhibit the synthesis of clotting factors in the liver B) Act on circulating thrombin but not thrombin attached to a clot C) Enhance the action of antithrombin III D) Bind to​ thrombin, preventing the formation of fibrin clots

D Thrombin inhibitors bind to​ thrombin, preventing fibrin clot formation. Thrombin inhibitors act on both circulating thrombin and thrombin already attached to a clot. Warfarin​ (Coumadin) inhibits the synthesis of clotting factors in the liver. Heparin enhances the action of antithrombin III. These two drugs are not classified as thrombin inhibitors.

The nurse is reviewing a hospital memo regarding the proper storage of monoclonal antibodies. Which drugs should be stored under the conditions prescribed in the ​memo? Select all that apply. Daclizumab​ (Zenapax) ​Muromonab-CD3 (Orthoclone​ OKT3) Mycophenolate mofetil​ (CellCept) Thalidomide​ (Thalomid) Basiliximab​ (Simulect)

Daclizumab​ (Zenapax) ​Muromonab-CD3 (Orthoclone​ OKT3) Basiliximab​ (Simulect) Rationale Basiliximab​ (Simulect) is a monoclonal antibody and a preventive for kidney transplant rejection. Daclizumab​ (Zenapax) is classified as an MAB and has immunosuppressant properties. Mycophenolate mofetil​ (CellCept) is not classified as an​ MAB, but it can be given to prevent​ heart, kidney, and liver transplant rejection. Thalidomide​ (Thalomid) is not a​ MAB; it is used to treat leprosy and refractory​ Crohn's disease.​ Muromonab-CD3 (Orthoclone​ OKT3) is an MAB that is given to prevent kidney transplant rejection and is administered by the IV route.

The nurse is assessing an older adult client with a history of progressive dysphagia who is receiving enteral nutrition through a nasogastric tube. The nurse should observe for which metabolic​ complication? Dehydration Subcutaneous emphysema Pneumothorax Thromboembolism

Dehydration

The nurse is assessing an older adult client with a history of progressive dysphagia who is receiving enteral nutrition through a nasogastric tube. The nurse should observe for which metabolic​ complication? Pneumothorax Thromboembolism Dehydration Subcutaneous emphysema

Dehydration

The nurse is caring for an older adult client who is diagnosed with inflammatory bowel​ disease, and receiving enteral nutrition therapy to rest the bowel. The nurse should watch for which complications of enteral ​nutrition? Select all that apply. Thromboembolism Subcutaneous emphysema Dehydration Nausea Aspiration

Dehydration Nausea Aspiration

The nurse is going over the side effects of penicillins with the parents of an infant who has a bacterial infection. Which side effects will the nurse include when educating the parents on possible allergic ​reactions? Select all that apply. Abnormal crying Fever Urticaria Sensitivity to cephalosporins Delayed skin reaction

Delayed skin reaction Fever Urticaria

The nurse is caring for a teenage client who has been diagnosed with anorexia nervosa. The nurse should be concerned with which potential complications of ​malnutrition? Select all that apply. Delayed wound healing Death Insufficient immunity Excessive muscle bulk Muscle wasting

Delayed wound healing Death Insufficient immunity Muscle wasting

The nurse is caring for a teenage client who has been diagnosed with anorexia nervosa. The nurse should be concerned with which potential complications of ​malnutrition?

Delayed wound healing Insufficient immunity Death Muscle wasting

The nurse is evaluating why a wound is not healing. Which​ medication, taken by the​ client, can delay wound​ healing? Digoxin​ (Lanoxin) Esomeprazole​ (Nexium) Nebivolol​ (Bystolic) Dexamethasone​ (Decadron)

Dexamethasone​ (Decadron)

A young adult with a history of type 1 diabetes presents to the emergency room with complaints of abdominal​ pain, nausea, and vomiting. A few days​ ago, he reduced his insulin dose because flu symptoms caused a decrease in appetite. The​ nurse's assessment reveals fruity breath​ odor, dry mucous​ membranes, and poor skin turgor. What condition does the nurse​ suspect? Diabetic ketoacidosis​ (DKA) Hyperosmolar hyperglycemic state​ (HHS) Hypoglycemia Viral illness

Diabetic ketoacidosis​ (DKA)

The nurse is caring for an African exchange student who is diagnosed with malaria. What repeatedly occurring manifestations would the nurse observe in this ​client? Select all that apply. Diaphoresis Weight loss Fever Aches Chills and rigor

Diaphoresis Fever Chills and rigor

Mr. Warner is a​ 72-year-old male. The nurse is educating Mr.​ Warner, his​ wife, and his daughter about normal changes that go along with aging which place an elderly person at risk for impaired skin integrity. Which of the following expected changes that go along with aging place Mr. Warner at higher risk for skin​ breakdown? Diminished pain perception Increased oil production by the sebaceous glands Increased skin elasticity Increased lean body mass

Diminished pain perception

Which adverse effects are associated with​ sulfonylureas, a group of oral hypoglycemic ​agents? Select all that apply. Dizziness Hypoglycemia Drowsiness Nausea Metallic taste

Dizziness Hypoglycemia Drowsiness Nausea

Matthew Logan is a​ 16-year-old client who is undergoing treatment with isotretinoin​ (Claravis) for severe postulated acne. Which instruction should you include in your client education for​ Matthew? Do not discontinue the medication unless advised. Wash linens in hot water and dry them at high settings. Scrub the face three times per day. Do not apply any lotions or creams.

Do not discontinue the medication unless advised.

The nurse is observing a student who is administering medications. Which action requires intervention by the​ nurse? Instructing a client that the student will be giving methotrexate​ (Rheumatrex) intramuscularly​ (IM) as ordered Drawing up etanercept​ (Enbrel) to administer intramuscularly​ (IM) Preparing to administer anakinra​ (Kineret) subcutaneously​ (SC) Preparing to administer azathioprine​ (Azasan) intravenously​ (IV)

Drawing up etanercept​ (Enbrel) to administer intramuscularly​ (IM) Etanercept​ (Enbrel) is given​ subcutaneously; the nurse would need to intervene to prevent an error in administration of the drug if the student drew it up for IM injection. The student is acting correctly in administering anakinra​ (Kineret) SC, giving azathioprine​ (Azasan) IV or​ PO, and administering methotrexate​ (Rheumatrex) IM; the nurse need not to intervene in these instances.

Which complications can occur during the administration of enteral ​nutrition? Select all that apply. Drug and food interactions Clogged feeding tubes Muscle wasting Nausea and vomiting Aspiration

Drug and food interactions Clogged feeding tubes Nausea and vomiting Aspiration

The nurse is teaching the client about the most common adverse effects associated with nicotinic acid​ (Niacor, Niaspan) therapy. Which adverse effects should be included in the ​teaching? Select all that apply. Facial flushing and profuse sweating Nausea and vomiting ​Hard, infrequent stools Loose stools Gallstones

Facial flushing and profuse sweating Nausea and vomiting Loose stools

Identify the drugs used to treat different types of dermatologic infections. ​Instructions: Use the dropdown menus in the left column to select the type of infection for each drug in the right column. Infection Drug Name

Drugs for Fungal Infections Fluconazole​ (Diflucan) Drugs for Parasitic Infections Lindane​ (Kwell) Drugs for Fungal Infections Miconazole nitrate​ (Micatin) Drugs for Bacterial Infections Erythromycin​ (Erygel) Drugs for Fungal Infections Clotrimazole​ (Gyne-Lotrimin) Drugs for Parasitic Infections Pyrethrins​ (Pyrinyl) Drugs for Bacterial Infections Metronidazole​ (MetroGel) Drugs for Parasitic Infections Crotamiton​ (Eurax) Drugs for Bacterial Infections Mupirocin​ (Bactroban)

What complications are associated with fluid volume ​replacement? Select all that apply. Fever and chills Dyspnea Nausea Cardiovascular stress Fluid in the lungs

Dyspnea Cardiovascular stress Fluid in the lungs

In what ways can microorganisms enter the body. Select all that apply. Swimming in the ocean Eating spinach containing Escherichia coli Walking barefoot in the sand Exposure to droplets through a coworker​'s cough Through a laceration of the skin

Eating spinach containing Escherichia coli Exposure to droplets through a coworker​'s cough Through a laceration of the skin

Which condition is a chronic form of dermatitis with a familial​ tendency? Alopecia Eczema Measles Tinea cruris

Eczema

Total parenteral nutrition​ (TPN) is ordered for an adult client with a severe gastrointestinal disorder. The nurse expects that the solution will contain which ​nutrients?

Electrolytes Lipids Amino acids Trace minerals

What is a critical nursing action to prevent aspiration when administering enteral​ feedings? Dilute the feeding. Elevate the head of the bed 30 degrees. Give metoclopramide​ (Reglan) to delay gastric motility. Assess for refeeding syndrome.

Elevate the head of the bed 30 degrees.

The nurse is caring for a client who is receiving enteral therapy through a gastrostomy tube. Which interventions by the nurse will reduce the risk of​ aspiration

Elevate the head of the bed 30degrees° during feeding. Check for gastric residual volumes. . Check for proper tube placement. Check for signs of respiratory​ distress, abnormal lung​ sounds, or frothy sputum.

An older adult client undergoing cancer chemotherapy is receiving total parenteral nutrition​ (TPN) through a central line. The nurse knows that the client is at risk for which mechanical ​complications? Select all that apply. Endocarditis Brachial plexus injury Bone demineralization Hemothorax Fluid volume overload

Endocarditis Brachial plexus injury Hemothorax

A pediatric client diagnosed with a helminthic infection is experiencing perianal itching during the night. The nurse explains to the​ client's mother that this manifestation is associated with which type of​ infection? Ascariasis Hookworms Enterobiasis Tapeworms

Enterobiasis

Before administering regular insulin​ (Humulin R), you assess your client. Which finding is the most concerning to​ you? Episodes of vomiting Fasting blood glucose of 120​ mg/dL Blood pressure​ 130/80 Muscle cramps

Episodes of vomiting

Which immunosuppressants are used to treat rheumatoid ​arthritis? Select all that apply. Etanercept​ (Enbrel) Anakinra​ (Kineret) Thalidomide​ (Thalomid) Methotrexate sodium​ (Trexall) Interferon​ alfacon-1 (Infergen)

Etanercept​ (Enbrel) Anakinra​ (Kineret) Methotrexate sodium​ (Trexall)

Rosa​ Dillon, a​ 10-year-old client, is in the hospital following a car accident. She has sustained two broken​ ribs, along with cuts and bruises. As you clean and dress her​ wounds, Rosa worries because they are so swollen. As her​ nurse, what should you do to address​ Rosa's concerns? Assess for relief of symptoms. Explain how inflammation helps her body heal. Explain to her how the lymphatic system works. Consult with her admitting nurse.

Explain how inflammation helps her body heal.

One of your clients has just been diagnosed with type 1 diabetes mellitus. Which characteristics support this ​diagnosis? Select all that apply. Fasting blood sugar of 80​ mg/dL Family history of type 1 diabetes mellitus Treated for mononucleosis 2 months ago Eleven years of age Five feet tall and weighs 100 pounds

Family history of type 1 diabetes mellitus Treated for mononucleosis 2 months ago Eleven years of age

A client is prescribed ivermectin​ (Stromectol) for a helminthic infection. Of which adverse effects would the nurse instruct the client to be ​aware? Select all that apply. Headache Fever Pruritus Abnormal liver function tests Rash

Fever Pruritus Rash

Adverse reactions of Blood Transfusions

Fever Chills Back Pain Dizziness Urticaria Headache Dyspnea Nausea/Vomiting Hypotension Tachycardia

The nurse is caring for a client diagnosed with African trypanosomiasis. What manifestations of this infection would the nurse assess in this ​client? Select all that apply. Fever and chills Aches Extreme malaise Irregular heart rhythm Weight loss

Fever and chills Aches Extreme malaise Weight loss

If a client has a​ fever, why may immunostimulant drug therapy have to be ​delayed? Select all that apply. Fever indicates the need for intravenous antibiotics to be added to the prescribed therapy. Fever puts the client more at risk for drug adverse effects. Fever makes it harder to distinguish adverse effects of drug therapy from effects of the infection. Fever indicates the need to choose a different medication regimen. Fever increases the needed duration of immunostimulant drug therapy.

Fever puts the client more at risk for drug adverse effects. Fever makes it harder to distinguish adverse effects of drug therapy from effects of the infection.

The​ 57-year-old client has been prescribed the​ "drug of​ choice" for treating her severe hypertriglyceridemia. A drug from which class has been​ selected? Fibric acid agents Nicotinic acid​ (Niacor, Niaspan), also called niacin Cholesterol absorption inhibitors Bile acid resins

Fibric acid agents

Which symptoms are clinical manifestations of urinary tract ​infections? Select all that apply. Headache Flank pain Difficulty voiding Increased urgency to void Leg pain

Flank pain Difficulty voiding Increased urgency to void

John Driscoll travels extensively for his work and was recently diagnosed with an acquired roundworm infection. The health care provider prescribes mebendazole. As Mr.​ Driscoll's nurse, what information should you include in your​ teaching? Frequent hand washing is key to preventing the spread of infection. Common adverse effects of mebendazole are pruritus and rash. A roundworm infection is rarely transmitted to other family members. His infection is likely related to eating raw food such as​ beef, pork, or fish.

Frequent hand washing is key to preventing the spread of infection.

The nurse is reviewing the description of​ anti-infective drugs before giving a lecture to new nursing students. Which pathogens cause conditions that can be treated using ​anti-invectives? Select all that apply. Allergies Fungi Viruses Parasites Bacteria

Fungi Viruses Parasites Bacteria

The nurse is administering pharmacotherapy to a client with severe hyperkalemia. Which agent helps the body to eliminate potassium through​ urination? Insulin with dextrose Calcium gluconate Polystyrene sulfonate​ (Kayexalate) Furosemide​ (Lasix)

Furosemide​ (Lasix)

The nurse is caring for an adult client who contracted lepromatous leprosy while on a mission trip. The client has a compromised immune system and will require pharmacotherapy. What side effects from the pharmacotherapy can be​ expected? Loss of digits Paresthesia Macular rash Further​ immune-system suppression

Further​ immune-system suppression

What is the most common physical issue related to Nonsteroidal Antiinflammatory Drug​ (NSAID) therapy that the nurse should assess for before and during​ administration? Migraine Gastrointestinal​ (GI) bleeding Autoimmune disorder Dysrhythmia

Gastrointestinal​ (GI) bleeding

A client receives 15 units of isophane insulin​ (Humulin N) at​ 8:00 a.m. At​ 3:00 p.m., the nurse observes that the client is​ pale, diaphoretic,​ anxious, and restless. What action should the nurse take​ first? Check the​ client's vital signs. Call the healthcare provider. Give the client 8 oz of skim milk. Check the​ client's liver function tests.

Give the client 8 oz of skim milk.

In the emergency​ department, the nurse is caring for an unconscious adult client with a blood glucose level of 37​ mg/dL. (Normal blood glucose is​ 60-100 mg/dL.) What medication does the nurse expect to see​ ordered? Glipizide​ (Glucotrol) Glyburide​ (DiaBeta) Glimepiride​ (Amaryl) Glucagon​ (GlucaGen)

Glucagon​ (GlucaGen)

Which statement is true regarding the administration of glucagon​ (GlucaGen)? Glucagon​ (GlucaGen) comes in oral and injectable forms. Glucagon​ (GlucaGen) should only be given if the client is unable to swallow. Liver function tests should be performed periodically when using glucagon​ (GlucaGen). After taking a dose of glucagon​ (GlucaGen), the client should not eat or drink until blood glucose levels are checked.

Glucagon​ (GlucaGen) should only be given if the client is unable to swallow.

If your client is diagnosed with scalp ringworm​ (tinea capitis), what medication would you expect to be included in the treatment​ regimen? Permethrin​ (Acticin) Acyclovir​ (Zovirax) Griseofulvin​ (Fulvicin) Gentamicin sulfate

Griseofulvin​ (Fulvicin)

Which categories of clients are often prescribed treatment for tuberculosis because they are considered high ​risk? Select all that apply. Military personnel ​School-age children HIV clients Incarcerated adults Clients receiving chemotherapy

HIV clients Incarcerated adults Clients receiving chemotherapy

During discharge​ teaching, the nurse explains to a client diagnosed with type 2 diabetes that lab testing will be done to check his overall management of the disease. Which test does the nurse say to expect in about​ 8-12 weeks? Random blood glucose Fasting plasma glucose​ (FPG) test HbA1C​ (hemoglobin A1C) Oral glucose tolerance test​ (OGTT)

HbA1C​ (hemoglobin A1C)

A client is complaining of stiffness and arthritic pain in the hands. Which application should the nurse expect to be included in the treatment​ plan? Ice glove Cold pack Heat pack ​Cool, moist compresses

Heat pack

The nurse is reviewing the record of a client who is at risk for skin breakdown. Which lab data would be of particular concern to the nurse who is concerned with skin​ integrity? Leukocyte level of​ 6,000 Potassium level of 4.0​ mEq/L Hemoglobin level of 10.2​ g/dL Albumin level of 4.0​ g/dL

Hemoglobin level of 10.2​ g/dL

Which statement about the​ nurse's role in the treatment of female alopecia is​ accurate? Treatment with PO finasteride​ (Propecia, Proscar) is successful only for clients under 50 years of age. Hormone levels should be assessed. Minoxidil​ (Loniten) PO is prescribed to decrease hair loss. Once the destruction of hair follicles is​ decreased, treatment may be stopped.

Hormone levels should be assessed.

In the emergency​ room, glucagon​ (GlucaGen) is administered to an unresponsive client with severe hypoglycemia. What potential adverse effect is most critical for the nurse to​ monitor? Hyperglycemia Nausea and vomiting Hyperkalemia Lactic acidosis

Hyperglycemia

The nurse is preparing to administer tolazamide​ (Tolinase) to a client with type 2​ diabetes, as prescribed by the healthcare provider. For what adverse effect does the nurse need to​ monitor? Hypokalemia Metallic taste Hypoglycemia Lactic acidosis

Hypoglycemia

identify the complication based on the sign or symptom. ​Instructions: Use the dropdown menus in the left column to select the correct complication for each sign or symptom in the right column. Condition Sign or Symptom

Hypoglycemia Sudden moodiness Hypoglycemia Hunger Hyperglycemia Increased thirst Hypoglycemia Sweating Hypoglycemia Pale skin color Hyperglycemia Elevated sugar levels in urine Hypoglycemia Seizure Hyperglycemia Frequent urination Hypoglycemia Confusion Hyperglycemia Elevated blood glucose level Hypoglycemia Dizziness Hypoglycemia Shakiness

Samantha King is receiving an intravenous infusion of amphotericin B​ (Amphocin) for a systemic fungal infection. For what serious adverse​ effect(s) should you​ monitor? Hypokalemia and shock Nausea and vomiting Muscle pain Topical rash at infusion site

Hypokalemia and shock

The pediatric nurse is assessing a child in the clinic who has tested positive for an influenza virus. Assessment vital signs reveal a temperature of 102.4​°F ​(39.4​°​C). Which medication does the nurse anticipate will be ordered for the​ client? Select all that apply.

Ibuprofen (advil) acetaminophen (tylenol)

The nurse is reviewing the orders of a client admitted with a diagnosis of gastrointestinal​ (GI) bleeding. Which order would the nurse​ question?

Ibuprofen​ (Advil) 200​ mg, 2 tablets PO every 6 hours as needed for pain

Eva Gonzales is a​ 35-year-old woman who is admitted for nutritional evaluation. She had a bilateral mastectomy 3 months prior to this hospital admission. Ms. Gonzales recently completed a combination treatment regimen of radiotherapy and chemotherapy. Her caregivers are concerned about her depression and suppressed immune status. What must you determine before selecting the form of supplemental nutrition for Ms.​ Gonzales? If she has a current infection If she is willing to cooperate with supplemental nutrition If she has any mechanical problems that will prevent the administration of enteral nutrition If she is on medications that would interact with parenteral nutrition

If she has any mechanical problems that will prevent the administration of enteral nutrition

A client is prescribed micafungin​ (Mycamine) for a systemic fungal infection. When instructing the client about this​ medication, what would the nurse include as possible adverse ​effects? Select all that apply. Nausea Vomiting Phlebitis Headache Pruritus

Nausea Vomiting Headache Pruritus

Ms. Wilson is too weak to move herself in​ bed; she is immobile. Why is immobility a dangerous​ condition? Immobility causes damage to blood vessels and deep tissues in those areas that adhere to the bed linens. Her skin and underlying tissues may become compressed over​ time, especially between a bone and the skin surface. Immobility causes pressure on skin​ surfaces, leading to poor circulation and oxygenation of​ tissues, and eventually to skin breakdown​ (ulceration). Because she has little body​ fat, the pressure of her weight may cause ulcers to develop on bony prominences.

Immobility causes pressure on skin​ surfaces, leading to poor circulation and oxygenation of​ tissues, and eventually to skin breakdown​ (ulceration).

Which condition does the nurse know is an indication for enteral nutrition​ therapy?

Inadequate oral intake

Which condition does the nurse know is an indication for enteral nutrition​ therapy? Inability to absorb nutrients Inadequate oral intake Complete bowel obstruction Intractable vomiting

Inadequate oral intake

Ms. Riveria came to the clinic today to discuss her lab results of 2 weeks ago. Her cholesterol was 220​ mg/dL and she had a triglyceride level of 344​ mg/dL. As the​ nurse, you explain that lifestyle changes can affect lipid levels. Which explanation about lifestyle modifications is​ best? Exercise will reduce lipid levels without other lifestyle modifications. Increase your dietary soluble fiber and decrease your intake of fats. Limit your use of tobacco until your triglyceride levels improve. Lose at least 25 pounds to eliminate the need for medication.

Increase your dietary soluble fiber and decrease your intake of fats.

David​ Flores, a​ 68-year-old carpenter, has a long history of irritable bowel syndrome and ulcerative colitis. He was recently admitted to the hospital with a severe exacerbation of his colitis that required parenteral nutrition therapy. ​Initially, a peripheral intravenous line was used until a central venous catheter was placed later. What is a risk associated with administering parenteral nutrition through the peripheral intravenous​ route? Pneumothorax Infection and phlebitis Puncture of the subclavian artery Cardiac arrythmias

Infection and phlebitis

Coagulation modifiers can alter hemostasis by four basic mechanisms:

Inhibition of specific clotting factors (anticoagulant) Inhibition of platelet action (antiplatelet) Dissolution of an existing clot (thrombolytic) Inhibition of fibrin destruction (hemostatic)

What is the primary action of a​ statin? Inhibit​ HMG-CoA reductase, decreasing the​ liver's production of cholesterol Bind to bile​ acids, increasing the excretion of cholesterol in the stool Decrease VLDL levels Prevent cholesterol from being absorbed

Inhibit​ HMG-CoA reductase, decreasing the​ liver's production of cholesterol

The nurse is assessing the extent of tunneling of a pressure ulcer on a client admitted to the wound care unit. Which assessment technique is​ appropriate? Apply sterile gloves and insert a gloved index finger until the full extent of the tunneling is reached. Use a ruler to measure the glossy appearance of the skin area involved. Insert a sterile​ cotton-tipped applicator to measure the involved area. Rotate a tongue blade into the tunneled area until resistance is met.

Insert a sterile​ cotton-tipped applicator to measure the involved area.

Melanie Baxter is a​ 56-year-old client who is being treated in your clinic for chronic hepatitis C. Ms. Baxter asks why she is prescribed interferon​ alfa-2b (Intron​ A) for hepatitis when this drug is used to treat different types of cancer. What is your best​ response? Because clinical manifestations of hepatitis C mimic those of a​ malignancy, hepatitis C responds well to interferon​ alfa-2b (Intron​ A). Interferon​ alfa-2b (Intron​ A) has a broad range of actions that help the immune system better fight both cancers and viruses. Interferon​ alfa-2b (Intron​ A) was prescribed to prevent the development of a malignancy related to hepatitis​ C, not to treat the hepatitis. Interferon​ alfa-2b (Intron​ A) is not used to treat hepatitis​ C, so you should consult your health care provider to discuss the error.

Interferon​ alfa-2b (Intron​ A) has a broad range of actions that help the immune system better fight both cancers and viruses.

A client presents to the clinic for a pelvic exam. The nurse notes multiple genital warts when preparing the client for the exam. Which medication does the nurse anticipate the health care provider will​ prescribe? Interferon​ alfa-n3 (Alferon​ N) Interferon​ beta-1a (Avonex) Interferon​ alfacon-1 (Infergen) Peginterferon​ alfa-2b (PEG-Intron)

Interferon​ alfa-n3 (Alferon​ N) Rationale Interferon​ alfa-n3 (Alferon​ N) is an interferon that has​ antiviral, antineoplastic, and​ anti-inflammatory actions; it is used to treat genital warts. Interferon​ beta-1a (Avonex) is an interferon used for multiple sclerosis. Peginterferon​ alfa-2b (PEG-Intron) is an interferon used for chronic hepatitis B or C. Interferon​ alfacon-1 (Infergen) is an interferon used for chronic hepatitis C.

Where is two thirds of the bodys total fluid located?

Intracellular fluid. One third in extracellular space.

What are the most common routes of administration for immunizing ​agents? Select all that apply. Subcutaneous Rectal Sublingual Intranasal Deltoid injection

Intranasal Deltoid injection Subcutaneous

The clinic nurse receives a​ mid-morning telephone call from the mother of a​ 7-year-old child who was recently diagnosed with type 1 diabetes mellitus. The mother states that her​ child's most recent​ finger-stick blood glucose level is 54​ mg/dL (normal:​ 80-120 mg/dL). Which symptom is consistent with those the nurse can expect the mother to report of the​ child? ​Hot, flushed skin Heart rate of 58 beats per minute​ (normal: 75-120​ beats/min) Fruity smell to the breath Irritability

Irritability

Mr.​ Skinner, a​ 33-year-old, fell when he was skiing and broke his left​ tibia, requiring surgery to correct the fracture. He tells the nurse he wants to​ learn, what he needs to do to heal efficiently and quickly so that he can return home because he does not have health insurance. The nurse will implement nursing interventions to help maintain skin integrity and promote wound healing. Which of the following interventions does not promote wound​ healing? Teach about nutrition and make sure he eats adequate​ protein, vitamins, and zinc. Maintain skin hygiene and prevent contamination of the wound. Keep head of the bed no higher than 30 degrees to prevent​ shearing, and also prevent friction. Encourage Mr. Skinner to drink at least​ 2,500 mL of fluids each day.

Keep head of the bed no higher than 30 degrees to prevent​ shearing, and also prevent friction.

Mrs. Welsh is an​ 84-year-old woman who is unable to ambulate and has been losing weight for the past 2 months. She has a red nonblanchable area on her left hip. Based on the​ nurse's assessment, which of the following are priorities for Mrs.​ Welsh? Treat her stage II pressure ulcer. Needs specific nursing interventions to prevent skin breakdown. Keep her off her left​ hip; reposition her every 2 hr​ (prevent further​ breakdown). Teach the importance of good​ nutrition; encourage her to eat five to six small​ meals/day. Massage the nonblanchable area on her left hip.

Keep her off her left​ hip; reposition her every 2 hr​ (prevent further​ breakdown). Teach the importance of good​ nutrition; encourage her to eat five to six small​ meals/day.

The nurse is caring for an adult client with hypomagnesemia. Which factors does the nurse recognize as causes of this ​disorder? Select all that apply. Laxative abuse Loop diuretic therapy Diarrhea and vomiting Kidney failure Magnesium supplements

Laxative abuse Loop diuretic therapy Diarrhea and vomiting Kidney failure

Which of the following drugs are fluoroquinolones or ​quinolones? Select all that apply. Levofloxacin​ (Levaquin) Ciprofloxacin​ (Cipro) Cinoxacin​ (Cinobac) Telithromycin​ (Ketek) Moxifloxacin​ (Avelox)

Levofloxacin​ (Levaquin) Ciprofloxacin​ (Cipro) Cinoxacin​ (Cinobac) Moxifloxacin​ (Avelox)

Total parenteral nutrition​ (TPN) is ordered for an adult client with a severe gastrointestinal disorder. The nurse expects that the solution will contain which ​nutrients? Select all that apply. Lipids Antibiotics Trace minerals Electrolytes Amino acids

Lipids Trace minerals Electrolytes Amino acids

A client receiving antifungal medication has a bowel movement of​ clay-colored stools. This finding is an indication of toxicity for which body​ organ? Kidneys Brain Liver Heart

Liver

When caring for a client who is undergoing antituberculosis drug​ therapy, the nurse should assess carefully for signs​ of: Difficulty swallowing Increased intracranial pressure Liver dysfunction Esophageal reflux

Liver dysfunction

Your​ client, Mrs. Valya​ Nelson, has been taking celecoxib​ (Celebrex) for three years for rheumatoid arthritis. She recently​ self-medicated with 800 mg of ibuprofen​ (Advil) twice a day for pain. Mrs. Nelson reports being chronically tired. Her eyes and skin are jaundiced. What adverse effect do you​ suspect? Kidney disease Low potassium Liver dysfunction High cholesterol

Liver dysfunction

What should be assessed prior to the administration of a medication for​ hyperlipidemia? Urinalysis Abdominal ultrasound Complete blood count Liver function tests

Liver function tests

Prostaglandins provide several beneficial effects. Which beneficial effects are reduced when aspirin​ (Bayer) is​ used? Select all that apply. Protection from stomach acid Blood flow to the kidneys Clotting of blood Reduction of inflammation Maintenance of bronchial smooth muscle

Maintenance of bronchial smooth muscle Protection from stomach acid Blood flow to the kidneys Clotting of blood

Which​ health-related issues may require administration of supplemental ​nutrition? Select all that apply. Malnutrition Bulimia Weakened immune status Major trauma Pneumonia

Malnutrition Bulimia Weakened immune status Major trauma

Cheryl Jones brings her​ 6-month-old infant to your clinic for a​ well-child checkup and immunizations. Mrs. Jones asks if the baby will have any side effects from the immunizations. How should you​ respond? Many children experience​ malaise, fever,​ irritability, and poor appetite. You may see redness and tenderness at the injection site but not a severe systemic response. Adverse effects from immunizations are uncommon. ​Don't be concerned if your child becomes very sleepy or has labored breathing.

Many children experience​ malaise, fever,​ irritability, and poor appetite.

Identify the drugs that cause hyperglycemia or hypoglycemia. ​Instructions: Use the dropdown menus in the left column to select the effect of each drug in the right column. Effect Drug Name

May Cause Hyperglycemia Corticosteroids May Cause Hypoglycemia ACE inhibitors May Cause Hyperglycemia Phenytoin​ (Dilantin) May Cause Hypoglycemia Alcohol May Cause Hyperglycemia Diuretics May Cause Hypoglycemia ​Beta-adrenergic blockers

Which pharmacologic agent is the only one available to treat a female client who is experiencing hair​ loss? Clotrimazole​ (Lotrimin) Isotretinoin​ (Sotret) Minoxidil​ (Rogaine) Finasteride​ (Propecia)

Minoxidil​ (Rogaine)

Which nursing action is associated with ongoing antiprotozoan drug​ therapy? Monitoring daily weights Monitoring for CNS toxicity Obtaining C​ & S test results Conducting a complete health history

Monitoring for CNS toxicity

Which category of immunosuppressants is developed by first injecting human T​ cells, B​ cells, or thymocytes into​ animals? Antimetabolites Cytotoxic agents Calcineurin inhibitors Monoclonal antibodies

Monoclonal antibodies

Electrolytes affect multiple functions in the body, including:

Muscle function Neurologic activity Water balance regulation Bone formation

Your​ client, Karen​ Larsh, is diagnosed with herpes zoster​ (shingles). She is prescribed acyclovir​ (Zovirax) as a topical ointment for treatment. Which common adverse effect of the medication may​ occur? Conjunctivitis Alopecia Nausea Fever

Nausea

Ms. Adams is a​ 48-year-old female. She is being discharged after undergoing a hysterectomy. Her wound dehisced​ (when a surgical wound bursts or splits open along the suture​ line), and she had to go back to surgery to have the wound repaired. She will be going home tomorrow. The nurse will teach her and her partner​ (Sue) how to complete dressing changes at home. Which of the following will they need to learn​ and/or review to promote continued healing of the​ wound? Select all that apply. Need for increased fluid intake Signs and symptoms of wound infection Importance of hand hygiene and principles of asepsis with dressing changes Nutritional​ support, that​ is, the need for adequate​ protein, carbohydrates, and vitamins How to prevent pressure ulcers

Need for increased fluid intake Signs and symptoms of wound infection Importance of hand hygiene and principles of asepsis with dressing changes Nutritional​ support, that​ is, the need for adequate​ protein, carbohydrates, and vitamins

Which conditions may require the infusion of a ​colloid? Select all that apply. Myocardial infarction Neonatal hemolytic disease Liver failure Shock Pulmonary edema

Neonatal hemolytic disease Liver failure Shock

Samantha King is evaluated for a severe​ flare-up of psoriasis. She is taking oral cyclosporine​ (Neoral) in addition to her topical agent. Which common adverse effect do you recognize will be important to monitor her​ for? Hypotension Nephrotoxicity Hypokalemia Hepatotoxicity

Nephrotoxicity

Mrs. Dorothy​ Washington, a​ 65-year-old African American​ female, comes to your clinic for a​ follow-up visit. She has been taking an oral hypoglycemic drug for 90 days. You evaluate for drug effectiveness. Which result is an appropriate therapeutic​ response? Weight loss of 5 pounds Fewer episodes of diabetic ketoacidosis Glucose levels of 160​ mg/dL Normal hemoglobin A1c level

Normal hemoglobin A1c level

Sodium

Normal levels: 135-145 mEq/L. Kidneys are main organ that regulate balance of sodium. When the level of sodium increases in the extracellular fluid, osmolality increases. Because water moves from an area of low solute concentration to an area of high solute concentration, water is "pulled" to where the sodium is located. (A good way to remember this concept is the phrase, "salt sucks.") When sodium intake does not equal sodium output, an imbalance occurs:

What term is used for infections acquired in the​ hospital? Resistant Mutated Nosocomial Transmitted

Nosocomial

What statement is most accurate about the administration of topical corticosteroids for a client with​ psoriasis? Notify the healthcare provider of any striae. Creams and lotions should be left on for at least 24 hours. After​ application, do not apply any occlusive dressings. Topical corticosteroids stain clothing and have a foul odor.

Notify the healthcare provider of any striae.

A client is diagnosed with a fungal infection. Before medications are started to treat this​ infection, the nurse would need to do​ what? Obtain a specimen of the organism Monitor for jaundice Monitor intake and output​ (I&O) Assess for hypokalemia

Obtain a specimen of the organism

Pulmonary Embolism

Occurs when a thrombus breaks free, travels to the pulmonary vascular system, and occludes arterial circulation to the lungs.

Identify the type of fungal infection based on the disease process. ​Instructions: Use the dropdown menus in the left column to select the type of fungal infection for each disease process in the right column. Type Disease Process

Opportunistic Infections Mucormycosis Community-Acquired Infections Blastomycosis Opportunistic Infections Candidiasis Community-Acquired Infections Sporotrichosis Opportunistic Infections Aspergillosis Community-Acquired Infections Histoplasmosis Opportunistic Infections Cryptococcosis Community-Acquired Infections Coccidioidomycosis

What are common clinical manifestations of fluid volume​ deficit? Select all that apply. Orthostatic hypotension Tachycardia Nystagmus Decreased capillary refill Headache

Orthostatic hypotension Tachycardia Decreased capillary refill

A client is experiencing manifestations of a nonmalarial protozoan infection. The nurse would prepare to administer which medication that is effective against all of the major types of nonmalarial protozoan​ infections? Metronidazole​ (Flagyl) Tinidazole​ (Tindamax) Iodoquinol​ (Yodoxin) Paromomycin sulfate

Paromomycin sulfate

Which two factors affect a ​micro-organism​'s ability to cause a bacterial​ infection? Pathogenicity and virulence Susceptibility and immune response Invasiveness and toxicity Cellular structure and sensitivity

Pathogenicity and virulence

Which of the following types of antibiotics affect the bacterial cell ​wall? Select all that apply. Penicillins Cephalosporins Tetracyclines Carbapenems Aminoglycosides

Penicillins Cephalosporins Carbapenems

The nurse is caring for a client with swallowing difficulties who is expected to require enteral nutrition for 6dash-8 weeks. What type of tube would the nurse expect to be used for this​ client?

Percutaneous endoscopic gastrostomy tube

The nurse is educating an adult client who is newly diagnosed with diabetes mellitus. The teaching includes information about substances that can raise blood glucose levels. Which​ medication(s) can produce hyperglycemic​ effects? Select all that apply. ​Angiotensin-converting enzyme​ (ACE) inhibitors Phenytoin​ (Dilantin) Diuretics ​Beta-adrenergic blockers Corticosteroids

Phenytoin​ (Dilantin) Diuretics Corticosteroids

A client asks the nurse to help him understand what a lipid profile is. What will the nurse include in the teaching regarding the types of lipids found in the ​body? Select all that apply. Phospholipids Lipoproteins ​HMG-CoA reductase Steroids Triglycerides

Phospholipids Steroids Triglycerides

When caring for a client who is prescribed a​ sulfonamide, the nurse should assess for which adverse ​effects? Select all that apply. Photosensitivity Hypoglycemia Headache Dehydration Bleeding time

Photosensitivity Hypoglycemia Bleeding time

The nurse is caring for a homeless older adult client who was admitted for severe malnourishment. The client​'s history is negative for​ kidney, liver, and lung​ disease, and he has no digestive issues. Which type of enteral formula should the nurse expect to see​ ordered? Polymeric Oligomeric Modular Specialized

Polymeric

The nurse is caring for a homeless older adult client who was admitted for severe malnourishment. The client​'s history is negative for​ kidney, liver, and lung​ disease, and he has no digestive issues. Which type of enteral formula should the nurse expect to see​ ordered? Oligomeric Modular Polymeric Specialized

Polymeric

Mrs. Maria​ Gomez, a​ 40-year-old Hispanic​ woman, arrives at your clinic for her annual physical exam. She is 5 feet tall and weighs 170 pounds. Vital signs are BP​ 130/80 mmHg, P 88​ bpm, R 20 per​ min, and T​ 99°F (37.2°C). You suspect type 2 diabetes​ mellitus, based on elevated blood glucose levels. Which​ sign/symptom supports this​ possibility? Cloudy urine sample Weight gain of 10 pounds this month ​Cool, clammy skin Poor skin turgor to the sternum

Poor skin turgor to the sternum

Fibrinolysis

Process by which the body removes the clots. (After its stopped bleeding) Initiated within 24-48 hours of clot formation.

The nurse is developing a plan of care for a female​ client, Ms.​ Sate, who has a stage II pressure ulcer. Which of the following is an appropriate goal for Ms.​ Sate? Promote wound healing by secondary intention Apply appropriate devices on the bed to keep joints mobile Maintain skin integrity Prevent impaired skin integrity

Promote wound healing by secondary intention

The nurse is developing a plan of care for a female​ client, Ms.​ Sate, who has a stage II pressure ulcer. Which of the following is an appropriate goal for Ms.​ Sate? Promote wound healing by secondary intention Apply appropriate devices on the bed to keep joints mobile Maintain skin integrity Prevent impaired skin integrity

Promote wound healing by secondary intention Your answer is correct.

Which statement is true about​ protozoa? Protozoa are​ single-celled organisms. Protozoa do not usually cause death. Protozoa are easily repelled by a healthy immune system. Protozoa are all structured similarly.

Protozoa are​ single-celled organisms.

Which adverse effects associated with antifungal therapy require notification of the health care ​provider? Select all that apply. Pruritus Decreased urinary output Sudden weight gain Peripheral numbness Edema

Pruritus Decreased urinary output Sudden weight gain Edema

When the immune system chronically produces​ cytokines, what autoimmune conditions may ​result? Select all that apply. Malignant melanoma Psoriasis Rheumatoid arthritis Hepatitis C Crohn disease

Psoriasis Rheumatoid arthritis Crohn disease

The nurse sees a health care provider order to apply a hydrocolloid dressing to a client with an infected pressure ulcer. Which is the most appropriate nursing​ action? Apply a transparent dressing to the area instead of the hydrocolloid. Implement the dressing change procedure as ordered. Question the health care provider​'s order. Administer pain medication prior to applying the hydrocolloid dressing.

Question the health care provider​'s order.

Identify the type of drug action for each medication. ​Instructions: Use the dropdown menus in the left column to select the type of drug action for each dug name in the right column. Drug Action Drug Name

Rapid Acting Insulin aspart​ (NovoLog) Short Acting Insulin regular​ (Humulin R) Rapid Acting Insulin lispro​ (Humalog) Long Acting Insulin glargine​ (Lantus) Rapid Acting Insulin glulisine​ (Apidra) Long Acting Insulin detemir​ (Levemir) Intermediate Acting Isophane insulin​ (Humulin N)

The nurse is explaining to an adult client about the class of medications called​ carbapenems, which are being​ prescribed, and the side effects of the antibiotic vancomycin hydrochloride​ (Vancocin). Which side effects will the nurse want to include in the ​discussion? Select all that apply. Ototoxicity ​bleeding/bruising Rash Nausea ​"Red Man​ Syndrome" Vaginitis and superinfections

Rash Nausea ​"Red Man​ Syndrome" Vaginitis and superinfection Ototoxicity

The nurse is caring for a client with a treated wound. What would the nurse be looking for as she assesses the​ wound? How the wound is immobilized Redness and swelling Location and extent of tissue damage A foreign object

Redness and swelling This is the correct answer.

When the osmolality of plasma increases, what is triggered in addition to the thirst mechanism?

Release of antidiuretic hormone.

The nurse is reviewing with parents the expected outcomes of antibiotic therapy for their child. What are appropriate expected outcomes of antibiotic therapy for a ​child? Select all that apply. Report any increase in signs and symptoms related to superinfections. Parents will verbalize an understand the side effects of the medications. The client may stop taking the medications when symptoms disappear. The client should finish all medications as prescribed. Parents will report a decrease in signs and symptoms of illness.

Report any increase in signs and symptoms related to superinfections. Parents will verbalize an understand the side effects of the medications. The client should finish all medications as prescribed. Parents will report a decrease in signs and symptoms of illness.

The nurse plans to administer medications to a client with hyperlipidemia. Which nursing actions will be included in the ​assessment? Select all that apply. Assess effectiveness of the medication 30 to 60 minutes after administration. Review liver function tests and lipid profiles. Review the​ client's medical chart to clearly determine the​ client's medical diagnosis. Use several methods to accurately and appropriately identify the client. Monitor baseline vital signs and weigh the client.

Review liver function tests and lipid profiles. Review the​ client's medical chart to clearly determine the​ client's medical diagnosis. Use several methods to accurately and appropriately identify the client. Monitor baseline vital signs and weigh the client.

The nurse is reviewing the pathophysiology of the immune system. Which condition may occur in clients who have continual secretion of the immune response​ chemicals? Diabetes Peptic ulcers Hypertension Rheumatoid arthritis

Rheumatoid arthritis

Which are serious health consequences that can result from lack of proper ​nutrients? Select all that apply. Risk for infection Severe diarrhea Nausea and vomiting Muscle wasting Decreased wound healing

Risk for infection Muscle wasting Decreased wound healing

When administering a vaccination to a​ client, what should the nurse ​consider? Select all that apply. Route of administration Anaphylaxis and adverse reactions Precautions and contraindications Time of last food intake Dose and timing

Route of administration Anaphylaxis and adverse reactions Precautions and contraindications Dose and timing

Which statements are appropriate to include in a teaching plan for a client with a parasitic ​infestation? Select all that apply. Seal items that cannot be washed in plastic for at least 2 weeks. Apply eyedrops for dry eyes. Wash linens in hot water. Wash the face with nonoily soap. Vacuum frequently.

Seal items that cannot be washed in plastic for at least 2 weeks. Wash linens in hot water. Vacuum frequently.

For which conditions is benzocaine​ (Americaine, Anbesol) ​indicated? Select all that apply. Infant teething pain Abscess or boils ​Second-degree burns Canker sores Mosquito bites

Second-degree burns Canker sores Mosquito bites

What instructions should be included in the client education for​ lipid-lowering agents? Go to the emergency department if any adverse effects are experienced. Diet and lifestyle modifications are unnecessary but encouraged. Take these medications first thing in the morning on an empty stomach. See the health care provider for periodic lipid and liver profiles.

See the health care provider for periodic lipid and liver profiles.

Ms. Peterson has slid down toward the foot of her bed and is unable to move herself back up in the bed. You call the nursing assistant to help you move her back up to the top of the bed. If you try to move Ms. Peterson by​ yourself, you will slide her body​ up, and her body weight​ (and moisture on the​ skin) will cause the skin on her sacrum to stick to the linens and not move while the underlying tissues and muscles do move. This movement may cause​ _________ on her sacrum and deep tissues. Pressure An abrasion Friction Shearing force

Shearing force

The nurse is assessing the skin of a newly admitted​ 70-year-old client when a rash is noted on the trunk of the body. What​ condition, caused by a virus but preventable by​ vaccination, does the nurse​ suspect? Psoriasis Rheumatoid arthritis Shingles Atopic dermatitis

Shingles

Mr. Li asks if there is anything that he needs to be aware of while taking azathioprine​ (Imuran). As the​ nurse, what do you recognize as one of the most important clinical manifestations to watch​ for? Signs and symptoms of any infection ​Raised, red rash on the torso Development of progressive tooth decay Progressive muscle weakness

Signs and symptoms of any infection Your answer is correct.

A client is admitted with sporotrichosis. On which body systems would the nurse focus when assessing this ​client? Select all that apply. Lungs Skin Brain Lymph nodes Blood vessels

Skin Lymph nodes

Shan Jung recently had a radical neck resection for throat cancer. He has a nasogastric tube and is receiving bolus enteral feedings. Mr. Jung complains of nausea and bloating. He asks you if there is anything that can be done to relieve the discomfort that accompanies his feedings. What should you tell Mr. Jung you will​ do? Test him for Clostridium difficile. Add 8 ounces of water to each feeding. Add fiber to the nutritional supplement. Slow the rate of administration.

Slow the rate of administration.

Common electrolytes include:

Sodium Potassium Calcium Chloride Magnesium Phosphate

Results indicating fluid volume deficit.

Sodium increased. Serum osmolality increased. Hematocrit increased. Urine specific gravity increased. Central venous pressure decreased.

The nurse on a surgical unit is assessing the incision of a client who underwent an exploratory laparotomy. Which incision description indicates a normal inflammatory​ response? Pink​ skin, separation, and pain ​Drainage, pallor, and pain ​Odor, necrosis, and hot to touch ​Redness, edema, and warmth to touch

​Redness, edema, and warmth to touch

You are completing an admission assessment of your​ client, Ms.​ Dean, who is 31 years old. You note an open area on the skin over her left trochanter that measures 2 cm by 2 cm. The open area is a deep crater with undermining of adjacent tissue. There is no exposure of​ bone, muscle, or tendons. Based on this assessment​ data, the nurse determines this pressure ulcer is​ a: Stage II pressure ulcer Stage I pressure ulcer Stage III pressure ulcer Stage IV pressure ulcer

Stage III pressure ulcer

Mr.​ Richards, a​ 51-year-old client who is confined to a​ wheelchair, has a large wound on his sacrum with full thickness tissue​ loss; bone,​ tendons, and muscle are​ exposed; eschar is present on the lower half and proximal edges of the wound. What stage is this​ ulcer? Stage III Stage IV Stage II Stage I

Stage IV

Ms.​ Glenn, a​ 23-year-old female, was riding with her parents in their car when she was 5 years old. During a​ collision, she was not restrained in a child seat and went through the windshield in the impact. She has been a paraplegic since the accident. She has suffered only a few pressure ulcers on her coccyx over the past 17 years because she knows what to look for and what are considered preulcer signs and symptoms. Sort the following characteristics of pressure ulcers according to the stage in which they belong.

Stage of Ulcer Characteristics Stage I Pressure Ulcer Red area warmer or cooler than surrounding skin Stage II Pressure Ulcer May have intact or ruptured​ serum-filled blister Stage III Pressure Ulcer May have slough Stage IV Pressure Ulcer Full thickness tissue loss with exposed bone Stage I Pressure Ulcer Skin firm Stage II Pressure Ulcer No slough or bruising Stage III Pressure Ulcer Subcutaneous fat may be visible Stage I Pressure Ulcer Most commonly localized over bony prominences Stage II Pressure Ulcer Presents with​ red?pink wound bed Stage III Pressure Ulcer May have slough Stage IV Pressure Ulcer Slough or eschar present Stage I Pressure Ulcer Skin intact Stage II Pressure Ulcer Partial thickness loss of dermis Stage III Pressure Ulcer Full thickness tissue​ loss, bone not exposed Stage I Pressure Ulcer May have pain on palpation Stage II Pressure Ulcer Presents as​ shallow, open ulcer Stage IV Pressure Ulcer ​Bone, tendon,​ and/or muscle exposed Stage I Pressure Ulcer Skin​ reddened, does not blanch Stage II Pressure Ulcer May appear as shiny or dry shallow ulcer Stage IV Pressure Ulcer Extends into muscle​ and/or supporting structures

Before administering isotretinoin​ (Amnesteem, Claravis,​ Sotret), what client education will the nurse ​perform? Select all that apply. Submitting to pregnancy tests prior to and during treatment is required. Notify your healthcare provider of any mood changes. For male​ clients, birth control is optional. Optimum results can take up to 2 weeks to be seen. When going​ outside, wear sunscreen.

Submitting to pregnancy tests prior to and during treatment is required. Notify your healthcare provider of any mood changes. When going​ outside, wear sunscreen.

The nurse is preparing an oral dose of a sulfonamide antibiotic for an adult male client with a urinary tract infection. Which describes the action of sulfonamide​ medications? Sulfonamides only work on the kidney. Sulfonamides are folic acid inhibitors. Sulfonamides are considered a specific antibiotic. Sulfonamides are calcium inhibitors.

Sulfonamides are folic acid inhibitors.

The nurse is preparing to administer total parenteral nutrition​ (TPN) to an older adult client. Because TPN is a hypertonic​ solution, where must the catheter tip of the central venous line be​ positioned? Select the correct answer choice below. Aorta Superior Vena Cava Pulmonary Artery Inferior Vena Cava

Superior Vena Cava

Which factor increases a client​'s risk for developing a fungal​ infection? Suppressed immune system Break in skin integrity ​Pre-existing bacterial infection Overexposure to sunlight

Suppressed immune system

A client with arthritis has been prescribed acetylsalicylic acid​ (aspirin [Bayer]). Which information should the nurse teach the client about the ​drug? Select all that apply. Swallow the aspirin with a full glass of​ water, milk, or food. Use ear plugs for ringing in the ears. Avoid using alcohol while taking aspirin. Expect black stools as a side effect. Stop using aspirin 7 to 14 days prior to any scheduled surgery.

Swallow the aspirin with a full glass of​ water, milk, or food. Avoid using alcohol while taking aspirin. Stop using aspirin 7 to 14 days prior to any scheduled surgery. RATIONALE: Aspirin should be administered with adequate amounts of​ water, milk, or food to decrease the incidence of gastrointestinal irritation. Aspirin should be stopped prior to surgery and dental work to decrease the chance of excessive bleeding. Alcohol combined with aspirin increases the chance of stomach irritation. Any ringing in the ears or difficulty with hearing should be reported to the health care provider for possible intervention.​ Black, tarry stools are an adverse effect of aspirin and should be reported to the health care​ provider, because they could indicate bleeding in the gastrointestinal tract.

Signs of fluid imbalance.

Tachycardia. Orthostatic hypotension. Decreased skin turgor. Decreased capillary refill. Decreased urine output. Increased urine specific gravity. Dry mouth. Altered mental status.

The​ client, a​ 35-year-old woman with a history of rheumatoid arthritis​ (RA), was recently diagnosed with hypertriglyceridemia. A statin has been prescribed. When teaching the client about taking this​ medication, which instructions will the nurse​ include? When taking​ statins, diet and lifestyle changes are not necessary. It is safe to take statins while also taking immunosuppressants. Take the statin medication in the evening for optimal results.

Take the statin medication in the evening for optimal results.

The nurse has a specific responsibility to educate clients about antibiotic therapy. What specific instruction regarding fluid intake should be included when educating an adult client about​ fluoroquinolones? Take with milk Take with a full glass of water Take with coffee Taking with alcohol is acceptable

Take with a full glass of water

During the health​ history, a client with a helminthic infection tells the nurse about routinely eating steak tartare. The nurse suspects the client is experiencing which type of​ infection? Ascariasis Enterobiasis Tapeworm Hookworms

Tapeworm

Which drug is a​ broad-spectrum antibacterial medication in the pharmacologic class of folic acid​ inhibitors? Nitrofurantoin​ (Furadantin) Methenamine mandelate​ (Mandelamine) Methenamine hippurate​ (Hiprex) ​Sulfamethoxazole-trimethoprim (Bactrim)

​Sulfamethoxazole-trimethoprim (Bactrim)

Identify the class of helminthic infections based on the description. ​Instructions: Use the dropdown menus in the left column to select the class of helminthic infections for each description in the right column. Helminthic Infections Description

Tapeworms Acquired by eating raw​ beef, pork, or fish Roundworms Ascaris lumbricoides Hookworms Enter through skin Roundworms Nematodes Tapeworms Cestodes Roundworms Most often seen in children aged​ 3-8 Hookworms Necator americanus Tapeworms May be asymptomatic Hookworms Ancylostoma duodenale

The client asks the nurse to teach him about lifestyle changes that will help lower his blood lipid levels. Which strategies will give the client clear guidance for reducing ​hyperlipidemia? Select all that apply. Teach the client how to read nutrition labels to make better choices based on​ fat, cholesterol, and fiber content. Encourage the client to decrease consumption of plant lipids. Encourage the client to begin monitoring his tobacco use. Teach the client the differences between soluble and insoluble fiber so that he can choose foods that will increase his soluble fiber intake. Emphasize the importance of exercising most days of the week and maintaining a healthy body weight.

Teach the client how to read nutrition labels to make better choices based on​ fat, cholesterol, and fiber content. Teach the client the differences between soluble and insoluble fiber so that he can choose foods that will increase his soluble fiber intake. Emphasize the importance of exercising most days of the week and maintaining a healthy body weight.

A client is experiencing a nonacute fluid volume deficit after walking to a nearby clinic for an appointment on a very warm summer day. The client feels slightly thirsty but does not feel lightheaded or have other problems. The nurse monitors the client​'s blood pressure and finds it to be slightly low​ (100/72 mmHg). To efficiently and comfortably bring the client​'s fluid volume back to a more normal​ level, which intervention would the nurse​ implement? IV administration of a hypertonic solution Teaching the client to drink approximately​ 2,500 mL of water per day Inserting a feeding tube and administering fluids via the feeding tube IV administration of an isotonic solution

Teaching the client to drink approximately​ 2,500 mL of water per day

The nurse is developing a care plan to prevent skin breakdown. Which body fluid does the nurse recognize as the least likely to cause skin​ excoriation? Excessive saliva Gastric juices Perspiration Tears

Tears

A client is diagnosed with tinea pedis. Which antifungal medication is available to treat this client​'s superficial fungal​ infection? Clotrimazole​ (Mycelex-G) Terbinafine hydrochloride​ (Lamisill) Ketoconazole​ (Nizoral) Fluconazole​ (Diflucan)

Terbinafine hydrochloride​ (Lamisill)

The nurse is caring for a client who is receiving enteral nutrition through a feeding​ tube, and is experiencing diarrhea. What actions should the nurse ​take? Select all that apply. Increase the concentration of the feeding. Increase the infusion rate. Test for Clostridium difficile. Add fiber to the nutritional supplement. Administer loperamide​ (Imodium), an​ antidiarrheal, as ordered.

Test for Clostridium difficile. Add fiber to the nutritional supplement. Administer loperamide​ (Imodium), an​ antidiarrheal, as ordered.

What does the nurse need to know before administering oral enteral ​nutrition? Select all that apply. That the client will need less direct nursing supervision That the client is able to swallow That the client will adhere to the feeding plan That a central line must be placed That a gastrostomy tube must be in place first

That the client will need less direct nursing supervision That the client is able to swallow That the client will adhere to the feeding plan

Hemostasis

The body process that begins to stop the blood flow

The nurse on the orthopedic unit is caring for a client with bone infection secondary to an open fracture of the right leg. The client was found in a field following a tornado injury. Which best explains the cause of the​ infection? The introduction of bacteria when the dressings were changed Inadequate nutrition to boost immunity The orthopedic surgeon​'s lack of sterility when treating the fracture The break in the skin caused by the​ open-fracture injury

The break in the skin caused by the​ open-fracture injury

A client was diagnosed with hyperlipidemia one month ago. Which client actions indicate that the client has a good understanding of how to lower lipid​ levels? The client plans to lose at least​ 10% to​ 20% of her body weight within 6 months. The client adheres to a​ low-calorie diet. The client can verbally express how to​ self-administer her own​ medication, indicates the correct​ dose, and describes the most common side effects of the drug. The client has started to exercise by walking at a very fast pace once per week.

The client can verbally express how to​ self-administer her own​ medication, indicates the correct​ dose, and describes the most common side effects of the drug.

The nurse is administering blood products to a client who was admitted to the emergency department following a motor vehicle crash. Which assessment findings indicate adverse reactions to the blood ​product? Select all that apply. 1) The nurse notes that the client is developing slurred speech. Your answer is correct. The client reports that his​ face, neck, and upper chest are red and feel warm. 2) The client reports that he has to urinate. 3) The client reports that he feels anxious and is beginning to have difficulty breathing. 4) The client reports that he is feeling lightheaded and​ "itchy."

The client reports that his​ face, neck, and upper chest are red and feel warm. The nurse notes that the client is developing slurred speech. The client reports that he feels anxious and is beginning to have difficulty breathing. The client reports that he is feeling lightheaded and​ "itchy."

The nurse is educating a female client who continues to have frequent urinary tract infections. When the nurse is explaining the anatomy of the urinary tract and why females get more UTIs than​ males, which information should be included in client ​education? Select all that apply. The ureters are part of the urinary tract. Urine enters the bladder from the kidneys and stays there until released. The urethra may become inflamed or infected. The female urethra is considerably shorter compared to the male urethra. The anus is in close proximity to the urethra.

The female urethra is considerably shorter compared to the male urethra. The anus is in close proximity to the urethra.

Hemodynamics

The flow of blood through the body.

After reviewing this​ client's chart, what statement about the​ client's condition would the nurse identify as being most​ accurate? Click the three Exhibit links below for additional information about the client. EXHIBIT History LOADING... Physical Assessment LOADING... Vital Signs LOADING... Treatment with IV antibiotics is indicated. The likely causative agent is Staphylococcus aureus. Instruct the client that the antibiotics will be administered until the infection looks better. The abscess will be treated initially with topical antibiotics until a culture can be obtained.

The likely causative agent is Staphylococcus aureus.

Joseph Hirschbaum is planning a trip to Africa to participate in an international meeting. His health care provider prescribes chloroquine phosphate​ (Aralen) prior to his scheduled departure. When teaching Mr. Hirschbaum about the medication​ regimen, what information is important to​ include? If the medication regimen is completed prior to​ travel, there is minimal risk of developing malaria. This medication will prevent transmission of malaria from contaminated drinking water and food. The medication should be started immediately after the onset of any fever or chills. The medication should be taken​ 1-2 weeks before travel and continued for at least 4 weeks after returning home.

The medication should be taken​ 1-2 weeks before travel and continued for at least 4 weeks after returning home.

Which statements about the treatment of herpes simplex virus with acyclovir​ (Zovirax) are ​accurate? Select all that apply. The ointment should be applied directly to the open lesion with a glove. PO acyclovir​ (Zovirax) is indicated for treatment of acute outbreaks. Administration of acyclovir will prevent spread of the virus. The client should report any​ headaches, as they may be an adverse reaction. Administration is recommended for 2 weeks to eradicate the disease.

The ointment should be applied directly to the open lesion with a glove. The client should report any​ headaches, as they may be an adverse reaction.

Gary Levoi is an​ HIV-positive client who presents to the clinic for​ follow-up after his sputum culture tested positive for Mycobacterium tuberculosis. The health care provider prescribes isoniazid​ (INH), 300 mg by mouth daily for 6​ months; rifampin​ (Rifadin), 600 mg by mouth daily for 6​ months; and pyrazinamide​ (PZA), 2 g by mouth daily for 2 months. Mr. Levoi asks why he needs to take so many medications for such a long time. What knowledge is the correct basis of your​ response? Mr.​ Levoi's HIV status makes him susceptible to tuberculosis​ recurrence, making prolonged treatment more necessary than for other clients. The medications are prescribed together for Mr. Levoi as prophylaxis against tuberculosis infection. The pathogen that causes tuberculosis has a very thick mycolic acid layer that makes penetration by antibiotics difficult. After the symptoms from the infection​ resolve, there is no possibility of recurrence.

The pathogen that causes tuberculosis has a very thick mycolic acid layer that makes penetration by antibiotics difficult.

Which statements about rosacea are ​accurate? Select all that apply. Rosacea is classified as an infectious condition. Clients with rosacea often report itching and redness. The rhinophyma that often occurs with rosacea can be distressing to clients. Rosacea is​ self-limiting, and typically does not require treatment. Rosacea is more common in women than in men.

The rhinophyma that often occurs with rosacea can be distressing to clients. Rosacea is more common in women than in men.

Which statements about cholesterol are ​true? Select all that apply. There is no need for dietary​ cholesterol, because the liver is able to synthesize it. Cholesterol is a vital component of plasma membranes. Cholesterol has a definite role in the development of atherosclerosis. Cholesterol is a building block for amino​ acids, vitamin​ D, and estrogen. The body needs large amounts of cholesterol.

There is no need for dietary​ cholesterol, because the liver is able to synthesize it. Cholesterol is a vital component of plasma membranes. Cholesterol has a definite role in the development of atherosclerosis.

A client is being treated for systemic mycosis. What does this infection suggest to the nurse about the client​'s health​ status? The client was exposed to unsanitary conditions. The client has a cat in the house. The​ client's immune system is suppressed. The client was bitten by a mosquito.

The​ client's immune system is suppressed.

What statement about the clinical manifestations of skin disorders is most​ accurate? ​Erythema, urticaria, and pruritus are common adverse effects of systemic medications. Redness definitively indicates an infectious process. Pruritus is a definitive symptom of a skin infection. Urticaria and pruritus can occur together in a hypersensitivity response.

Urticaria and pruritus can occur together in a hypersensitivity response.

The charge nurse is giving a pharmacology lecture on bacterial infections to new employees. Which factors that affect a​ microorganism's ability to cause infection could the nurse include in the ​discussion? Select all that apply. The​ microorganism's invasiveness How quickly the organism can multiply The​ organism's release of exotoxins The​ microorganism's shape The ability of the microorganism to avoid the​ body's defenses

The​ microorganism's invasiveness How quickly the organism can multiply The​ organism's release of exotoxins The ability of the microorganism to avoid the​ body's defenses

Mr.​ Johnson, an​ 89-year-old male, lives with his daughter and her family. Which of the following changes commonly associated with​ aging, if present in Mr.​ Johnson, may indicate a greater risk for skin ​breakdown? Select all that apply. Thinning of the epidermis and decreased skin elasticity Digestive and metabolic issues​ (the prevalence of gastrointestinal problems rises with​ age) Changes in hearing​ (50% of those 85 and older have hearing​ loss) Decreased pain sensation Loss of lean body mass and a decrease in venous and arterial blood flow

Thinning of the epidermis and decreased skin elasticity Decreased pain sensation Loss of lean body mass and a decrease in venous and arterial blood flow

When Ms. Baxter asks how often she will need to take interferon​ alfa-2b (Intron​ A), what should your response​ be? Once a week Once a day Three times per week Once every other week

Three times per week

Two types of coagulation disorders

Thromboembolic, abnormal clotting or too much clotting. Coagulation, abnormal bleeding or not enough clotting.

An adult client diagnosed with Crohn disease is receiving total parenteral nutrition​ (TPN). The nurse will need to monitor for which mechanical​ complication?

Thromboembolism

An adult client diagnosed with Crohn disease is receiving total parenteral nutrition​ (TPN). The nurse will need to monitor for which mechanical​ complication? Thromboembolism Bone demineralization Refeeding syndrome Fluid volume overload

Thromboembolism

Which skin disorders are types of fungal ​infections? Select all that apply. Tinea capitis Ringworm Acne Chickenpox Athlete​'s foot

Tinea capitis Ringworm Athlete's foot

Which protozoan infection is acquired through cat feces and places a fetus at risk for stillbirth or​ abortion? Trypanosomiasis Toxoplasmosis Trichomoniasis Leishmaniasis

Toxoplasmosis Your answer is correct.

Goal of Drug Therapy for CV disorders

Treat: Hypertension Heart failure Coronary heart disease Cardiac dysrhythmias

Which treatments would the nurse identify as most appropriate for this​ client? Ethinyl estradiol​ (Estinyl) Tretinoin​ (Retin-A) Education on face washing Calcipotriene​ (Dovonex) Amcinonide​ (Cyclocort)

Tretinoin​ (Retin-A) Education on face washing

Identify the use for each type of feeding tube. ​Instructions: Use the dropdown menus in the left column to select the use of each feeding tube in the right column. Use Feeding Tube

Tubes for Short-Term Use Nasogastric Tubes for Long-Term Use Percutaneous endoscopic jejunostomy Tubes for Short-Term Use Nasoduodenal Tubes for Long-Term Use Percutaneous endoscopic gastrostomy Tubes for Short-Term Use Nasojejunal

A nurse is preparing a discharge plan for a client with a risk of skin breakdown. What would the nurse include in the discharge ​plan? Select all that apply. Turn and reposition at least every 2 hour. Position to prevent pressure on bony prominences. Diet should be adequate in​ fluids, protein, vitamins B and​ C, iron, and calories. Massage the bony prominences. If persistent redness​ occurs, apply lotion to the area.

Turn and reposition at least every 2 hour. Position to prevent pressure on bony prominences. Diet should be adequate in​ fluids, protein, vitamins B and​ C, iron, and calories.

Identify the type of immune response based on the description. cell-mediated immune response OR humoral immune response? t cells produce cytokines b lymphocytes become plasma cells cytokines kill antigens plasma cells produce immunoglobulins t lymphocytes are cloned b cells are cloned cytokines strength macrophages ANTIbodies code antigens for elimination

Type Description Cell-Mediated Immune Response T cells produce cytokines Humoral Immune Response B lymphocytes become plasma cells Cell-Mediated Immune Response Cytokines kill antigens Humoral Immune Response Plasma cells produce immunoglobulins Cell-Mediated Immune Response T lymphocytes are cloned Humoral Immune Response B cells are cloned Cell-Mediated Immune Response Cytokines strengthen macrophages Humoral Immune Response Antibodies code antigens for elimination

Ms. Glenn has a wound and quickly identifies what is happening with her skin. Sort the following examples and characteristics of the three different types of healing patterns.

Type of Healing Characteristics Primary Intention Healing Minimal scarring Secondary Intention Healing Longest healing time Tertiary Intention Healing May have unresolved edema or infection Primary Intention Healing Wound closed with tissue adhesive Secondary Intention Healing Greatest scarring Tertiary Intention Healing Wound left open for 3 to 5 days Primary Intention Healing Minimal or no tissue loss Secondary Intention Healing Healing of pressure ulcer Tertiary Intention Healing Surgical wound left open to drain abscess Primary Intention Healing Minimal granulation tissue Secondary Intention Healing More susceptible to infection Primary Intention Healing Tissue surfaces approximated Tertiary Intention Healing Wound closed with sutures or staples Primary Intention Healing Closed surgical wound

What are the goals of pharmacotherapy for clients who are taking medications for high ​cholesterol? Select all that apply. Understanding​ self-administration of the drug Weight loss Decreased levels of lipids Knowledge of the​ drug's adverse effects Lack of adverse effects

Understanding​ self-administration of the drug Decreased levels of lipids Knowledge of the​ drug's adverse effects Lack of adverse effects

The nurse is caring for an emaciated older adult client who is immobile and is experiencing frequent watery stools. Which technique would the nurse use to protect the client​'s skin​ integrity? Wipe the soiled skin firmly with a towel. Use a gel and foam combination mattress. Place the client in Fowler​'s position in bed. Use a​ firm, circular motion to cleanse the sacral area.

Use a gel and foam combination mattress.

An antibiotic is usually effective against pathogens with similar classifications. Which characteristics can be used to classify ​bacteria? Select all that apply. Use of oxygen Cell wall structure Moisture Color Shape

Use of oxygen Cell wall structure Shape

Parenteral nutrition may be administered by which​ route? Via tube feeding Via the oral route Via central intravenous access Via the gastrointestinal system

Via central intravenous access

Parenteral nutrition may be administered by which​ route? Via central intravenous access Via tube feeding Via the oral route Via the gastrointestinal system

Via central intravenous access

The nurse is caring for an older adult client who is receiving total parenteral nutrition​ (TPN). What should the nurse assess to minimize ​complications? Select all that apply. Feeding tube placement Weight and lab results Intake and output Vital signs Catheter placement

Weight and lab results Intake and output Vital signs Catheter placement

Which statements about the use of permethrin​ (Acticin, Elimite,​ Nix) are ​accurate? Select all that apply. The medication must stay on the scalp for 10 minutes prior to rinsing. When permethrin​ (Acticin, Elimite,​ Nix) is used​ correctly, nit removal with a comb is unnecessary. When rinsing the​ hair, take care to avoid the eyes. The shampoo may not be used on children under 2 months of age. If itching persists after the initial treatment of​ scabies, a medication such as crotamitron​ (Eurax) may be necessary.

When rinsing the​ hair, take care to avoid the eyes. The medication must stay on the scalp for 10 minutes prior to rinsing.

A client on the oncology unit has received the drug filgrastim​ (Neupogen). The nurse is reviewing the laboratory reports. Which laboratory value indicates that the drug has been​ effective? White blood cells​ (WBCs) 5500 ​µL ​(normal: 4500 to​ 10,000) Hemoglobin 14​ g/dL (normal: 13.5dash-18 ​g/dL) Hematocrit​ 38% (normal: ​40%dash-​54%) Potassium 3.5​ mEq/L (normal: 3.5 ​mEq/Ldash-5.5 ​mEq)

White blood cells​ (WBCs) 5500 ​µL ​(normal: 4500 to​ 10,000)

The nurse is assessing an adult client who has been prescribed antibiotics for an infection. What signs and symptoms of a superinfection will the nurse include when educating the client before ​discharge? Select all that apply. Fine red rash White patches in mouth Vaginal discharge ​Foul-smelling feces Sore mouth

White patches in mouth Vaginal discharge ​Foul-smelling feces Sore mouth

A client sustained a​ right-wrist strain following a fall. Prior to applying the order ACE​ wrap, the nurse notes a superficial abrasion. Which nursing asessment should be completed prior to applying the ACE​ wrap? Pain on a scale from 1 to 10 Client​'s ability to reapply the dressing Wound drainage Adequacy of the circulation in the right arm

Wound drainage

Which sources may cause fungal ​infections? Select all that apply. AIDS Soil Yeasts Molds Mushrooms

Yeasts Molds Mushrooms

Low-molecular-weight heparins (LMWHs)

a newer class of medications that work similar to heparin. Their mechanism of action is to inhibit clotting factor X. LMWHs are the drugs of choice for DVT prevention: LMWHs have a number of advantages. These medications: Have a duration of action that is two to four times longer than heparin Produce a more stable response than heparin Require fewer follow-up laboratory tests May be administered at home Are less likely to cause thrombocytopenia

Which​ anti-inflammatory drug inhibits only one type of​ cyclooxygenase? Ecotrin Nuprin Motrin Celebrex

celebrex

Thromboembolic disorder

describes conditions in which the body forms unwanted clots. (classified and venous or arterial)

Antiplatelet drugs

have an anticoagulant effect by inhibiting platelet aggregation. -prevent arterial clot formation rather than venous clot formation as the anticoagulants do. four types of antiplatelet drugs: Aspirin ADP receptor blockers Glycoprotein IIb/IIIa receptor inhibitors Drugs for intermittent claudication, such as cilostazol (Pletal) and pentoxifylline (Pentoxil, Trental)

Thrombocytopenia

occurs when platelet levels drop below 150,000 mm3.

When administering a recommended dose of acetaminophen​ (Tylenol) to an adult​ client, what results can you​ expect? Reduction of pain Few adverse effects No progression of inflammation Reduction of inflammation Reduction of fever

reduction of PAIN few adverse effects reduction of FEVER

Which signs are associated with aspirin​ (Bayer) toxicity? Select all that apply. Tinnitus Decreased hearing Stomach discomfort Inflammation of joints Blood clots

tinnitus decreased hearing stomach discomfort

What is the only type of lipid that serves as an energy​ source? Triglycerides Cholesterol Phospholipids Steroids

triglycerides

Hypertonic crystalloids:

~Examples—hypertonic saline (3% NaCl), 5% dextrose in normal saline, 5% dextrose in lactated Ringer's solution, 5% dextrose in Plasma-Lyte 56 ~Increases blood plasma osmolality and volume ~Treatment for dehydration in clients with hypertonic plasma ~Treatment for cerebral edema

Hypotonic crystalloids:

~Examples—hypotonic saline (0.45% NaCl), Plasma-Lyte 56 ~Decrease serum osmolality ~Treatment for hypernatremia and cellular dehydration

Isotonic crystalloids:

~Examples—normal saline (0.9% NaCl), lactated Ringer's solution, Plasma-Lyte 148, 5% dextrose in water (D5W), 5% dextrose in 0.2% saline ~Administered to increase fluid volume of the vascular system ~Frequently used to replace fluid loss due to vomiting and diarrhea ~Treatment for sodium deficiency (isotonic saline solutions)

The nurse is caring for a child with pneumonia and fever of 100​°F ​(38.1​°​C). The child​'s parent asks the​ nurse, "Why doesn​'t the doctor have something ordered for the temperature before it gets to 101​ degrees?" Which is the nurse​'s best​ response? ​"Your doctor does not want to give your child unnecessary​ medication." ​"A little fever is not going to hurt your​ child." ​"A low-grade fever can aid in defense and repair of the​ body." ​"Don​'t worry. We will not let your child​'s temperature get too​ high."

​"A low-grade fever can aid in defense and repair of the​ body."

Which statement by the client with pediculus humanis capitis indicates understanding of the nurse​'s ​instructions? ​"If I am unable to wash an​ item, I can spray it with an​ insecticide." ​"The other members of the household do not need to be inspected and​ treated." ​"Although lice live for only 24​ hours, I should continue inspecting for reinfection since nits can hatch after the first 24​ hours." ​"The rash on my child​'s chest may itch for up to two weeks after​ treatment."

​"Although lice live for only 24​ hours, I should continue inspecting for reinfection since nits can hatch after the first 24​ hours."

A client who is taking a prescription antiprotozoan medication is experiencing​ vomiting, flushing, and a headache. What would the nurse ask when assessing this​ client? ​"Are you washing your hands as​ instructed?" ​"Are you skipping any doses of the​ medication?" ​"Are you sharing any personal items with other​ people?" ​"Are you drinking any​ alcohol?"

​"Are you drinking any​ alcohol?"

A client is diagnosed with a helminthic infection. What would the nurse ask this client when determining the origin of this​ infection? ​"Have you recently been in a country or area with poor​ sanitation?" ​"Have you recently experienced a mosquito or tick​ bite?" ​"Do you have cats at​ home?" ​"Have you recently been exposed to​ molds, mushrooms, or​ yeast?"

​"Have you recently been in a country or area with poor​ sanitation?"

You interview Mrs. Gomez for additional signs and symptoms of diabetes. Which statement by the client most likely suggests that complications of this disease may be​ present? ​"I have to get up all night because I have a dry​ cough." ​"I haven't had much of an appetite​ lately." ​"I get pain in my calves when I walk up the​ stairs." ​"I feel dizzy when I get up in the​ morning."

​"I get pain in my calves when I walk up the​ stairs."

The nurse is assessing a recently admitted male client who has osteoarthritis and hypertension. The record indicates that the client has been taking​ high-dose aspirin. Which client comment causes the nurse the most​ concern? Click the three Exhibit links below for additional information about the client. EXHIBIT Vital Signs LOADING... Home Medications LOADING... Testing LOADING... ​"I need to speak with the doctor when he​ arrives." ​"I have been having trouble hearing for the past 2​ weeks." ​"My arthritis makes me​ stiff, especially in the​ morning." ​"I usually take my scheduled medication with​ food."

​"I have been having trouble hearing for the past 2​ weeks." THIS IS AN ADVERSE EFFECT

The​ 58-year-old male client with​ Crohn's disease​ (chronic inflammation of the digestive​ tract) was admitted because of hyperlipidemia. His nurse has provided education regarding his prescribed medications. Which client response indicates that more teaching is​ needed? ​"If I develop an ear​ infection, which usually happens each​ winter, I need to be careful about taking the statin medication along with the antibiotic that is usually​ prescribed." ​"The nurse said I need to take this medication in the evening because the liver produces more cholesterol during the evening and night​ hours." ​"I have to take this medication for the next 12​ months, no matter​ what, even if I have to begin taking the immunosuppressants again. I will just do​ it." ​"I need to call the health care provider if I begin to have severe muscle or joint​ pains, or if my other health care provider prescribes medications for my​ Crohn's disease​ (chronic inflammation of the digestive​ tract)."

​"I have to take this medication for the next 12​ months, no matter​ what, even if I have to begin taking the immunosuppressants again. I will just do​ it."

The nurse has been educating a client who will receive isotretinoin​ (Amnesteem, Claravis,​ Sotret). What statement indicates that the client understands the​ treatment? ​"I will have to get my blood sugar​ checked, as my medication can cause it to​ increase." ​"I must use two reliable forms of birth control while taking my​ medication." ​"I can donate​ platelets, but cannot donate red blood​ cells." ​"If I don​'t see results in a few​ weeks, I can just stop taking​ it."

​"I must use two reliable forms of birth control while taking my​ medication."

The nurse has performed client teaching on how to cleanse a wound. Which statement by the client indicates further instruction is​ necessary? ​"I should cleanse the wound with a cotton​ pad." ​"If the wound appears​ clean, I will not have to clean it each time I change the​ dressing." ​"I should clean in an outward​ direction." ​"I should avoid drying the wound after I clean​ it."

​"I should cleanse the wound with a cotton​ pad."

The nurse is caring for a teenaged client who is newly diagnosed with type 1 diabetes mellitus. Which statement made by the client indicates that further teaching is​ necessary? ​"I will have to take my pill just once a​ day." ​"I will eat my meals before giving myself insulin​ injections." ​"I will take my insulin as​ prescribed." ​"I must check my weight every​ day."

​"I will have to take my pill just once a​ day."

The nurse asks a client with type 2 diabetes to explain what he should do if his​ finger-stick blood glucose is 55​ mg/dL on the sliding scale.​ (The lowest blood glucose on the scale is 60​ mg/dL.) Which client response indicates that further teaching is​ necessary? ​"I will eat some hard candy and retest my blood sugar in 15​ minutes." ​"I should drink some orange juice​ immediately." ​"I will inject three units of​ insulin." ​"I have to take something to increase my blood glucose or things could get​ bad."

​"I will inject three units of​ insulin."

Which client statement indicates that the teaching the nurse provided about newly prescribed antiprotozoan medication has been​ effective? ​"I will take the medication until the symptoms​ disappear." ​"I will keep a record of adverse reactions to the​ medication." ​"I will take the complete course of the​ medication." ​"I am to expect changes in the symptoms associated with the​ infection."

​"I will take the complete course of the​ medication."

Mr. Roberts needs to apply a cold compress to a wound on his knee. The nurse is teaching Mr. Roberts and his wife about the benefits of​ cold, how to apply cold​ treatment, and for what conditions cold is useful. Which of the following responses indicates Mr. Roberts understood the teaching ​provided? Select all that apply. ​"Ice increases circulation to the area better than​ heat." ​"Ice helps to decrease​ pain; it definitely helped while I was in the​ hospital." ​"If I see that my knee becomes inflamed and​ red, using ice will help relieve​ it." ​"Cold therapy is the best bet for joint stiffness and muscle​ spasms." ​"Putting ice on my knee will help decrease the​ swelling."

​"Ice helps to decrease​ pain; it definitely helped while I was in the​ hospital." ​"If I see that my knee becomes inflamed and​ red, using ice will help relieve​ it." ​"Putting ice on my knee will help decrease the​ swelling."

The client tells the nurse he knows his cholesterol has been very high despite taking his statin medication​ regularly, eating a​ low-fat/high-fiber diet, and trying to get more exercise. He questions why the health care provider prescribed colestipol hydrochloride​ (Colestid). Which response best answers this​ client's question? ​"Bile acid resins bind to bile acids in the intestines and are excreted from the body. Bile acids are essential for​ digestion, and the body makes up for the loss by converting blood cholesterol into bile acids. As a​ result, LDL​ (bad) cholesterol is removed from the​ blood, and this lowers your LDL cholesterol​ level." ​"Statins are the most effective​ cholesterol-lowering medications for reducing LDL cholesterol. Bile acid​ resins, such as colestipol hydrochloride​ (Colestid), are often combined with​ statins." ​"The main adverse effects for bile acid resins are gastrointestinal​ complaints, particularly​ constipation." ​"Sometimes a second medication is needed to lower cholesterol to a safe level. The new medication will help your body get rid of cholesterol through your​ stools."

​"Sometimes a second medication is needed to lower cholesterol to a safe level. The new medication will help your body get rid of cholesterol through your​ stools."

The client asks the nurse to teach him about cholesterol in more detail. Which responses will the nurse include in the ​teaching? Select all that apply. ​"The body uses cholesterol to produce estrogen and progesterone hormones in women and testosterone in​ men." ​"There is no dietary need for cholesterol because the liver is able to synthesize​ it." ​"Cholesterol has minimal value to the​ body's health." ​"Many people are surprised to learn that cholesterol is a steroid produced by the​ body." ​"Too much cholesterol contributes to hardening of the​ arteries, which is also known as​ atherosclerosis."

​"The body uses cholesterol to produce estrogen and progesterone hormones in women and testosterone in​ men." ​"There is no dietary need for cholesterol because the liver is able to synthesize​ it." ​"Many people are surprised to learn that cholesterol is a steroid produced by the​ body." ​"Too much cholesterol contributes to hardening of the​ arteries, which is also known as​ atherosclerosis."

Mr. Jerome​ Johnson, a​ 55-year-old African American​ male, is diagnosed with diabetes and prescribed metformin​ (Glucophage). Mr. Johnson​ asks, "Will this medication make me feel​ bad?" Which statement is a correct​ response? ​"You may have a dry cough from​ bronchospasms, but it will go away in a few​ weeks." ​"You will feel bad if you stay out in the sun too​ long, because of an increased risk of​ sunburn." ​"The most common side effects are​ diarrhea, flatulence, and​ nausea, but taking the drug with food will​ help." ​"The most common side effects are back pain and​ flu-like symptoms."

​"The most common side effects are​ diarrhea, flatulence, and​ nausea, but taking the drug with food will​ help."

The nurse sent a wound specimen for culture and sensitivity​ testing, after which the health care provider ordered​ imipenem-cilastatin, an antibiotic from the carbapenem class. The client​ asks, "I usually take penicillin. Why​ aren't I taking that​ now?" What is the​ nurse's best​ response? ​"This class of antibiotic is a​ broad-spectrum drug and is often used to treat infections until we can identify the specific organism that is causing the​ infection." ​"Since you have been taking penicillin in the​ past, we want to avoid any​ allergies; therefore, a different class of antibiotic will be best for you to prevent this from​ happening." ​"The penicillin drugs are much older​ drugs, and the newer ones are often much​ better." ​"This type of antibiotic is much cheaper and is just as effective as​ penicillin."

​"This class of antibiotic is a​ broad-spectrum drug and is often used to treat infections until we can identify the specific organism that is causing the​ infection."

Which important teaching statement should you provide before a client starts a treatment regimen of isotretinoin​ (Claravis)? ​"Be sure to notify the school nurse about your​ treatment." ​"I am going to show you how to effectively use a nit​ comb." ​"We need to rule out pregnancy at least 14 days before treatment​ starts." ​"Report a weight change of 2 or more pounds to the healthcare​ provider."

​"We need to rule out pregnancy at least 14 days before treatment​ starts."

Mr. Donald​ Walker, a​ 53-year-old White​ male, takes a​ beta-adrenergic blocker. Because Mr. Walker is a newly diagnosed​ diabetic, the healthcare provider adds glyburide​ (DiaBeta) to the drug regimen. As the​ nurse, what teaching should you​ provide? ​"Do not be concerned if your skin turns​ yellow." ​"Finger-stick glucose testing is not accurate when taking multiple​ drugs." ​"You may have two alcoholic beverages per​ day." ​"You should keep a quick source of sugar on​ hand."

​"You should keep a quick source of sugar on​ hand."

The nurse is assessing a client with tonsillitis. The client asks the nurse why the tissues in the neck seem swollen. Which nursing response is best? ​"Your lymph nodes and tissues sometimes swell in attempts to fight​ infection." ​"The swelling is a direct effect of histamine being released throughout the​ body." ​"T cells are activated in the neck​ area, which causes the neck to​ swell." ​"The neck area contains proteins that collect and cause the​ swelling."

​"Your lymph nodes and tissues sometimes swell in attempts to fight​ infection."

The orthopedic nurse educator is preparing a​ post-conference seminar on osteomyelitis for a group of nurses. Which client is identified to have the greatest risk of developing this​ condition? ​60-year-old with​ full-thickness tissue loss and subcutaneous fat visible ​35-year-old paraplegic with tissue loss extending through the muscle ​22-year-old with gastroenteritis and redness in the perianal area ​50-year-old with paralysis and redness on the sacrum

​35-year-old paraplegic with tissue loss extending through the muscle

The nurse administers 20 units of isophane insulin​ (Humulin N) at​ 7:00 a.m. to a client with​ insulin-dependent diabetes mellitus. What time should the nurse observe the client for signs of​ hypoglycemia? ​3:00 p.m. ​1:00 a.m. ​8:00 a.m. ​10:00 a.m.

​3:00 p.m.

Which clinical manifestation is an early sign of hepatotoxicity related to systemic antifungal​ therapy? ​Clay- or​ orange-colored stools Hypotension Elevated BUN and creatinine levels Petechiae

​Clay- or​ orange-colored stools

The nurse in the emergency department is caring for a​ 45-year-old client with a history of type 2 diabetes mellitus. Which clinical manifestations are consistent with those the nurse can expect to​ observe? Select all that apply. ​Client's report of increased food intake Fasting blood glucose 165​ mg/dL ​Client's complaint of a​ "pins and​ needles" sensation in the upper extremities Fluid overload ​Client's report of weight gain

​Client's report of increased food intake Fasting blood glucose 165​ mg/dL ​Client's complaint of a​ "pins and​ needles" sensation in the upper extremities

Current studies indicate that thrombolytic therapy is of no value if administered after​ when? A) 1 hour of onset of MI symptoms B) 10 hours of onset of MI symptoms C) 2 hours of onset of MI symptoms D) 24 hours of onset of MI symptoms

​D Thrombolytics, which are used to dissolve blood​ clots, are most effective when administered between 20 minutes and 12 hours after the onset of MI symptoms.​ Ideally, they should be given within 30 minutes or less. According to current​ studies, thrombolytic therapy is of no value if administered more than 24 hours after the onset of MI.

The nurse is assessing a diabetic client who has a blood glucose level of 799​ mg/dL (normal:​ 60-100 mg/dL), and is unresponsive. What finding leads the nurse to suspect diabetic​ ketoacidosis? Hypertension Bradycardia Decreased potassium level ​Rapid, deep respirations

​Rapid, deep respirations


Ensembles d'études connexes

AWS Cloud Practitioner Exam - Official Practice Sets

View Set